Edisi Guru
N
ATA K G N I T
Dw
4
a s ibaha
Sebabnya Modul Ini Menitikberatkan Penggunaan Unit S.I. Pada Setiap Langkah Penghitungan Dalam Fizik • Sebagai ahli Fizik, penggunaan unit amat penting. Unit memberikan makna kepada nombor dalam setiap penghitungan dalam Fizik. • Nilai nombor menjadi berbeza apabila unit yang berlainan digunakan, (misalnya, 9.2 m dan 9.2 mm mewakili panjang yang berbeza). • Unit merupakan sebahagian penting dalam 'bahasa Fizik' yang kita sering gunakan. Unit mesti disebut dengan jelas apabila kuantiti fizik dikemukakan. Unit dapat 'menceritakan kisah Fizik'. • Unit menggambarkan konsep fizik dengan lebih jelas. Unit merupakan blok-blok pembinaan dalam Fizik. • Penggunaan unit dalam pengiraan membolehkan murid mengelakkan kesilapan secara automatis. • Dengan menggunakan unit secara teliti dan lengkap dalam pengiraan, seseorang murid akan mencapai kejayaan dalam peperiksaan Fizik dan juga membina batu asas dalam pendidikan Fizik. Walau bagaimanapun, cikgu-cikgu yang mengajar berhak untuk memilih sama ada untuk menggunakan unit dalam pengiraan setiap langkah atau mengikuti format peperiksaan SPM di mana murid hanya perlu meletakkan unit yang betul pada akhir jawapan sahaja.
FIZIK • SIZE CLOSE: W8.5’’ x H11‘’
Edisi Guru
Nilam Publication Sdn. Bhd. (919810-T)
Tingkat 1, No. 35, Jalan 5/10B, Spring Crest Industrial Park 68100 Batu Caves, Selangor, Malaysia. Tel: 03 - 6178 0132 • Fax: 03 - 6185 2402 www.nilampublication.com •
[email protected] Hak Cipta terpelihara. Tidak dibenarkan mengeluarkan mana-mana bahagian dalam buku ini dalam apa juga bentuk atau cara, sama ada secara elektronik atau mekanikal tanpa kebenaran bertulis daripada pihak Nilam Publication Sdn. Bhd. © Nilam Publication Sdn. Bhd. (919810-T), 2014 Dicetak oleh Pramaju Sdn. Bhd. No. 35, Jalan 5/10B Spring Crest Industrial Park 68100 Batu Caves Selangor Darul Ehsan
Muatturun softcopy dari www.nilampublication.com atau email kepada
[email protected]
Dengan Ingatan Tulus Ikhlas daripada Nilam Publication Sdn. Bhd. CHONG: 013-311 1713 (WM) ALVIN GOH: 012-392 7921 (EM)
KANDUNGAN CONTENTS
UNIT
1
UNIT
Pengenalan kepada Fizik Introduction to Physics
2
Daya dan Gerakan
UNIT
Daya dan Tekanan
3
UNIT
4
UNIT
5
00 FIZ Form 4 pre 1LP.indd 1
Forces and Motion
Forces and Pressure
Haba Heat
Cahaya Light
1
19
112
136
159
9/10/13 4:42 PM
MODUL • Fizik TINGKATAN 4
UNIT
1
PENGENALAN KEPADA FIZIK INTRODUCTION TO PHYSICS
1.1 KUANTITI ASAS DAN KUANTITI TERBITAN
BASE QUANTITIES AND DERIVED QUANTITIES
• Menerangkan kuantiti asas dan kuantiti terbitan
Explain what base quantities and derived quantities are
U N I T
• Menyenaraikan kuantiti asas dan unitnya List base quantities and their units
• Menyenaraikan kuantiti terbitan dan unitnya List some derived quantities and their units
• Mengungkapkan kuantiti terbitan dan unitnya dalam bentuk kuantiti asas dan unit asas Express derived quantities as well as their units in terms of base quantities and base units
1
1.2 IMBUHAN DAN NOTASI SAINTIFIK (BENTUK PIAWAI)
PREFIX AND SCIENTIFIC NOTATION (STANDARD FORM)
• Mengungkapkan kuantiti dengan menggunakan imbuhan dan notasi saintifik Express quantities using prefixes and scientific notation
• Menyelesaikan masalah yang melibatkan pertukaran unit Solve problems involving conversion of units
1.3 KUANTITI SKALAR DAN KUANTITI VEKTOR
SCALAR AND VECTOR QUANTITIES
• Menyatakan takrif kuantiti skalar dan kuantiti vektor State the definitions of scalar quantity and vector quantity
• Memberikan contoh kuantiti skalar dan kuantiti vektor Give examples of scalar quantity and vector quantity
1.4 MEASUREMENTS
PENGUKURAN
• Memilih peralatan yang sesuai untuk mengukur kuantiti fizik Choose appropriate instruments to measure physical quantities
• Menerangkan kepersisan, kejituan dan kepekaan Explain consistency, accuracy and sensitivity
• Menerangkan jenis-jenis ralat dalam eksperimen Explain types of experimental errors
• Menggunakan teknik yang sesuai untuk mengurangkan ralat dalam eksperimen Using appropriate techniques to reduce experimental errors
1
01 FIZ Form 4 Ch1 2F.indd 1
© Nilam Publication Sdn. Bhd.
9/9/13 5:44 PM
MODUL • Fizik TINGKATAN 4
KUANTITI ASAS DAN KUANTITI TERBITAN
1.1
BASE QUANTITIES AND DERIVED QUANTITIES
1 Kuantiti fizik ialah kuantiti yang boleh diukur. A physical quantity is a quantity that can be measured.
Kuantiti fizik Physical quantity
1
asas
Kuantiti
U N I T
Base
Kuantiti
terbitan
Derived
quantity
quantity
2 Kuantiti asas: ialah kuantiti fizik yang tidak boleh ditakrifkan dalam istilah kuantiti asas yang lain. A base quantity: is a physical quantity which cannot be defined in terms of other base quantities.
3 Terdapat lima kuantiti asas dalam Unit Sistem Antarabangsa (unit S.I.). There are 5 base quantities in the International System of units (S.I. units).
Kuantiti asas
Simbol untuk kuantiti asas
Unit S.I.
Simbol untuk Unit S.I.
Base quantity
Symbol for base quantity
S.I. Unit
Symbol for S.I. Unit
l
meter metre
m
m
kilogram
kg
t
saat second
s
I
ampere
A
T
Kelvin
K
Panjang Length
Mass Jisim
Masa Time
Arus elektrik Electric current
Suhu Temperature
4 Kuantiti terbitan: ialah kuantiti yang diterbitkan daripada kuantiti asas melalui pendaraban atau pembahagian atau kedua-duanya. A derived quantity: is a physical quantity which is derived from base quantities through multiplication or division or both.
© Nilam Publication Sdn. Bhd.
01 FIZ Form 4 Ch1 2F.indd 2
2
9/9/13 5:44 PM
MODUL • Fizik TINGKATAN 4
5 Tentukan unit terbitan untuk kuantiti terbitan yang berikut. Determine the derived unit for the following derived quantities. Kuantiti terbitan
Hubungan dengan kuantiti asas
Derived quantity
Relationship with base quantity
Luas Area
Isi padu Volume
Ketumpatan Density
Halaju Velocity
Pecutan Acceleration
Berat Weight
Momentum Momentum
Tekanan Pressure
Daya Force
Derived unit from base units
Luas = panjang × panjang
Area = length × length
Isi padu = panjang × panjang × panjang
Volume = length × length × m × m × m = m3 length
Ketumpatan
Density mass = length × length × length
jisim = panjang × panjang × panjang
Halaju =
sesaran masa
m × m = m2
displacement time
kg = kg m-3 m3
m = m s–1 s
Pecutan perubahan halaju = masa
Acceleration change in velocity = time
m s–1 = m s–2 s
Berat = jisim × pecutan graviti
Weight = mass × gravitational acceleration
kg m s-2
Momentum = jisim × halaju
Momentum = mass × velocity
Tekanan =
daya luas
Pressure =
Daya = jisim × pecutan
U N I T
1
Velocity =
force area
Force = mass × acceleration
3
01 FIZ Form 4 Ch1 2F.indd 3
Kuantiti terbitan dari unit asas
kg m s-1
kg m–1 s–2 ; N m–2 ; pascal (Pa)
kg m s-2
© Nilam Publication Sdn. Bhd.
9/9/13 5:44 PM
MODUL • Fizik TINGKATAN 4
Kuantiti terbitan
Hubungan dengan kuantiti asas
Derived quantity
Relationship with base quantity
Kerja Work
U N I T
1
Kuasa Power
Tenaga kinetik Kinetic energy
Tenaga keupayaan graviti Gravitational potential energy
Cas Charge
Voltan Voltage
Rintangan Resistance
© Nilam Publication Sdn. Bhd.
01 FIZ Form 4 Ch1 2F.indd 4
Kerja = daya × sesaran
Kuantiti terbitan dari unit asas Derived unit from base units
Work = force × displacement
kerja masa
kg m2 s-2 ; joule (J)
work time
kg m2 s-3 ; watt (W)
Tenaga kinetik 1 = × jisim × (halaju)2 2
Kinetic energy 1 × mass × (velocity)2 = 2
kg m2 s–2 ; joule (J)
Tenaga keupayaan graviti = jisim × pecutan graviti × tinggi
Gravitational potential energy = mass × gravitational acceleration × height
kg m2 s–2 ; joule (J)
Cas = arus × masa
Charge = current × time
A s ; coulomb (C)
Kuasa =
Voltan =
Power =
kerja cas
Rintangan =
Voltage =
voltan arus
work charge
Resistance =
voltage current
J C–1 ; volt (V)
V A-1 ; ohm (Ω)
4
9/9/13 5:44 PM
MODUL • Fizik TINGKATAN 4
IMBUHAN DAN BENTUK PIAWAI
1.2
PREFIXES AND STANDARD FORM
1 Beberapa kuantiti fizikal mempunyai nilai yang sangat kecil atau sangat besar. Some physical quantities have very small or very large values.
2 Untuk mengendalikan nombor tersebut dengan mudah, imbuhan dan notasi saintifik diwujudkan. To handle such numbers more easily, prefixes and scientific notations have been developed.
3 Imbuhan dikaitkan dengan unit S.I. untuk mengungkapkan beberapa nilai yang tertentu. Prefixes are attached to S.I. units to express these values.
4 Jadual imbuhan/Table of prefixes: Imbuhan
Simbol
Nilai
Imbuhan
Simbol
Nilai
Prefixes
Symbol
Value
Prefixes
Symbol
Value
tera
T
1012
senti/centi
c
10-2
giga
G
109
mili/milli
m
10-3
mega
M
106
mikro/micro
µ
10-6
kilo
k
103
nano
n
10-9
desi/deci
d
10-1
piko/pico
p
10-12
U N I T
1
Contoh/Examples 1 Untuk menukar dari imbuhan kepada nombor biasa, darabkan dengan nilai imbuhan itu. To convert from prefixes to normal number, multiply with the value of the prefixes.
(b) 260 mg = (a) 4 Gm = m g Giga (G) = 109 mili / milli (m) = 10-3 Jadi/Therefore, Jadi/Therefore, 4 Gm = 4 × 109 m 260 mg = 260 × 10-3 g = 0.26 g 4 000 000 000 = m 2 Untuk menukar dari nombor biasa kepada imbuhan, bahagikan dengan nilai imbuhan itu. To convert from normal numbers to prefixes, divide by the values of the prefixes.
(a) 325 s = Mega (M) = 106 Jadi/Therefore, 325 s = 325 ÷ 106 Ms 0.000325 =
(b) 12 800 000 m = km kilo (k) = 103 Jadi/Therefore, 12 800 000 m = 12 800 000 ÷ 103 km = 12 800 km
Ms
Ms
3 Untuk menukar dari satu imbuhan kepada satu imbuhan yang lain, tukarkan kepada unit yang asal, kemudian baru tukar kepada imbuhan yang diminta. To change from one prefix to another prefix, change the prefix to the original unit, then only change it to the requested prefix.
(b) 2 430 µm = cm 2 430 µm = 2 430 × 10-6 m = 0.002 43 m = 0.002 43 ÷ 10-2 cm = 0.243 cm
(a) 3 060 kg = Tg 3 060 kg = 3 060 × 103 g = 3 060 000 g = 3 060 000 ÷ 1012 Tg 0.000 003 06 = Tg 5
01 FIZ Form 4 Ch1 2F.indd 5
© Nilam Publication Sdn. Bhd.
9/9/13 5:44 PM
MODUL • Fizik TINGKATAN 4
5 Bentuk piawai boleh diungkapkan dalam A × 10n di mana 1 A 10 dan n ialah integer. Standard form can be expressed in the form A × 10n where 1 A 10 and n is an integer.
Tuliskan kuantiti fizik yang berikut dalam bentuk piawai.
Write the following physical quantities in standard form. Kuantiti fizik/Physical quantities
Bentuk piawai/Standard form
Ketumpatan aluminium = 2 700 kg m-3
2.7 × 103 kg m–3
Density of aluminum = 2 700 kg m-3
Jarak planet Merkuri dari Matahari = 57 850 000 km
5.785 × 107 km
Distance of Mercury planet from the Sun = 57 850 000 km
Kelajuan cahaya = 380 000 000 m s-1
3.8 × 108 m s–1
Speed of light = 380 000 000 m s-1
U N I T
Unit jisim atom = 0.000 000 000 000 000 000 000 000 001 66 kg Atomic mass units = 0.000 000 000 000 000 000 000 000 001 66 kg
1
Cas satu elektron = 0.000 000 000 000 000 000 16 C
1.6 × 10–27 kg 1.6 × 10–19 C
Charge of an electron = 0.000 000 000 000 000 000 16 C
6 Kuantiti fizik yang ditulis dalam suatu unit boleh ditulis dalam unit yang lain. A physical quantity that is written in a certain unit can be rewritten in another unit.
Contoh/Examples (a) 1 cm2 =
m2
(b) 80 km j–1 =
1 cm
1 cm 2
1 cm = 1 cm × 1 cm = (1 × 10–2)m × (1 × 10–2)m = 1 × 10–4 m2
m s–1
80 km 1j 80 × 103 m = 3 600 s = 22.22 m s–1
80 km j–1 =
1 j = 3 600 s
Tuliskan kuantiti fizik berikut dengan unit yang diberikan./Write the following physical quantities in the unit given. (a) 1 m2 =
1 × 104
cm2
(b) 5 m3 =
5 × 106
cm3
1 × 10–3
(c) 1 000 cm3 =
1.3
(d) Kelajuan sebuah kereta/Speed of a car 33.33 = 120 km j–1 = m s–1 (e) Ketumpatan ais/Density of ice 900 = 0.9 g cm–3 =
m3
kg m–3
KUANTITI SKALAR DAN KUANTITI VEKTOR
SCALAR QUANTITIES AND VECTOR QUANTITIES
1 Beberapa kuantiti fizik digunakan untuk menggambarkan pergerakan sesebuah objek. Some physical quantities are used to describe the motion of objects.
2 Kuantiti-kuantiti ini dapat dibahagikan kepada dua kategori: These quantities can be divided into two categories:
Kuantiti fizik Physical quantity
Kuantiti Scalar
© Nilam Publication Sdn. Bhd.
01 FIZ Form 4 Ch1 2F.indd 6
skalar
Kuantiti Vector
quantity
vektor quantity
6
9/9/13 5:44 PM
MODUL • Fizik TINGKATAN 4
3 Dua kategori ini boleh dibezakan antara satu sama yang lain melalui definisi yang jelas: These two categories can be distinguished from one another by their distinct definitions:
i. Kuantiti skalar/Scalar quantity • Seorang budak perempuan berjalan sejauh 4 meter./A girl walks 4 meters. 4 meter Tiada arah • Magnitud: Arah: Magnitude:
4 metres
Direction:
No direction
ii. Kuantiti vektor/Vector quantity • Seorang budak perempuan berjalan sejauh 4 meter ke Timur. A girl walks 4 meters East.
• Magnitud:
4 meter
Arah:
Magnitude:
4 metres
Direction:
Timur
U N I T
East
4 Kuantiti skalar merupakan kuantiti fizik yang mempunyai
magnitud
magnitude
A scalar quantity is a physical quantity which has
1
only.
Kuantiti vektor merupakan kuantiti fizik yang mempunyai kedua-dua arah . magnitude
A vector quantity is a physical quantity which has both
sahaja.
and
magnitud direction
dan
.
Contoh/Examples Pertimbangkan kuantiti-kuantiti yang disenaraikan di bawah. Kategorikan setiap kuantiti sama ada kuantiti vektor atau kuantiti skalar. Consider the following quantities listed below. Categorise each quantity as being either a vector quantity or a scalar quantity. Kuantiti/Quantity 5m 30 cm s–1, Timur/East 5 km, Utara/North 20 °C 256 bytes 4 000 kalori/calories
Kategori/Category Kuantiti skalar (tiada arah disertakan pada jarak) Scalar quantity (there is no direction listed for the distance)
Kuantiti vektor (terdapat arah disertakan pada kelajuan) Vector quantity (there is direction listed for the speed)
Kuantiti vektor (terdapat arah disertakan pada jarak) Vector quantity (there is direction listed for the distance)
Kuantiti skalar (arah tidak dikaitkan) Scalar quantity (there is no direction involved)
Kuantiti skalar (arah tidak terlibat) Scalar quantity (there is no direction involved )
Kuantiti skalar (arah tidak terlibat) Scalar quantity (there is no direction involved)
1 Contoh-contoh bagi kuantiti skalar adalah/Examples of scalar quantities are: Suhu, masa, laju, jarak, jisim/Temperature, time, speed, distance, mass. 2 Contoh-contoh bagi kuantiti vektor adalah/Examples of vector quantities are: Halaju, pecutan, sesaran, momentum, daya/Velocity, acceleration, displacement, momentum, force. 7
01 FIZ Form 4 Ch1 2F.indd 7
© Nilam Publication Sdn. Bhd.
9/9/13 5:44 PM
MODUL • Fizik TINGKATAN 4
PENGUKURAN
1.4
MEASUREMENT
1 Pengukuran adalah proses untuk menentukan nilai sesuatu kuantiti fizik dengan menggunakan alat saintifik. Measurement is the process of determining the value of a physical quantity using scientific instruments.
panjang
2 Meter (m), kilogram (kg) dan saat (s) adalah tiga unit asas untuk mengukur masa dan masing-masing.
,
The metre (m), the kilogram (kg) and .the second (s) are three basic units for measuring mass time and respectively. U N I T
1
jisim length
,
Mengukur panjang / Measuring length 1 Panjang sesuatu objek boleh diukur dengan menggunakan pembaris meter, angkup vernier atau tolok skru mikrometer. The length of an object can be measured by using a metre rule, vernier callipers or micrometer screw gauge.
2 Kesesuaian alat ini bergantung kepada panjang yang diukur dan ketepatan yang diperlukan. The suitability of the instrument depends on the length to be measured and the accuracy required. Alat pengukur
Senggatan terkecil
Measuring instrument
Smallest scale division
Pembaris meter / Meter rule
0.1 cm
Angkup vernier / Vernier callipers
0.01 cm
Tolok skru mikrometer / Micrometer screw gauge
0.01 mm
(Senggatan terkecil pada alat pengukur menunjukkan kepekaan sesuatu alat) (The smallest scale division on the measuring instruments shows the sensitivity of the instruments.)
(a) Pengukuran menggunakan pembaris meter Measurement using a metre rule
Pembaris meter memberikan bacaan dalam ketepatan 0.1
A metre rule gives readings to an accuracy of
0.1
cm.
cm.
Latihan/Exercises Ukur dan tuliskan setiap yang berikut./Measure and write each of the following. (i)
Lebar/Width = Panjang/Length =
(ii)
cm
2
3
4
5
7.0 cm
6
6.2 cm – 1.0 cm = 5.2 cm
Berdasarkan rajah, panjang objek =
Based on the diagram, the length of the object =
© Nilam Publication Sdn. Bhd.
01 FIZ Form 4 Ch1 2F.indd 8
1
1.6 cm
6.2 cm – 1.0 cm = 5.2 cm
8
9/9/13 5:44 PM
MODUL • Fizik TINGKATAN 4
(iii)
Pensel/Pencil
Sesiku Set square
1
cm
2
3
4
5
6
7
8
Berdasarkan gambar rajah, diameter pensel =
Based on the diagram, the diameter of the pencil =
9
5.0 cm – 3.5 cm = 1.5 cm 5.0 cm – 3.5 cm = 1.5 cm
(b) Pengukuran dengan menggunakan angkup vernier/Measurement by using vernier callipers 1 Angkup vernier digunakan untuk mengukur/A pair of Vernier callipers is used to measure: (b) dimensi luar / outer dimensions (c) dimensi dalam / inner dimensions (a) kedalaman/ depth 2 Angkup vernier memberikan bacaan dalam ketepatan
0.01 0.01
A pair of Vernier callipers gives readings to an accuracy of
cm
Ekor
Skru pemutar
Tail
Screw
Skala utama
Skala utama
Main scale
Main scale
Skala vernier
cm 0
1
2 0
Vernier scale
3 5
Skru pemutar
10
Screw
Ekor
Skala vernier
Rahang luar
Tail
Vernier scale
Outside jaw
Rahang dalam
Rahang luar
Inside jaw
Outside jaw
3 Angkup vernier mempunyai dua skala:
skala utama
dan
skala vernier
main scale
and
vernier scale
A pair of Vernier callipers has two scales:
4 (i) Panjang skala vernier =
0.9
cm/Length of vernier scale = 10 bahagian
(ii) Skala vernier boleh dibahagikan kepada
The vernier scale is divided into
10
Length of the smallest scale division of the vernier scale =
(iv) Panjang untuk senggatan terkecil pada skala utama =
Length of the smallest scale division of the main scale =
0.9
cm
divisions
(iii) Panjang untuk senggatan terkecil oleh skala vernier =
1
cm.
Rahang dalam Inside jaw
U N I T
0.09 cm 0.09 cm
0.1 cm 0.1 cm
(v) Perbezaan antara senggatan terkecil oleh skala utama dan senggatan terkecil oleh skala vernier
= 0.1 cm – 0.09 cm = 0.01 cm The difference between the smallest scale division of the main scale and the smallest scale division of the vernier scale = 0.1 cm – 0.09 cm = 0.01 cm
9
01 FIZ Form 4 Ch1 2F.indd 9
© Nilam Publication Sdn. Bhd.
9/9/13 5:44 PM
MODUL • Fizik TINGKATAN 4
Latihan/Exercises Apakah bacaan yang ditunjukkan oleh angkup tersebut?/What are the readings shown by the callipers? 1 2 cm
1
cm
2 0
0
2
3 5
0
10
10
Bacaan/Reading = (1.6 + 0.08) cm
Bacaan/Reading = (1.2 + 0.06) cm 1.26 cm =
U N I T
1
5
1
1.68 cm
=
(c) Ukuran dengan menggunakan tolok skru mikrometer Measurement using a micrometer screw gauge Spindal
Kunci
Anvil
Racet
Bidal (skala vernier)
Sleeve (skala utama)
Spindle
Lock
Anvil
Ratchet knob
Thimble (vernier scale)
Sleeve (main scale)
Sleeve (skala utama) Sleeve (main scale)
Spindal Spindle
mm 01
Anvil Anvil
40 35 30 25
Bidal (skala vernier) Thimble (vernier scale)
Kunci Lock
Racet Ratchet knob
1 Tolok skru mikrometer digunakan untuk mengukur objek yang sangat kecil sehingga 0.01 mm 0.01 (Ketepatan dalam mm)
A micrometer screw gauge is used to measure very small objects to 0.01 mm. (Accuracy of
2 (a) Skru dalam spindal mempunyai jarak benang atau senggatan
0.01
0.01
mm.
When the thimble is turned one complete rotation, the sliding jaw moves a distance of
3 (a) Skala bidal mempunyai
The scale on the thimble has
50
0.50
mm. mm.
divisions.
One division on the thimble scale represents
© Nilam Publication Sdn. Bhd.
0.50
bahagian (senggatan).
(b) Satu bahagian di atas skala bidal menunjukkan
01 FIZ Form 4 Ch1 2F.indd 10
50
mm)
mm.
(b) Apabila bidal dipusing satu pusingan lengkap, rahang bergerak dalam jarak
The screw in the thimble has a thread distance or pitch of
0.01
0.01 mm .
0.01 mm
.
10
9/9/13 5:44 PM
MODUL • Fizik TINGKATAN 4
Latihan/Exercises Tuliskan bacaan tolok skru mikrometer yang ditunjukkan di rajah bawah. Write down the readings of the micrometer screw gauges shown in the diagrams below.
(a)
(b) mm
0 1
5
mm
0
0 1
25
0
45
20
45
40
15
1.01 mm
(c) 5
mm 0 1
1.96 mm
U N I T
1.21 mm
1
Latihan/Exercises Lengkapkan jadual di bawah./Complete the table below. (a)
Kuantiti fizik/Physical quantity
Alat pengukur/Measuring instrument
Lebar sebuah meja
Pembaris meter Metre rule
Width of the table
Diameter dalam paip air kuprum
Angkup vernier Vernier callipers
The inner diameter of copper water pipe
Ketebalan wayar
Tolok skru mikrometer Micrometer screw gauge
Thickness of a piece of wire
Jarak seorang atlit melempar
Pita pengukur Measuring tape
The distance of a javelin throw
Angkup vernier Vernier callipers
Diameter sebatang paip The diameter of a pipe
Ketebalan duit syiling 50 sen
Tolok skru mikrometer Micrometer screw gauge
Thickness of a 50 cent coin
(b)
Panjang/Length
Alat pengukur/Measuring instrument
4.5 cm
Pembaris meter/Metre rule
1.94 cm
Angkup vernier/Vernier callipers
6.72 cm
Angkup vernier/Vernier callipers
3.55 mm
Tolok skru mikrometer/Micrometer screw gauge
Mengukur Jisim/Measuring Mass neraca tuas
Jisim suatu objek boleh diukur dengan menggunakan The mass of an object can be measured by using a
lever balance
or a
atau
neraca tiga alur
beam balance
.
.
Mengukur Masa/Measuring Time Masa diukur dengan menggunakan
jam randik
./Time is measured by using a 11
01 FIZ Form 4 Ch1 2F.indd 11
stopwatch
.
© Nilam Publication Sdn. Bhd.
9/9/13 5:44 PM
MODUL • Fizik TINGKATAN 4
1.5
KEJITUAN, KEPERSISAN DAN KEPEKAAN ACCURACY, CONSISTENCY AND SENSITIVITY
Kejituan
Accuracy
1 Kejituan dalam ukuran ialah betapa hampir sesuatu
Accuracy in measurement is the
nilai ukuran
itu
degree of closeness of a measurement
value to the
U N I T
1
.
actual value .
kejituan yang tinggi
2 Ukuran yang mempunyai peratusan ralat yang sangat kecil mempunyai
nilai sebenar
.
higher accuracy .
A measurement with a smaller percentage error has a
Kepersisan
Consistency
1 Kepersisan ialah kemampuan sesuatu alat pengukur untuk mengukur kuantiti dengan sedikit atau tiada sisihan relatif dalam bacaan yang diperoleh. Consistency is the ability of a measuring instrument to measure in a consistent manner with little or no relative deviation in the readings obtained. The repeated measurement will produce almost the same reading.
2 Pengukuran dengan sisihan relatif yang lebih kecil mempunyai kepersisan yang lebih tinggi. A measurement with a smaller relative deviation has a higher consistency.
Latihan/Exercises Dalam pertandingan menembak, tiga orang peserta A, B dan C masing-masing melepaskan enam tembakan pada sasaran. Bandingkan kejituan dan kepersisan tiga penembak ini. In a shooting competition, three participants A, B and C each takes six shots at a target. Compare the accuracy and the consistency of the three shooters.
A
B
C
Jitu dan persis
Persis tetapi tidak jitu
Tidak jitu dan tidak persis
Accurate and consistent
Consistent but not accurate
Not accurate and not consistent
Kepekaan Sensitivity
1 Kepekaan ialah kebolehan sesebuah alat untuk mengesan kuantiti fizik yang diukur. Sensitivity is the ability of an instrument to detect
© Nilam Publication Sdn. Bhd.
01 FIZ Form 4 Ch1 2F.indd 12
small changes
perubahan yang kecil
dalam sebarang
in the physical quantity being measured.
12
9/9/13 5:44 PM
MODUL • Fizik TINGKATAN 4
2 Alat pengukur yang lebih sensitif dapat A more sensitive measuring instrument is able to
(a) mengesan perubahan yang sangat kecil dalam kuantiti fizik yang diukur
detect very small changes in the physical quantity that is being measured
(b) bertindak balas dengan cepat kepada perubahan dalam kuantiti fizik yang diukur
respond more quickly towards changes in the physical quantity that is being measured
lebih kecil
3 Alat pengukur yang mempunyai senggatan A measuring instrument which has
smaller
adalah lebih peka.
scale divisions is more sensitive.
Contoh/Examples
U N I T
Senaraikan perbezaan antara dua alat pengukur yang berikut. List the differences between these two measuring instruments. Pembaris meter/A meter rule cm 0
1
2
3
1
Pita pengukur/A measuring tape cm 0
4
1
2
3
Pembaris meter
Pita pengukur
A measuring tape
10
2
0.1 cm
0.5 cm
Tinggi/High
Rendah/Low
Number of divisions per cm
Senggatan skala terkecil Smallest scale division Sensitivity
5
A meter rule
Bilangan senggatan per cm
Kepekaan
4
RALAT RAWAK DAN RALAT SISTEMATIK
RANDOM ERRORS AND SYSTEMATIC ERRORS
Semua ketidaktentuan eksperimen (ralat) adalah disebabkan oleh sama ada ralat sistematik . random errors
All experimental uncertainties (errors) are due to either
or
ralat rawak systematic errors
atau .
Ralat Rawak
Random Errors
1 Ralat rawak ialah ketidakpastian dalam pengukuran yang disebabkan oleh: Random errors are uncertainties in the measurement due to:
(a) pemerhati/the observer (b) keadaan sekeliling/the surroundings (c) alat-alat/the instruments 2 Apabila kuantiti diukur untuk beberapa kali, ralat rawak akan menyebabkan bacaan menjadi lebih besar atau lebih kecil daripada bacaan sebenar. When a quantity is measured a few times, random errors will cause the readings either to be larger or smaller than the actual values.
13
01 FIZ Form 4 Ch1 2F.indd 13
© Nilam Publication Sdn. Bhd.
9/9/13 5:44 PM
MODUL • Fizik TINGKATAN 4
3 Contoh bagi ralat rawak:
Ralat paralaks Parallax error
Examples of random errors:
(a) kesilapan pengukuran yang berlaku apabila seseorang membaca bacaan skala dari kedudukan mata pada alat yang salah ( ralat paralaks ) measurement error that happens when one reads a scale’s reading from a wrong parallax error position of the eye or instrument ( )
Kedudukan mata yang betul sepatutnya berada berserenjang dengan skala . The correct position of the eye should be perpendicular to the scale .
(i)
U N I T
cm
1
2
1
Penjelasan/Explanation: (pandangan pemerhati tidak berserenjang dengan skala instrumen yang sedang dibaca) (the view of the observer is not perpendicular to the scale of the instrument that is being read) Cara yang betul untuk membaca skala pembaris ditunjukkan dalam rajah. Mata mesti diletakkan tegak di atas tanda pada skala untuk mengelakkan ralat paralaks. The correct way to read the scale of a ruler is shown in the diagram. The eye must be placed vertically above the mark on the scale to avoid parallax error. Y
X
(ii)
(iii)
Mata Eye
Mata Eye Merkuri Mercury
Air Water
(b) kesilapan yang dibuat apabila membaca skala suatu alat the error made when reading the scale of an instrument
(c) kiraan bilangan ayunan yang salah dalam sistem yang bergetar a wrong count of the number of oscillations in a vibrating system
(d) tekanan yang tidak konsisten semasa merapatkan rahang tolok skru mikrometer (Contoh: apabila mengukur diameter dawai) inconsistent pressures applied when closing the gap of a micrometer screw gauge (Example: when measuring the diameter of a wire)
(e) perubahan dalam persekitaran semasa eksperimen (Contoh: perubahan suhu yang mana ia telah dianggap malar) changes in the surroundings during an experiment (Example: the change of temperature which has been assumed to be constant)
4 Ralat rawak boleh dielakkan (atau dikurangkan) dengan mengulangi eksperimen mengira nilai purata (mengambil beberapa bacaan) dan . Random errors can be eliminated (or reduced) by repeating measurements (taking calculating the average value several readings) and .
© Nilam Publication Sdn. Bhd.
01 FIZ Form 4 Ch1 2F.indd 14
0
Z
cm
Ralat paralaks boleh menghasilkan bacaan yang lebih besar atau lebih kecil daripada nilai sebenar. Apabila kedudukan mata berada pada Parallax errors can produce readings that are bigger or smaller than the actual value. When the position of the eye is at X – bacaan akan menjadi lebih besar daripada nilai sebenar. the reading will be greater than the actual value. Z – bacaan akan menjadi lebih kecil daripada nilai sebenar. the reading will be smaller than the actual value. Y – tiada apa-apa kesilapan paralaks. there will not be any parallax errors. Seorang pemerhati yang cekap akan mendapat bacaan tanpa atau dengan ralat paralaks yang kecil. An efficient observer will get readings without or with small parallax errors. Ketiadaan ralat paralaks semasa menggunakan ammeter/voltmeter – menggunakan cermin di bawah penunjuk akan memastikan bahawa imej jarum tidak boleh dilihat semasa ukuran. No paralax when using an ammeter/voltmeter – using a mirror under the pointer will ensure that the image of the pointer cannot be seen during measurement.
14
9/9/13 5:44 PM
MODUL • Fizik TINGKATAN 4
Ralat Sistematik
Systematic Errors
1 Ralat sistematik ialah ketidakpastian dalam pengukuran disebabkan oleh: Systematic errors are uncertainties in the measurements due to:
(a) pemerhati the observer
Contoh/Example: (i) masa tindak balas (menggunakan jam randik, dll.)
reaction time (using stopwatch, etc.)
(ii) rabun jauh atau rabun dekat (eksperimen cahaya, dll)
short-sightedness or long-sightedness (light experiments, etc)
(b) persekitaran the surroundings
Contoh/Example: Andaian nilai pecutan disebabkan oleh graviti, 10 m s–2, adalah tidak tepat jika nilai g di tempat di mana eksperimen dijalankan berbeza dari 10 m s–2.
U N I T
1
Assuming the value of the acceleration due to gravity, 10 m s–2, is inaccurate if the value of g at the place where the experiment is carried out differs from 10 m s–2.
(c) alat-alat the instruments
(i) ralat sifar
zero error
Contoh/Example: • pembaris meter yang telah rosak atau haus di hujungnya a metre rule which has worn ends cm
1
2
3
• bacaan angkup vernier atau tolok skru mikrometer ditutup.
bukan sifar
the reading of vernier callipers or micrometer screw gauge is closed.
not zero
walaupun rahang telah even when the jaws are
• ammeter dan voltmeter yang tidak menunjukkan bacaan sifar walaupun terputus dari litar. an ammeter and a voltmeter which do not show zero reading even when disconnected from a circuit.
• penimbang yang jarumnya tidak menunjukkan bacaan sifar walaupun tiada objek yang diletakkan di atas penimbang itu. a balance which does not show zero reading even when no object is being placed on it.
(ii) kesalahan dalam alat
fault in the instrument
Alat-alat ditentukurkan dalam kilang dalam keadaan suhu dan tekanan atmosfera tertentu. Kadangkala alat-alat ini digunakan dalam keadaan fizikal yang berbeza. Instruments are calibrated in the factory under specific temperature and atmospheric pressure. Sometimes these instruments are used under different physical conditions.
Contoh/Example: panjang angkup vernier keluli akan berubah apabila suhu berubah. the length of a steel vernier callipers will change when temperature changes.
15
01 FIZ Form 4 Ch1 2F.indd 15
© Nilam Publication Sdn. Bhd.
9/9/13 5:44 PM
MODUL • Fizik TINGKATAN 4
2 Ralat sistematik tidak boleh dielakkan dengan mengambil beberapa bacaan menggunakan alat yang sama, pemerhati yang sama atau alat yang sama. Systematic errors cannot be eliminated by taking several readings using the same instrument, same observer or same instrument.
3 Ralat sistematik boleh dielakkan dengan cara/Systematic errors can be eliminated by berbeza different (i) menggunakan alat yang / using instruments (ii) mengambil ukuran dengan
U N I T
Zero Error for Vernier Callipers cm
0
1
(c)
1
0
5
0
cm
5
10
0
cm
0.04 cm
1
0
Ralat sifar/Zero error = –0.03 cm
5
10
(ii) Bacaan sebenar = 1.03 cm – 0.04 cm = 0.99 cm The correct value = 1.03 cm – 0.04 cm = 0.99 cm
Ralat Sifar untuk Tolok Skru Mikrometer Zero Error for Micrometer Screw Gauge
(a)
(c) mm
0
mm
5 0 45
Ralat sifar/Zero error = +0.01 mm
5 0 45
+0.01 mm
mm
0
mm 0 1
0 45 40
Ralat sifar/Zero error = –0.04 mm
© Nilam Publication Sdn. Bhd.
0 45 40
(ii) Bacaan sebenar = 1.46 mm – 0.01 mm = 1.45 mm
01 FIZ Form 4 Ch1 2F.indd 16
0
(i) Ralat sifar/Zero error =
(b)
5
10
1
(i) Ralat sifar/Zero error =
Penerangan Ralat sifar Explanation : Zero error = –0.10 cm + 0.07 cm = –0.03 cm
cm
0
1
0
0
10
Ralat sifar/Zero error = +0.04 cm
(b)
carefully
/ taking measurements
Ralat Sifar untuk Angkup Vernier
(a)
teliti
The correct value = 1.46 mm – 0.01 mm = 1.45 mm
16
9/9/13 5:44 PM
MODUL • Fizik TINGKATAN 4
1.5
KAJIAN SAINTIFIK
SCIENTIFIC INVESTIGATIONS
Proses kajian saintifik adalah seperti ditunjukkan di bawah: The process of scientific investigation is shown as below:
Pernyataan masalah
Inferens
Problem statement
Inference
Hipotesis Hypothesis
Pemboleh ubah/Variables • Pemboleh ubah dimanipulasikan Prosedur
Manipulated variable
Senarai bahan dan radas
Procedure
• Pemboleh ubah bergerak balas
List of materials and apparatus
Responding variable
• Pemboleh ubah dimalarkan Constant variable(s)
Pemerhatian
Analisis
Perbincangan
Kesimpulan
Observation
Analysis
Discussion
Conclusion
Eksperimen Experiment
U N I T
1
Bandul ringkas / Simple pendulum
Inferens Inference
Hipotesis Hypothesis
Tujuan eksperimen Aim of the experiment
Panjang bandul ringkas mempengaruhi tempoh bandul ringkas The length of a simple pendulum affects the period of the simple pendulum
Jika panjang bandul ringkas bertambah, maka tempohnya juga akan bertambah If the length of the simple pendulum increases, its period will also increase
Untuk menyiasat hubungan antara tempoh bandul ringkas dengan panjangnya To investigate the relationship between the period of a simple pendulum and its length
Pemboleh ubah dimanipulasikan/Manipulated variable: Panjang bandul, l/The length of the pendulum, l Pemboleh ubah Variables
Pemboleh ubah bergerak balas/Responding variable: Tempoh bagi bandul ringkas, T/the period of the simple pendulum, T Pemboleh ubah dimalarkan/Constant (fixed) variable: Jisim bandul/Mass of pendulum bob
Bahan dan radas Materials and apparatus
Bandul, benang berpanjangan 70 cm, kaki retort dan pengapit, dua keping syiling kecil, jam randik, pembaris meter Pendulum bob, 70-cm length of thread, retort stand and retort clamps, two small coins, stopwatch, metre rule
17
01 FIZ Form 4 Ch1 2F.indd 17
© Nilam Publication Sdn. Bhd.
9/9/13 5:44 PM
MODUL • Fizik TINGKATAN 4
(a) Semua radas disusun seperti yang ditunjukkan dalam rajah itu.
All the apparatus are set up as shown in the diagram.
(b) Panjang benang dilaraskan supaya panjang, l = 10.0 cm.
The thread is adjusted so that its length, l = 10.0 cm.
(c) Bandul diayun untuk memulakan ayunan melalui sudut yang kecil (tidak lebih daripada 10o).
Susunan radas dan prosedur U N I T
Arrangement of the apparatus and procedure
A gentle push is given to the pendulum bob to start swinging through a small angle (not more than 10o).
(d) Masa yang diambil untuk 10 ayunan lengkap, t, diambil menggunakan jam randik.
The time taken for 10 complete oscillations, t, is taken by using the stopwatch.
Benang Thread
Bandul Pendulum bob Kaki retort Retort stand
(e) Tempoh bandul ringkas, T (iaitu masa yang diambil untuk satu ayunan lengkap) dikira dengan menggunakan:
1
The period of the simple pendulum, T (i.e. the time taken for one complete oscillation) is calculated by using:
Tempoh/Period, T =
Masa untuk 10 ayunan lengkap Time for 10 complete oscillations / 10 10
(f) Langkah (c) hingga (e) diulangi untuk l = 20.0 cm, 30.0 cm, 40.0 cm, 50.0 cm dan 60.0 cm.
Steps (c) to (e) are repeated for l = 20.0 cm, 30.0 cm, 40.0 cm, 50.0 cm and 60.0 cm.
(g) Graf T melawan l diplotkan.
A graph of T against l is plotted.
Panjang, l (cm) Length, l (cm)
Masa yang diambil untuk 10 ayunan lengkap, t (s) The time taken for 10 complete oscillations, t (s)
t1
t2
tpurata/average
Tempoh bandul, T (s) The period of the pendulum, T (s)
10.0 Pengumpulan data Data collection
20.0 30.0 40.0 50.0 60.0 T (s)
Graf (menggunakan kertas graf) Graph (using graph paper) l (cm) 0
Bandul yang lebih panjang mengambil masa yang lebih panjang untuk membuat satu Kesimpulan Conclusion
ayunan lengkap (atau apabila panjang bandul ringkas bertambah, tempohnya bertambah). The longer pendulum takes longer time to make a complete oscillation (or the period of a simple pendulum increases with its length).
© Nilam Publication Sdn. Bhd.
01 FIZ Form 4 Ch1 2F.indd 18
18
9/9/13 5:44 PM
MODUL • Fizik TINGKATAN 4
UNIT
2
DAYA DAN GERAKAN FORCES AND MOTION
2.1 MENGANALISIS GERAKAN LINEAR/ANALYSING LINEAR MOTION • Menyatakan maksud jarak dan sesaran/Define distance and displacement s • Menyatakan maksud laju dan halaju dan menyatakan v = s /Define speed and velocity and state that v = t t • Menyatakan maksud pecutan dan nyahpecutan dan menyatakan a = v – u t v–u Define acceleration and deceleration and state that a =
t
• Menghitung laju dan halaju Calculate speed and velocity
• Menghitung pecutan dan nyahpecutan Calculate acceleration and deceleration
• Menyelesaikan masalah gerakan linear dengan pecutan seragam dengan
U N I T
Solve problems on linear motion with uniform acceleration using
(i) v = u + at (ii) s = ut + 1 at2 2 (iii) v2 = u2 + 2as
2
2.2 MENGANALISIS GRAF GERAKAN/Analysing motion graphs • Melakar dan mentafsirkan graf sesaran-masa dan halaju-masa Plot and interpret displacement-time and velocity-time graphs
• Membuat kesimpulan daripada bentuk graf sesaran-masa apabila jasad dalam keadaan: Deduce from the shape of a displacement-time graph when a body is: (i) rehat/at rest (ii) bergerak dengan halaju seragam/moving with uniform velocity (iii) bergerak dengan halaju tidak seragam/moving with non-uniform velocity
• Menentukan jarak, sesaran dan halaju daripada graf sesaran-masa
Determine distance, displacement and velocity from a displacement-time graph
• Membuat kesimpulan daripada bentuk graf halaju-masa apabila jasad dalam keadaan: Deduce from the shape of a velocity-time graph when a body is: (i) rehat/at rest (ii) bergerak dengan halaju seragam/moving with uniform velocity (iii) bergerak dengan pecutan seragam/moving with uniform acceleration
• Menentukan jarak, sesaran, halaju dan pecutan daripada graf halaju-masa
Determine distance, displacement, velocity and acceleration from a velocity-time graph
• Menyelesaikan masalah gerakan linear dengan pecutan seragam Solve problems on linear motion with uniform acceleration
2.3 Memahami Inersia/Understanding inertia • Menerangkan apa itu inersia Explain what inertia is
• Menghubungkaitkan jisim dengan inersia Relate mass to inertia
• Memberi contoh situasi yang melibatkan inersia Give examples of situations involving inertia
• Mencadangkan cara untuk mengurangkan kesan negatif inersia Suggest ways to reduce the negative effects of inertia
19
02 FIZ Form 4 Ch2 A 2F.indd 19
© Nilam Publication Sdn. Bhd.
9/10/13 4:26 PM
MODUL • Fizik TINGKATAN 4
2.4 Menganalisis Momentum/Analysing momentum • Menyatakan maksud momentum suatu objek Define the momentum of an object
• Menyatakan maksud momentum sebagai hasil darab jisim (m) dan halaju (v), iaitu momentum = mv Define momentum as the product of mass (m) and velocity (v), i.e. momentum = mv
• Menyatakan Prinsip Keabadian Momentum
State the Principle of Conservation of Momentum
• Menghuraikan aplikasi Prinsip Keabadian Momentum Describe applications of Conservation of Momentum
• Menyelesaikan masalah yang melibatkan momentum Solve problems involving momentum
2.5 Memahami Kesan Daya/Understanding the effect of force • Menghuraikan kesan daya seimbang yang bertindak ke atas objek Describe the effects of balanced forces acting on the object
• Menghuraikan kesan daya yang tidak seimbang yang bertindak ke atas objek Describe the effects of unbalanced forces acting on an object
• Menyelesaikan masalah dengan menggunakan F = ma U N I T
2
Solve problems using F = ma
2.6 MenganalisIS Impuls dan Daya Impuls/AnalySing impulse and impulsive force • Menerangkan apa itu Impuls
Explain what an impulsive force is
• Memberi contoh situasi yang melibatkan daya impuls Give examples of situations involving impulsive forces
• Menyatakan maksud impuls sebagai perubahan momentum, iaitu Ft = mv – mu Define impulse as a change of momentum, i.e. Ft = mv – mu
• Menyatakan maksud daya impuls sebagai kadar perubahan momentum dalam perlanggaran atau letupan, iaitu Define impulsive force as the rate of change of momentum in a collision or explosion, i.e.
F = mv – mu t • Menerangkan situasi di mana atau mengurangkan masa perlanggaran terhadap nilai daya impuls Explain the effect of increasing or decreasing the time of impact on the magnitude of the impulsive force
• Menghuraikan situasi di mana daya impuls perlu dikurangkan dan cara untuk mengurangkannya Describe situations where an impulsive force needs to be reduced and suggest ways to reduce it
• Menghuraikan situasi di mana daya impuls mendatangkan manfaat Describe situations where an impulsive force is beneficial
• Menyelesaikan masalah yang melibatkan daya impuls Solve problems involving impulsive forces
2.7 Ciri-ciri keselamatan yang diperlukan dalam kenderaan Being aware of the need for safety features in vehicles
• Menghuraikan kepentingan ciri-ciri keselamatan dalam kenderaan Describe the importance of safety features in vehicles
2.8 Memahami Graviti/Understanding gravity • Menerangkan pecutan yang disebabkan oleh graviti/Explain acceleration due to gravity • Menyatakan apa itu medan graviti/State what a gravitational field is • Menyatakan maksud kekuatan medan graviti/Define gravitational field strength • Menentukan nilai pecutan yang disebabkan oleh graviti/Determine the value of acceleration due to gravity • Menyatakan maksud Berat (W) sebagai hasil darab jisim (m) dan pecutan yang disebabkan oleh graviti (g), iaitu W = mg Define weight (W) as the product of mass (m) and acceleration due to gravity (g), i.e W = mg
• Menyelesaikan masalah yang melibatkan pecutan graviti Solve problems involving acceleration due to gravity
© Nilam Publication Sdn. Bhd.
02 FIZ Form 4 Ch2 A 2F.indd 20
20
9/10/13 4:26 PM
MODUL • Fizik TINGKATAN 4
2.9 MenganalisIS Keseimbangan Daya
Analysing forces in equilibrium
• Menghuraikan situasi keseimbangan daya
Describe situations where forces are in equilibrium
• Menyatakan apa itu daya paduan State what a resultant force is
• Tambahkan dua daya untuk menentukan daya paduan Add two forces to determine the resultant force
• Meleraikan daya kepada dua komponen daya
Resolve a force into two effective component forces
• Menyelesaikan masalah yang melibatkan keseimbangan daya Solve problems involving forces in equilibrium
2.10 Memahami Kerja, Tenaga, Kuasa dan KEcekapan
UNDERSTANDING WORK, ENGERY, POWER AND EFFICIENCY
• Menyatakan maksud Kerja (W) sebagai hasil darab daya (F) yang dikenakan dan sesaran (s) objek dalam arah daya tersebut, iaitu W = Fs
efine work (W) as the product of an applied force (F) and displacement (s) of an object in the direction of the D applied force, i.e W = Fs
• Menyatakan apabila Kerja dilakukan, tenaga dipindahkan daripada satu objek ke objek lain State that when work is done, energy is transferred from one object to another
• Menyatakan maksud tenaga kinetik, Ek, dan menyatakan bahawa Ek = 1 mv 2 2 1 Define kinetic energy, Ek, and state that Ek =
2
mv 2
• Menyatakan maksud tenaga keupayaan graviti, Ep , dan menyatakan bahawa Ep = mgh
U N I T
2
Define gravitational potential energy, Ep , and state that Ep = mgh
• Menyatakan Prinsip Keabadian Tenaga/State the Principle of Conservation of Energy • Menyatakan maksud Kuasa, P, dan menyatakan bahawa P = Tenaga Masa Energy Define power, P, and state that P =
Time
• Menerangkan apa itu kecekapan peralatan Explain what efficiency of a device is
• Menyelesaikan masalah yang melibatkan kerja, tenaga, kuasa dan kecekapan Solve problems involving work, energy, power and efficiency
2.11 Memahami Kekenyalan
Understanding elasticity
• Menyatakan maksud kekenyalan Define elasticity
• Menyatakan Hukum Hooke State Hooke’s Law
• Menyatakan maksud tenaga keupayaan kenyal, Ep, dan menyatakan bahawa Ep = 1 kx 2 2 1 Define elastic potential energy, Ep , and state that Ep =
2
kx2
• Menentukan faktor-faktor yang mempengaruhi kekenyalan Determine the factors that affect elasticity
• Menghuraikan aplikasi kekenyalan Describe applications of elasticity
• Menyelesaikan masalah yang melibatkan kekenyalan Solve problems involving elasticity
21
02 FIZ Form 4 Ch2 A 2F.indd 21
© Nilam Publication Sdn. Bhd.
9/10/13 4:26 PM
MODUL • Fizik TINGKATAN 4
MENGANALISIS GERAKAN LINEAR
2.1
ANALYSING LINEAR MOTION
Definisi jarak dan sesaran
Define distance and displacement
1 Jarak ialah jumlah laluan yang dilalui dari satu lokasi ke satu lokasi yang lain Distance is the total path length travelled from one location to another Kuantiti/Quantity: Skalar/Scalar Unit SI/SI unit: meter (m)
2 Sesaran ialah/Displacement is arah (a) jarak dalam tertentu./the distance in a specific (b) jarak antara dua lokasi yang diukur sepanjang laluan yang dalam arah tertentu.
direction
.
paling pendek yang menghubungkannya
the distance between two locations measured along the shortest path connecting them in a specific direction.
(c) jarak kedudukan akhir dari kedudukan awal dalam arah tertentu. U N I T
2
the distance of its final position from its initial position in a specified direction.
Kuantiti/Quantity: Vektor/Vector
Unit SI/SI unit: meter (m) Sekolah School
3 Rajah di sebelah kanan menunjukkan beza antara jarak dan sesaran. Apabila Ah Chong berjalan kaki di sepanjang Jalan Baik dari rumah A ke sekolah B,
Jalan Baik Good Road
The diagram on the right shows the difference between distance and displacement. When Ah Chong walked from House A to School B, along Good Road, Jarak/Distance = Panjang laluan di sepanjang Jalan Baik
Length of the road along Good Road Sesaran/Displacement = Panjang garis lurus AB / Length of the straight line AB A
Contoh/Example Rahim berjalan dari rumahnya ke simpang sejauh 1.5 km dari rumahnya. Kemudian dia berpatah balik dan berhenti di warung Pak Din yang sejauh 0.5 km dari rumahnya.
B
Rumah House
Warung Pak Din Rumah Rahim Pak Din’s stall Rahim’s house
B
0.5 km
O
1.5 km
Rahim walked from his house to the junction which is 1.5 km from his house. Then he turned back and stopped at Pak Din’s stall which is 0.5 km from his house. (a) Berapakah sesaran Rahim dari rumahnya/What is Rahim’s displacement from his house (i) apabila dia sampai di simpang?/when he reached the junction? 1.5 km ke timur/1.5 km to the east
Utara/North
(ii) apabila dia berada di warung Pak Din?/when he was at Pak Din’s stall? 0.5 km ke barat/0.5 km to the west (b) Selepas bersarapan pagi, Rahim berjalan pulang ke rumahnya. Apabila dia sampai di rumahnya, After breakfast, Rahim walked back to his house. When he reached home,
(i) berapakah jumlah jarak yang dilalui oleh Rahim?/what was the total distance travelled by Rahim? 1.5 km + 1.5 km + 0.5 km + 0.5 km = 4 km
(ii) berapakah jumlah sesaran Rahim dari rumahnya?/what was Rahim’s total displacement from his house? 0 km
© Nilam Publication Sdn. Bhd.
02 FIZ Form 4 Ch2 A 2F.indd 22
22
9/10/13 4:26 PM
MODUL • Fizik TINGKATAN 4
Definisi Laju dan Halaju
Define Speed and Velocity
1 Laju ialah kadar perubahan jarak Speed is the rate of change of distance
Laju, v =
Jarak dilalui Masa yang diambil
Speed, v =
Distance travelled Time taken
–1 Unit SI/SI unit: m s
Kuantiti/Quantity: Skalar/Scalar 2 Halaju ialah kadar perubahan sesaran Velocity is the rate of change of displacement
Halaju, v =
Sesaran Masa yang diambil
Velocity, v =
Displacement Time taken U N I T
–1
Kuantiti/Quantity: Vektor/Vector
Unit SI/SI unit: m s
2
3 Arah halaju adalah arah sesaran. Direction of velocity is the direction of the displacement.
Laju purata, Jumlah jarak dilalui, s v = Jumlah masa yang diambil, t Average speed, Total distance travelled, s v = Total time taken, t
Halaju purata: Sesaran, s v = Jumlah masa yang diambil, t Average velocity: Displacement, s v = Total time taken, t
Laju sekata
kekal Laju yang magnitudnya sama tanpa mempertimbangkan arahnya.
Uniform speed
Speed that remains the same regardless of its direction.
Halaju sekata
Halaju yang magnitud dan kekal sama.
Uniform velocity
Velocity that remains the same in magnitude and direction .
arahnya
Suatu objek mempunyai halaju tak sekata jika: An object has a non-uniform velocity if:
(a) arah the
gerakan direction
magnitud
(b) the
magnitude
23
02 FIZ Form 4 Ch2 A 2F.indd 23
in magnitude
berubah atau gerakan tidak linear. of motion changes or the motion is not linear.
halaju berubah. of its velocity changes.
© Nilam Publication Sdn. Bhd.
9/10/13 4:26 PM
MODUL • Fizik TINGKATAN 4
Contoh/Example
1
Sebuah kapal terbang menuju ke utara selama 1 jam dengan halaju 300 km j–1. Kemudian, kapal terbang bergerak ke timur selama 1 jam dengan halaju 400 km j–1.
An aeroplane flies towards the north for 1 hour with a velocity of 300 km h–1. Then, the plane moves to the east for 1 hour with a velocity of 400 km h–1.
A
B
300 km j–1
(a) Berapakah laju purata kapal terbang itu?
400 km j–1
O
What is the average speed of the plane?
Jumlah jarak Total distance Average speed = Total time Jumlah masa Jarak OA = LajuOA × MasaOA Distance OA = SpeedOA × TimeOA = 300 km j–1 × 1 jam = 300 km h–1 × 1 hour = 300 km = 300 km Jarak AB = LajuAB × MasaAB Distance AB = SpeedAB × TimeAB = 400 km j–1 × 1 jam = 400 km h–1 × 1 hour = 400 km = 400 km (300 km + 400 km) ∴ Laju purata = (300 km + 400 km) ∴ Average speed = 2 hours 2 jam 700 km 700 km = = = 350 km h–1 = 350 km j–1 2 hours 2 jam Laju purata =
U N I T
2
(b) Berapakah halaju purata kapal terbang itu?/What is the average velocity of the plane? Dari (a); From (a); JarakOA = 300 km DistanceOA = 300 km JarakAB = 400 km DistanceAB = 400 km ∴ SesaranOB = (300 km)2 + (400 km)2 ∴ DisplacementOB = (300 km)2 + (400 km)2 = 500 km = 500 km Displacement ∴ Halaju purata = Sesaran ∴ Average velocity = Time Masa 500 km 500 km = = 2 h 2j = 250 km h–1 = 250 km j–1
Contoh/Example
2
Bacaan meter had laju bagi sebuah kereta yang bergerak ke arah utara menunjukkan 80 km j–1. Sebuah kereta yang lain bergerak pada 80 km j–1 menuju ke selatan. Adakah kelajuan kedua-dua kereta itu sama? Adakah halaju kedua-dua kereta itu sama? Terangkan jawapan anda.
The speedometer reading for a car travelling north shows 80 km h–1. Another car is travelling at 80 km h–1 towards the south. Is the speed of both cars the same? Is the velocity of both cars the same? Explain your answer.
Kelajuan kedua-dua kereta itu adalah sama iaitu 80 km j–1 tetapi halaju adalah tidak sama kerana arah kedua-dua kereta itu berbeza. / The speed of both cars is the same, that is, 80 km h–1 but the velocity is not the same because the cars are in different directions.
© Nilam Publication Sdn. Bhd.
02 FIZ Form 4 Ch2 A 2F.indd 24
24
9/10/13 4:26 PM
MODUL • Fizik TINGKATAN 4
Definisi pecutan dan nyahpecutan
Definition of acceleration and deceleration
1 Pecutan, a, didefinisikan sebagai kadar perubahan halaju
.
Acceleration, a, is defined as the rate of change of velocity
.
2 Formula dan Unit SI:/Formula and SI unit: Perubahan halaju Change in velocity Acceleration, a = Time taken Masa yang diambil Halaju akhir, v – Halaju awal, u Final velocity, v – Initial velocity, u = = Time taken, t Masa yang diambil, t v–u v–u = = t t
Pecutan, a =
Unit SI/SI unit:
m s–2
‘a positif’ (pecutan): Halaju suatu objek yang lebih tinggi.
bertambah increases
‘positive a’ (acceleration): The velocity of an object velocity, v.
‘a negatif’ (nyahpecutan): Halaju suatu objek v yang lebih rendah.
dari halaju awal, u, kepada halaju akhir, v
berkurang
‘negative a’ (deceleration): The velocity of an object velocity, v.
decreases
from an initial velocity, u, to a higher final
dari halaju awal, u, kepada halaju akhir,
U N I T
2
from an initial velocity, u, to a lower final
Pecutan sifar bermaksud suatu objek berada dalam keadaan pegun atau bergerak pada halaju malar, a = 0 Zero acceleration means an object is at rest or is moving at a constant velocity, a = 0
Contoh/Example Seorang penunggang basikal bermula dari keadaan rehat dan menambahkan halajunya pada kadar seragam sehingga dia mencapai halaju 4.0 m s–1 dalam 5.0 s. Berapakah purata pecutannya?
A cyclist starts from rest and increases his velocity at a constant rate until he reaches a velocity of 4.0 m s–1 in 5.0 s. What is his average acceleration? v–u v–u pecutan, a = acceleration, a = Halaju awal/Initial velocity = 0 t t 4.0 m s–1 –1 Halaju akhir/Final velocity = (4.0 – 0) m s
Masa yang diambil/Time taken =
a=
5.0 s
5.0 s
= 0.8 m s–2
25
02 FIZ Form 4 Ch2 A 2F.indd 25
© Nilam Publication Sdn. Bhd.
9/10/13 4:26 PM
MODUL • Fizik TINGKATAN 4
Contoh/Example Perhatikan gerakan sebuah kereta dalam Rajah (a) dan Rajah (b) yang merupakan gambar stroboskop. Observe the motion of the car in Diagram (a) and Diagram (b) which are stroboscopic pictures.
(a) Diagram (a) Rajah (a)/
(b) Diagram (b) Rajah (b)/
(i) Huraikan perubahan halaju sebuah kereta dalam Rajah (a). Adakah kereta memecut atau menyahpecut?
Describe the changes in velocity of the car in Diagram (a). Is the car accelerating or decelerating?
Jarak antara dua gambar berturut-turut bertambah. Halaju kereta itu bertambah. Kereta itu memecut. The distance between two consecutive images increases. Velocity of the car is increasing. The car accelerates.
(ii) Huraikan perubahan halaju sebuah kereta dalam Rajah (b). Adakah kereta memecut atau menyahpecut?
Describe the changes in velocity of the car in Diagram (b). Is the car accelerating or decelerating?
Jarak antara dua gambar berturut-turut berkurang. Halaju kereta itu berkurang. Kereta itu mengalami U N I T
nyahpecutan. The distance between two consecutive images decreases. Velocity of the car is decreasing. The car decelerates.
2 Latihan/Exercises 1 Lengkapkan jadual berikut untuk membuat perbandingan antara dua istilah yang diberi. Complete the following tables to do comparisons between the terms given.
Jarak/Distance Jarak ialah jumlah panjang laluan yang dilalui dari satu lokasi ke satu lokasi yang lain. Total path length travelled from one location to another.
Sesaran/Displacement Sesaran ialah jarak dalam arah tertentu. Displacment is the distance in a specified direction.
Laju/Speed
Halaju/Velocity
Laju ialah kadar perubahan jarak.
Halaju ialah kadar perubahan sesaran.
Speed is the rate of change of distance.
Velocity is the rate of change of displacement.
Kuantiti asas
Kuantiti asas
Quantiti terbitan
Quantiti terbitan
Base quantity
Base quantity
Derived quantity
Derived quantity
Kuantiti skalar
Kuantiti vektor
Kuantiti skalar
Kuantiti vektor
Scalar quantity
Vector quantity
Scalar quantity
Vector quantity
2 Seorang budak berjalan sepanjang laluan PQ./A boy walks along path PQ. Cari/Find (a) jumlah jarak dilalui./total distance travelled. (5 + 7 + 5 + 10 + 10 + 10 + 10)m = 57 m
10 m
10 m
P
(b) sesaran./displacement. (7 + 10 + 10)m = 27 m
10 m
5m
Q
7m
© Nilam Publication Sdn. Bhd.
02 FIZ Form 4 Ch2 A 2F.indd 26
26
9/10/13 4:26 PM
MODUL • Fizik TINGKATAN 4
3 Isikan tempat kosong/Fill in the blanks: (a) Laju malar 10 m s–1
10 m
Jarak sejauh
A constant speed of 10 m s–1
A distance of
Sesaran
(b) Halaju malar –10 m s–1 bertentangan A constant velocity of –10 m s–1 the opposite direction.
(c) Pecutan malar 4 m s–2
Halaju
A constant acceleration of 4 m s–2
(d) Nyahpecutan malar 4 m s–2 A constant deceleration of 4 m s–2
10 m
second
is travelled every
saat
sejauh 10 m yang dilalui setiap displacement
A
saat
dilalui setiap
meningkat Velocity
of 10 m is travelled every
berkurang
Halaju
Velocity
decreases
second
in
saat
dengan 4 m s–1 setiap
increases
dalam arah
second
by 4 m s–1 every
dengan 4 m s–1 setiap
saat
second
by 4 m s–1 every
Hubung kait Sesaran, Halaju, Pecutan dan Masa Relating Displacement, Velocity, Acceleration and Time
Jangka masa detik:/Ticker timer:
U N I T
6 – 12 V a.c.
arus ulang-alik • Ia disambung ke bekalan kuasa 50 Hz. Apabila dihidupkan, bilah pengetuk akan bergetar 50 kali sesaat. alternating current It is connected to an supply of 50 Hz. When it is switched on, the iron strip will vibrate 50 times per second.
2 Jangka masa detik Ticker timer
Troli dinamik Dynamic trolley
• Masa diambil untuk membuat 50 titik pada pita detik ialah 1 saat. Jadi, selang masa antara dua titik yang berturutan ialah 1 s = 0.02 s. 50 The time taken to make 50 dots on the ticker tape is 1 second. Hence, the time interval between 2 consecutive 1 dots is s = 0.02 s. 50
selang masa antara 2 titik
• 1 detik didefinisikan sebagai
.
the time interval between 2 dots
1 tick is defined as
.
Aktiviti 1 Kaedah pengiraan
Activity 1 Method of calculation
Menyiasat gerakan dalam makmal iaitu menentukan jarak / sesaran, kelajuan / halaju, masa dan pecutan / nyahpecutan To investigate motion in laboratory to determine distance / displacement, speed / velocity, time and acceleration / deceleration 10 ticks/detik
A
8.0 cm
B
27
02 FIZ Form 4 Ch2 A 2F.indd 27
© Nilam Publication Sdn. Bhd.
9/10/13 4:26 PM
MODUL • Fizik TINGKATAN 4
LANGKAH 1: Menentukan masa diambil untuk 1 detik. STEP 1: Determination of time taken for 1 tick.
(a) Masa yang diambil untuk 50 detik = 1 saat (b) Masa yang diambil untuk 1 detik = 0.02 s (c) Masa yang diambil dari A ke B = 10 detik
(a) Time taken for 50 ticks = 1 second (b) Time taken for 1 tick = 0.02 s (c) Time taken from A to B = 10 ticks = 0.2 s
= 0.2 s
LANGKAH 2: Menentukan sesaran STEP 2: Determination of displacement
Sesaran suatu objek ditentukan dengan mengukur panjang pita detik yang ditarik melalui jangka masa detik The displacement of the object is determined by measuring the length of the ticker tape that is pulled through the ticker time Sesaran A ke B = 8.0 cm/Displacement from A to B = 8.0 cm
LANGKAH 3: Menentukan halaju U N I T
2
STEP 3: Determination of velocity
Halaju, v =
=
Sesaran Masa 8.0 cm = 0.2 s
40.0
cm s–1
Velocity, v =
Displacement Time
8.0 cm = 0.2 s
=
40.0
cm s–1
LANGKAH 4: Menentukan pecutan STEP 4: Determination of acceleration 1.5 cm
A
υ
3.5 cm
B
5.5 cm
C
7.5 cm
D
ν
E
Dari jalur pertama: Halaju pertama, u, pada AB/From the first strip: Initial velocity, u, at AB 1.5 cm uAB = = 7.5 cm s–1 0.2 s Dari jalur terakhir: Halaju terakhir, v, pada DE/From the final strip: Final velocity, v, at DE 7.5 cm vAB = = 37.5 cm s–1 0.2 s Selang masa, bagi perubahan halaju, t/The time interval, t, for the change in the velocity t = (4 – 1) × 0.2 s = 0.6 s Pecutan/Acceleration, a=
(37.5 – 7.5) cm s–1 = 50.0 cm s–2 0.6 s
© Nilam Publication Sdn. Bhd.
02 FIZ Form 4 Ch2 A 2F.indd 28
28
9/10/13 4:26 PM
MODUL • Fizik TINGKATAN 4
Latihan/Exercises 1
Arah pergerakan Direction of motion
2 A 2.0 cm B
4.0 cm
Arah pergerakan Direction of motion C D 6.0 cm
8.0 cm
E
Rajah di atas menunjukkan suatu pita detik yang mengandungi 5 detik untuk setiap selang AB-BC-CD Berdasarkan rajah di atas, hitungkan pecutan dan DE. Hitungkan pecutan objek itu. objek itu. The diagram above shows a ticker tape contains 5 ticks for 0.2 cm
1.4 cm
Based on the diagram above, calculate the every interval AB-BC-CD and DE. Calculate the acceleration acceleration of the object. of the object. Penyelesaian/Solution Penyelesaian/Solution
u = 0.2 cm = 10 cm s–1 0.02 s v = 1.4 cm = 70 cm s–1 0.02 s t = (5 – 1) × 0.02 s = 0.08 s –1 a = (70 – 10) cm s = 750 cm s–2 = 7.5 m s–2 0.08 s
u = 8.0 cm = 80.0 cm s–1 0.1 s 2.0 cm = 20.0 cm s–1 v= 0.1 s t = (4 – 1) × 0.1 s = 0.3 s a = (20.0 – 80.0) = –200 cm s–2 = –2.0 m s–2 0.3 s
3 Rajah di sebelah menunjukkan carta pita detik bagi sebuah troli yang bergerak. Frekuensi bagi jangka masa detik ialah 50 Hz. Setiap jalur pita merupakan panjang 10 detik. The diagram on the right shows a ticker tape chart for a moving trolley. The frequency of the ticker-timer used is 50 Hz. Each strip of the tape is a 10-ticks length.
U N I T
2
Panjang 10 detik/cm 10-tick length/cm 12.0
10.0
Jalur pita Strip of the tape
(a) Berapakah selang masa antara dua titik? What is the time interval between two dots?
8.0
0.02 s (b) Berapakah selang masa untuk satu jalur?
6.0
What is the time interval for one strip?
0.02 × 10 = 0.2 s
4.0
(c) Berapakah halaju awal? What is the initial velocity?
2.0 u = 2.0 cm = 10.0 cm s–1 0.2 s (d) Berapakah halaju akhir?/What is the final velocity? 0 Detik/Ticks v = 12.0 cm = 60.0 cm s–1 0.2 s (e) Berapakah selang masa yang diambil untuk berubah dari halaju awal kepada halaju akhirnya?
What is the time interval to change from its initial velocity to its final velocity?
t = (11 – 1) × 0.2 s = 2.0 s (f) Berapakah pecutan objek itu?/What is the acceleration of the object? –1 a = (60.0 – 10.0) cm s = 25.0 cm s–2 2.0 s 29
02 FIZ Form 4 Ch2 A 2F.indd 29
© Nilam Publication Sdn. Bhd.
9/10/13 4:26 PM
MODUL • Fizik TINGKATAN 4
Aktiviti 2 Untuk mengenal pasti jenis gerakan Activity 2 To identify the types of motion
PITA DETIK DAN CARTA TICKER TAPE AND CHARTS
(i)
Jenis gerakan:
Arah gerakan Direction of motion
Panjang/Length (cm)
Type of motion: Laju malar/Constant speed
Jarak dilalui antara dua titik berturutan: Distance between two consecutive dots: Sama/equal Detik/Ticks
(ii) U N I T
Arah gerakan Direction of motion
Panjang/Length (cm) 4.0
The distance between two consecutive dots increases uniformly.
3.3
2
• Halaju objek itu
2.6
bertambah
1.2
• Objek itu bergerak pada
0.5 2
3
4
5
6
Detik/Ticks
pecutan
• Jarak antara dua titik berturutan secara seragam.
Arah gerakan Direction of motion
seragam.
acceleration
The object moves at a uniform
(iii) Panjang/Length (cm)
secara seragam. increases
The velocity of the object uniformly.
1.9
1
bertambah
• Jarak antara dua titik berturutan secara seragam.
.
berkurang
The distance between two consecutive dots decreases uniformly.
• Halaju objek itu
berkurang
The velocity of the object uniformly.
secara seragam. decreases
• Objek itu bergerak pada nyahpecutan seragam.
Detik/Ticks
The object moves at a uniform
deceleration .
Aktiviti 3 Untuk menentukan sesaran, halaju purata dan pecutan
Activity 3 To determine displacement, average velocity and acceleration
Radas
Jangka masa detik, troli, bekalan kuasa 12 V, landasan, pita detik, pembaris
Apparatus
Ticker timer, trolley, 12 V power supply, runway, ticker tape, ruler
1 Naikkan satu hujung landasan kepada ketinggian yang munasabah. Prosedur Procedure
Raise one end of runway to a reasonable height.
2 Lalukan pita detik melalui jangka masa detik dan sambungkan kepada troli di atas landasan. Pass the ticker tape through the ticker timer and attach it to a trolley at the top of the runway.
© Nilam Publication Sdn. Bhd.
02 FIZ Form 4 Ch2 A 2F.indd 30
30
9/10/13 4:26 PM
MODUL • Fizik TINGKATAN 4
3 Hidupkan jangka masa detik dan lepaskan troli. Switch on the ticker timer and release the trolley.
4 Apabila troli itu sampai ke penghujung landasan, hentikan troli dan potongkan pita itu. When the trolley reaches the end of the runway, stop it and cut the tape. Jangka masa detik Ticker timer Troli Trolley
Pita detik Ticker tape
Landasan Runway
Prosedur
Blok kayu Wooden block
Procedure Sumber kuasa Power supply
5 Tanda dan potongkan pita kepada jalur-jalur 10 detik dari permulaan titik pertama jelas dilihat. Mark and cut the tape into 10-tick strips from the start of the first clear dot.
6 Lekatkan jalur-jalur 10 detik bersebelahan pada kertas untuk membuat satu carta pita. Paste the 10-tick strips side-by-side on a paper to make a tape chart.
Cari
Jalur pertama
Jalur terakhir
Find
1st strip
Last strip
Sesaran bagi 10 detik
U N I T
2
The displacement of the 10-tick
Masa yang diambil bagi jalur 10 detik The time interval for the 10-tick strip
Pengumpulan data
Halaju purata jalur 10 detik Average velocity of the 10-tick strip
Collecting data
Perubahan halaju antara dua jalur 10 detik Change in velocity between the two 10-tick strips
Masa yang diambil untuk perubahan halaju Time taken for the change in velocity
Pecutan Acceleration
Kesimpulan
Apabila halaju suatu objek bertambah, ia mengalami pecutan.
Conclusion
When the velocity of an object increases, it experiences acceleration.
Selesaikan masalah pada gerakan linear dengan pecutan seragam Solve problems on linear motion with uniform acceleration
(1) v = u + at
(2) s = u + v t 2
di mana/where: s: sesaran/displacement u: halaju awal/initial velocity
(3) s = ut + ½ at2
v: halaju akhir/final velocity a: pecutan/acceleration
31
02 FIZ Form 4 Ch2 A 2F.indd 31
(4) v2 = u2 + 2as
t: masa/time
© Nilam Publication Sdn. Bhd.
9/10/13 4:26 PM
MODUL • Fizik TINGKATAN 4
Latihan/Exercises 1 Sebuah kereta memecut dari keadaan rehat ke 25 m s–1 dalam 4 s. Cari pecutan kereta itu. A car accelerates from rest to 25 m s–1 in 4 s. Find the acceleration of the car.
Penyelesaian/Solution u = 0, v = 25 m s–1, t = 4 s, a = ? –1 v = u + at a = v – u = 25 m s – 0 = 6.25 m s–2 4s t 2 Sebuah kereta memecut dari keadaan rehat pada 3 m s–2 sepanjang suatu jalan lurus. Berapakah sesaran yang dilalui oleh kereta itu selepas 4 s?
0 m s–1 a = 3 m s–2
A car accelerates from rest at 3 m s–2 along a straight road. How far has the car travelled after 4 s?
U N I T
2
Sesaran / Displacement
Penyelesaian/Solution u = 0, a = 3 m s–2, t = 4 s, sesaran/displacement = ? s = ut + 1 at2 2 1 = 0 + (3 m s–2)(4 s)2 = 24 m 2 3 Sebuah kereta bergerak dengan halaju 20 m s–1 sepanjang jalan lurus. Pemandu itu menekan brek selama 5 s. Ia menyebabkan nyahpecutan 3 m s–2, berapakah halaju akhir kereta itu?
t=4s
20 m s–1
v a = –3 m s–2
A car is travelling at 20 m s–1 along a straight road. The driver brakes for 5 s. This causes a deceleration of 3 m s–2. What is the final velocity of the car?
Penyelesaian/Solution u = 20 m s–1, t = 5 s, a = –3 m s–2, v = ? v = u + at = 20 m s–1 + (–3 m s–2)(5 s) = 5 m s–1
t=5s
4 Sebuah kereta bergerak dengan halaju malar 40 m s–1. Pemandu ternampak suatu penghalang di hadapannya dan dia segera menekan brek. Dia dapat memberhentikan keretanya dalam masa 8 s. Jarak antara penghalang itu dari kereta apabila pemandu ternampak penghalang itu ialah 180 m. Berapakah jarak penghalang itu dari kereta selepas ia berhenti?
A car was moving at a constant velocity of 40 m s–1. The driver saw an obstacle in front and he immediately stepped on the brake pedal. He managed to stop the car in 8 s. The distance of the obstacle from the car when the driver spotted it was 180 m. How far was the obstacle from the car after it stopped?
Penyelesaian/Solution u = 40 m s–1, v = 0, t = 8 s, s = ? –1 s = u + v t = (40 m s + 0)(8 s) = 160 m 2 2 Jarak penghalang dari kereta itu selepas berhenti / The distance of the obstacle from the car after it stopped = 180 m – 160 m = 20 m
© Nilam Publication Sdn. Bhd.
02 FIZ Form 4 Ch2 A 2F.indd 32
32
9/10/13 4:26 PM
MODUL • Fizik TINGKATAN 4
MENGANALISIS GRAF GERAKAN
2.2
ANALYSING MOTION GRAPH
Daripada graf gerakan, anda akan memahami: From motion graphs, you will understand:
(i) berapa jauh yang telah dilalui oleh sesuatu objek – (jarak, sesaran)
how far an object has travelled – (distance, displacement)
(ii) berapa cepat ia bergerak – (laju, halaju)
how fast it is moving – (speed, velocity)
(iii) perubahan halaju terhadap masa – (pecutan / nyahpecutan)
change of velocity with time – (acceleration / deceleration)
Lakar dan tafsir graf sesaran-masa
Plot and interpret displacement-time graph
t=0s
Rajah di sebelah kanan menunjukkan gerakan sebuah kereta pada masa yang berlainan. The diagram on the right shows the motion of a car with respect to time.
Kedudukan: 0 m Position
1s
2s
3s
4s
5s
10 m
20 m
30 m
40 m
50 m
Graf sesaran-masa
(a) Lakarkan graf sesaran-masa kereta itu.
(b) Hitungkan kecerunan graf. Calculate the gradient of the graph.
Kecerunan/Gradient = (50 – 0) m = 10 m s–1 5s (c) Apakah unit bagi kecerunan?
50
Kedudukan/Position (m)
2
Displacement-time graph
Plot a displacement-time graph for the car.
What is the unit of this gradient? –1
ms
U N I T
40 30 20 10 0
(d) Apakah kuantiti fizik yang ditunjukkan oleh unit ini?
1
0
2
3
4
5
Masa/Time (s)
Kecerunan graf = halaju
What is the physical quantity shown by this unit? Halaju/velocity
The gradient = velocity of the graph
Membuat kesimpulan daripada bentuk graf sesaran-masa Deduction from the shape of a displacement-time graph
Objek berada dalam keadaan rehat kerana ia kedudukan yang sama berada pada pada bilabila masa.
Sesaran/Displacement, s/m
Objek pada keadaan rehat
The object is at at any time.
Object at rest
rest
because it is at the same position
Halaju = kecerunan graf 0
t/s
Velocity = gradient of the graph
=
33
02 FIZ Form 4 Ch2 A 2F.indd 33
0
m s–1
© Nilam Publication Sdn. Bhd.
9/10/13 4:26 PM
MODUL • Fizik TINGKATAN 4
malar kerana • Objek bergerak pada halaju sesaran bertambah secara seragam dalam setiap saat.
Sesaran/Displacement, s/m
Objek bergerak dengan halaju malar
Object travels at constant velocity because displacement increases constantly in every second.
20
• Garis lurus graf mempunyai kecerunan yang tetap .
Object moving at constant velocity
The straight line of the graph has a 0
5
t/s
• Halaju = kecerunan Velocity = gradient
4
m s–1
Object moves with acceleratiom because the rate of change increasing . of displacement is
Objek bergerak dengan pecutan
• Kecerunan lengkungan menunjukkan halaju
Object moving with acceleration
2
=
• Objek bergerak dengan pecutan kerana kadar bertambah . perubahan sesaran
Sesaran/Displacement, s/m
U N I T
0
constant gradient.
t/s
bertambah , dan ini bertambah.
increasing The gradient of the curve is velocity the is increasing.
showing that
• Objek mengalami pecutan . The object experiences
acceleration
.
Menentukan jarak, sesaran dan halaju dari graf sesaran-masa
Determine distance, displacement and velocity from the displacement-time graph
Rajah di bawah menunjukkan graf sesaran-masa bagi suatu objek. The diagram below shows a displacement-time graph of an object.
Berdasarkan graf, Based on the graph,
(a) Hitungkan halaju objek antara
calculate the velocity of the object between (i) A dan/and B (ii) B dan/and C
v = 0 v = 20 m 10 s = 2 m s–1
Sesaran/m Displacement / m 20
(iii) C dan/and D v = – 20 m 5s = –4 m s–1
A 0
B
C
10
30
D 35
Masa/s Time / s
(b) Gambarkan gerakan objek itu antara Describe the motion of the object between
(i) A dan B: halaju malar (ii) B dan C: dalam keadaan rehat A and B: constant velocity B and C: at rest (iii) C dan D: halaju malar tetapi objek bergerak pada arah yang bertentangan
C and D: constant velocity but in opposite direction
© Nilam Publication Sdn. Bhd.
02 FIZ Form 4 Ch2 A 2F.indd 34
34
9/10/13 4:26 PM
MODUL • Fizik TINGKATAN 4
(c) Cari/Find (i) jumlah jarak/total distance (ii) jumlah sesaran/total displacement = (20 + 0 + 20)m = 40 m = (20 + 0 – 20)m = 0 (d) Hitungkan/Calculate (i) laju purata
(ii) halaju purata gerakan zarah itu.
the average speed
the average velocity of the moving particle.
= jumlah jarak masa = 40 m 35 s = 1.14 m s–1
= sesaran masa
=
= 0
= 0
total distance time 40 m = 35 s = 1.14 m s–1
=
displacement time
Lakar dan tafsir graf halaju-masa
Plot and interpret velocity-time graph
Rajah menunjukkan kereta bergerak dari rehat dalam suatu garis lurus. The diagram shows the car move from rest in a straight line.
(a) Lakarkan graf halaju-masa bagi kereta ini.
t=0s
Halaju: Velocity
1s
2s
0 m s–1 2 m s–1 4 m s–1
5s
6 m s–1
8 m s–1
10 m s–1
2
Velocity-time graph
Calculate the gradient of the graph.
Halaju/Velocity (m s–1)
Kecerunan = (10 – 0) m s–1 = 2 m s–2 (5 – 0) s Gradient
10 8
(c) Apakah unit bagi kecerunan ini?
4s
Graf halaju-masa
Plot the velocity-time graph for the car.
(b) Hitungkan kecerunan graf.
3s
U N I T
6
What is the unit of this gradient?
4
m s–2
2
(d) Apakah kuantiti fizik yang ditunjukkan oleh unit ini?
0
0
1
What is the physical quantity shown by this unit? pecutan/acceleration
2
3
4
5
Masa/Time (s)
Graf halaju-masa Velocity-time graph
• Kecerunan graf = pecutan atau nyahpecutan The gradient of graph = acceleration or deceleration
• Luas di bawah graf = sesaran The area under the graph = displacement
35
02 FIZ Form 4 Ch2 A 2F.indd 35
© Nilam Publication Sdn. Bhd.
9/10/13 4:26 PM
MODUL • Fizik TINGKATAN 4
Membuat kesimpulan daripada bentuk graf halaju-masa Deduce from the shape of velocity-time graph
Objek berada dalam keadaan rehat Object at rest v/m s-1
• Kecerunan = pecutan =
0
Gradient = acceleration =
0
• Luas di bawah graf = sesaran =
0 0
Area under the graph = displacement =
rehat
• Objek berada dalam keadaan t/s
0
Objek bergerak dengan halaju malar Object moves with constant velocity
Object is at
rest
0 • Kecerunan/Gradient = pecutan/acceleration = –1 • Luas di bawah graf = sesaran = 10 m s × 2 s = 20 m • Sesaran dilalui =
10
2
.
–1 Area under the graph = displacement = 10 m s × 2 s = 20 m
v/m s-1
U N I T
20
m 20
Displacement travelled =
0
2
t/s
Object moving with constant acceleration v/m s-1
constant
velocity
18 m s–1 = 6 m s–2 3s • Kecerunan/Gradient = malar/constant = –2 • Pecutan/Acceleration = 6 m s • Luas di bawah graf/ Area under the graph 1 (18 m s–1)(3 s) = 27 m 2 =
18
3
t/s
27 • Sesaran dilalui/Displacement travelled = malar • Objek bergerak dengan pecutan Object moves with
Objek bergerak dengan nyahpecutan malar Object moves with constant deceleration v
constant
• Kecerunan graf adalah
m
acceleration
malar
negatif
dan
The gradient of the graph is
constant
• Objek itu bergerak dengan
nyahpecutan
The object is moving with uniform 0
m
malar
• Objek bergerak dengan halaju Object moves with
Objek bergerak dengan pecutan malar
0
.
and
negative
seragam
deceleration
t
Nota/Notes: • Dalam analisis gerakan linear, jika halaju suatu zarah adalah sifar, ini bermakna zarah itu berhenti bergerak. In linear motion analysis, if the velocity of a particle is zero, it means that the particle has stopped moving.
• Jika halaju zarah itu menjadi negatif, maka zarah itu bergerak dalam arah bertentangan dengan arah gerakan awalnya. If the velocity of the particle becomes negative, then the particle is moving opposite to its earlier direction of motion.
© Nilam Publication Sdn. Bhd.
02 FIZ Form 4 Ch2 A 2F.indd 36
36
9/10/13 4:26 PM
MODUL • Fizik TINGKATAN 4
Menentukan jarak, sesaran, halaju dan pecutan daripada graf halaju-masa
Determine distance, displacement, velocity and acceleration from a velocity-time graph
Berikut menunjukkan graf halaju-masa bagi sebuah kereta. Berdasarkan graf, The following shows the velocity-time graph of a car. Based on the graph,
(a) hitungkan pecutan kereta itu bagi
calculate the acceleration of the car for
(i) JK
Halaju/Velocity / m s-1
–1 20 m s a= = 2 m s–2 10 s (ii) KL –1 a = – (20 – 10) m s = –1 m s–2 (20 – 10) s
M
O
J 0
10
20
30 35
–10
(iv) MN
L
10
(iii) LM a = 0
K
20
40 45 50
Masa/s Time / s
N
–1 a = – 20 m s = –2 m s–2 10 s
(b) Nyatakan jenis gerakan kereta itu bagi
State the types of motion of the car for
(i) JK : pecutan malar
(ii) KL : nyahpecutan malar (tetap)
U N I T
2
constant acceleration constant deceleration
(iii) LM : halaju tetap
(iv) MN : nyahpecutan tetap
constant velocity constant deceleration
(c) Hitungkan jumlah sesaran yang dilalui oleh kereta itu semasa
Calculate the total displacement travelled by the car during
(i) gerakan bagi 10 s yang pertama
(ii) kereta bergerak dengan halaju seragam
the first 10 s of motion
the car moves with uniform velocity
Sesaran
Sesaran
= 1 (20 m s–1)(10 s) 2 Displacement = 100 m (iii) gerakan bagi 10 s yang terakhir
Displacement
= 10 m s–1 × 10 s = 100 m
the last 10 s of motion Sesaran / Displacement = 0
(d) Hitungkan / Calculate (i) jumlah jarak bagi keseluruhan perjalanan (ii) jumlah sesaran bagi keseluruhan perjalanan.
the total distance for the whole journey
the total displacement for the whole journey.
Sesaran / Displacement = (100 + 150 + 100 + 25 + 50) m = 425 m
Sesaran / Displacement = 425 m – 50 m = 375 m
(e) Hitungkan / Calculate (i) laju purata / the average speed Laju purata / Average speed = 425 m = 9.44 m s–1 45 s
(ii) halaju purata / the average velocity Halaju purata / Average velocity = 375 m = 8.33 m s–1 45 s 37
02 FIZ Form 4 Ch2 A 2F.indd 37
© Nilam Publication Sdn. Bhd.
9/10/13 4:26 PM
MODUL • Fizik TINGKATAN 4
Menyelesaikan masalah gerakan linear dengan pecutan seragam Solve problems on linear motion with uniform acceleration
1 Hitungkan/Calculate (i) halaju bagi OP, QR dan RS
Sesaran/Displacement, s/m
velocity of OP, QR and RS
P
20
(ii) sesaran/displacement Penyelesaian/Solution:
Q
10 O 0
R 2
4
6
–10
U N I T
2
t/s
8
S
Halaju / m s-1 Velocity / m s-1
A
10
B
5 O 0
(i) OP: halaju/velocity = 20 m = 10 m s–1 2s QR: halaju/velocity = – 20 m = –10 m s–1 2s RS: halaju/velocity = – 10 m = –10 m s–1 1s (ii) s = (20 + 0 – 20 – 10)m = –10 m
C 5
10
15
20
25
2 Hitungkan/Calculate (i) pecutan bagi OA dan BC acceleration of OA and BC (ii) jumlah sesaran/total displacement
Penyelesaian/Solution: –1 (i) OA: pecutan = 10 m s = 1 m s–2 10 s acceleration –1 = – 10 m s = –2 m s–2 BC: pecutan 5s acceleration t/s 1 (ii) Jumlah sesaran = (25 + 10)s (10 m s–1) 2 Total displacement = 175 m
Kesimpulan/Conclusion 1 Kecerunan graf s melawan t memberi halaju suatu objek. Gradient of the graph s against t gives the velocity of an object.
2 Kecerunan graf v melawan t memberi pecutan suatu objek. Gradient of the graph v against t gives the acceleration of an object.
3 Luas di bawah graf v melawan t memberi sesaran yang dilalui oleh objek. Area under the graph v against t gives the displacement travelled by the object.
© Nilam Publication Sdn. Bhd.
02 FIZ Form 4 Ch2 A 2F.indd 38
38
9/10/13 4:26 PM
MODUL • Fizik TINGKATAN 4
Ringkasan bagi bentuk graf gerakan Summary of shapes of motion graphs
Graf
s melawan t
v melawan t
a melawan t
Graph
s against t
v against t
a against t
v
s
Halaju sifar Zero velocity 0
t
0
s
t
v
Halaju negatif
0
Negative velocity 0
t
t
s
U N I T
a
v
Halaju seragam Uniform velocity 0
t
0
t
v
s
0
t
2
a
Pecutan seragam Uniform acceleration 0
t
0
t
v
s
0
a
Nyahpecutan seragam
0
Uniform deceleration 0
t
0
t
t
t
39
02 FIZ Form 4 Ch2 A 2F.indd 39
© Nilam Publication Sdn. Bhd.
9/10/13 4:26 PM
MODUL • Fizik TINGKATAN 4
2.3
MEMAHAMI INERSIA
UNDERSTANDING INERTIA
Jelaskan apakah inersia/Explain what inertia is Inersia suatu objek ialah kecenderungan objek itu kekal dalam keadaan rehat atau terus bergerak dalam keadaan gerakannya The inertia of an object is the tendency of the object to remain at rest or, if moving, to continue its motion
• Suatu objek berada dalam keadaan rehat akan cenderung kekal dalam keadaan rest
An object in a state of rest tends to remain at
rehat
.
.
• Suatu objek yang berada dalam keadaan bergerak cenderung untuk kekal dalam keadaan gerakan . motion
An object in a state of motion tends to stay in U N I T
2
.
Hukum Newton pertama/Newton’s first law: Setiap objek akan terus berada dalam keadaan rehat atau keadaan gerakannya dengan halaju seragam kecuali ia dikenakan daya luar. Every object continues in its state of rest or of uniform motion unless it is acted upon by an external force.
Hubung kait inersia dengan jisim/Relate mass to inertia inersia Semakin besar jisim, semakin besar /The larger the mass, the larger the • Dua baldi kosong digantung dengan tali dari siling.
inertia
Two empty buckets are hung with rope from the ceiling.
• Sebuah baldi diisi dengan pasir manakala baldi yang lain adalah kosong.
Tali Ropes
One bucket is filled with sand while the other bucket is empty.
• Kemudian, kedua-dua baldi ditolak. Then, both buckets are pushed.
• Didapati baldi kosong itu yang diisi dengan pasir.
senang
It is found that the empty bucket is with sand.
ditolak berbanding dengan baldi
easy
to push compared to the bucket
• Baldi yang diisi dengan pasir adalah lebih The bucket filled with sand is more
Pasir Sand
difficult
susah
Baldi Buckets
untuk bergerak.
to move.
• Apabila kedua-dua baldi diayun dan diberhentikan, baldi yang diisi dengan pasir lebih susah untuk diberhentikan. When both buckets are oscillating and an attempt is made to stop them, it is more difficult to stop the bucket filled with sand.
• Ini menunjukkan baldi dengan jisim yang lebih besar menghasilkan rintangan yang lebih untuk berubah dari keadaan rehat atau dari keadaan gerakan. This shows that the bucket with a bigger mass offers a greater resistance to change from its state of rest or from its state of motion.
• Oleh itu, suatu objek dengan jisim yang besar mempunyai inersia yang lebih So, an object with a larger mass has a
© Nilam Publication Sdn. Bhd.
03 FIZ Form 4 Ch2 B 2F.indd 40
larger
besar
.
inertia.
40
9/10/13 4:30 PM
MODUL • Fizik TINGKATAN 4
Aktiviti yang melibatkan inersia/Activities involving inertia Apabila sekeping duit syiling 20 sen dikuis ke arah timbunan duit syiling 20 sen pada permukaan yang licin, duit syiling di bawah dihentam keluar tanpa menggerakkan duit inersia syiling yang lain. Ini menunjukkan bahawa bagi timbunan duit syiling di atas bercenderung untuk kekal dalam keadaan rehat dan menentang gerakan. When a 20 cent coin is flicked towards a stack of 20 cent coins on a smooth surface, the bottom coin is knocked off without moving the rest of coins. This shows that the inertia of the stack of coins above tends to remain at rest and resists motion.
Apabila kadbod ditarik keluar dengan cepat, duit syiling itu Inersia terus jatuh ke dalam gelas. duit syiling itu rehat mengekalkannya dalam keadaan walaupun kadbod itu ditarik keluar. When the cardboard is pulled away quickly, the coin drops straight into the glass. inertia The of the coin maintains it in its rest position even when the cardboard is withdrawn.
U N I T
2
Letakkan segelas air di atas sekeping kertas A4. Dengan cepat tarik keluar kertas itu secara mendatar. Apakah yang akan berlaku kepada gelas air itu? Place a glass of water on a piece of A4 paper. Suddenly you pull the paper horizontally. What happens to the glass of water?
Gelas air itu kekal dalam keadaan rehat. Inersia gelas yang berisi air itu cenderung mengekalkan gelas air dalam kedudukan rehat. The glass of water remains at rest. The inertia of the glass of water still wants it to remain at rest position.
Sebuah blok kayu diletakkan di atas sebuah troli yang bergerak menuruni landasan. Apabila gerakan troli itu dihalang oleh suatu penghalang, blok kayu itu akan kekal dalam keadaan gerakan dan ia menggelongsor ke hadapan. Inersia
Troli Trolley Blok kayu Wooden block
Penghalang Obstracle
blok kayu itu berkecenderung untuk mengekalkan keadaan gerakannya. A wooden block is placed on top of a moving trolley down a runway. When the motion of the trolley is stopped by an obstacle, the wooden motion block will continue its state of and slide forward. The
inertia
41
03 FIZ Form 4 Ch2 B 2F.indd 41
of the wooden block tends to keep its state of motion. © Nilam Publication Sdn. Bhd.
9/10/13 4:30 PM
MODUL • Fizik TINGKATAN 4
Aktiviti yang melibatkan inersia/Activities involving inertia Sebuah buku ditarik keluar dari kedudukan tengahnya. Buku jatuh ke bawah secara terus di atasnya akan . Inersia menentang cuba perubahannya dari keadaan rehat, iaitu, apabila buku ditarik keluar, buku-buku di atas tidak akan bergerak bersama-sama. A book is pulled out from its central position. The books on top will straight downwards drop . Inertia tries to resist the change from rest, that is, when the book is pulled out, the books on top do not follow suit.
Contoh situasi yang melibatkan inersia/Examples of situations involving inertia Penumpang di dalam bas akan terhumban ke belakang apabila bas yang pegun memecut ke hadapan. Mengapa? Passengers in a bus will be thrown backwards when a stationary bus starts to accelerate. Why?
U N I T
2
Apabila bas itu bergerak ke depan secara tiba-tiba dari rehat, inersia badan penumpang masih kekal dalam keadaan rehat. Bas yang pegun/The bus is stationary
Ini menyebabkan badannya terhumban ke
belakang .
When the bus moves forward suddenly from rest, the inertia of the passenger's body tends to keep him at rest. This causes his body backwards . to be thrown
Bas bergerak secara tiba-tiba ke depan The bus moves forward suddenly
Penumpang dalam bas yang bergerak terhumban ke hadapan apabila bas itu berhenti secara tiba-tiba. Mengapa? Passengers in a moving bus will be thrown forward when the bus comes to a halt suddenly. Why?
gerakan Penumpang berada dalam keadaan apabila bas itu sedang bergerak. Apabila bas itu berhenti secara tiba-tiba, Bas sedang bergerak/The bus is moving
inersia badan penumpang cenderung untuk terus bergerak ke hadapan . Ini menyebabkan badan penumpang terhumban ke hadapan . The passengers are in a state of motion when the bus is moving. When the bus stops suddenly, the inertia of the passenger tends to continue in its forward motion. This causes his body to be thrown forward .
Bas berhenti secara tiba-tiba The bus stops suddenly © Nilam Publication Sdn. Bhd.
03 FIZ Form 4 Ch2 B 2F.indd 42
42
9/10/13 4:30 PM
MODUL • Fizik TINGKATAN 4
Contoh situasi yang melibatkan inersia/Examples of situations involving inertia Gerakan ke bawah yang cepat Fast downward motion
Sos cili dalam botol boleh dituang keluar dengan senang jika botol digerakkan turun dengan cepat dan berhenti secara tiba-tiba. Jelaskan. Chili sauce in the bottle can be easily poured out if the bottle is moved down fast with a sudden stop. Explain.
• Sos dalam botol bawah. Sos Sauce
bergerak
The sauce in the bottle
bersama-sama dengan botol semasa pergerakan ke
moves
with the bottle during the downward movement.
• Apabila botol itu berhenti secara tiba-tiba, inersia sos menyebabkan ia terus bergerak ke bawah dan mengakibatkan sos dituang keluar dari botol itu. When the bottle is stopped suddenly, the inertia of the sauce causes it to continue in its downward movement and thus the sauce is poured out of the bottle.
Gerakan ke bawah yang cepat Fast downward motion
mengetuk Kepala tukul dicantum dengan ketat kepada pemegangnya dengan penghujung pemegangnya, secara menegak, di atas permukaan yang keras. knocking
The head of hammer is secured tightly to its handle by held vertically, on a hard surface.
one end of the handle,
• Ini menyebabkan kepala tukul meneruskan gerakan ke bawah apabila gerakan pemegang itu diberhentikan. Dengan ini, hujung atas pemegang itu akan dimasukkan lebih dalam ke dalam kepala tukul.
U N I T
2
This causes the hammer head to continue on its downward motion when the motion of the handle is stopped. So that the top end of the handle is slotted deeper into the hammer head.
Titisan air pada payung basah akan jatuh apabila budak itu memusingkan payung itu. The water droplets on a wet umbrella will fall when the girl rotates the umbrella.
• Ini adalah disebabkan titisan air pada permukaan payung itu bergerak secara serentak apabila payung itu dipusingkan. This is because the water droplets on the surface of the umbrella move simultaneously as the umbrella is rotated.
• Apabila payung itu berhenti memusing, mengekalkan pergerakannya. When the umbrella stops rotating, the continue in its original motion.
inersia
inertia
titisan air akan terus
of the of water droplets will
Seorang budak melarikan diri dari lembu dalam gerakan zig-zag. Mengapa? A boy runs away from a cow in a zig-zag motion. Why?
Lembu itu mempunyai jisim yang lebih besar, maka inersianya juga lebih besar. Jadi, lembu itu sukar untuk menukar arah gerakannya. A cow has a larger mass, so it has a larger inertia. So the cow has difficulty to change its direction of motion.
43
03 FIZ Form 4 Ch2 B 2F.indd 43
© Nilam Publication Sdn. Bhd.
9/10/13 4:30 PM
MODUL • Fizik TINGKATAN 4
Contoh situasi yang melibatkan inersia/Examples of situations involving inertia Sebuah kapal minyak yang besar mengambil masa yang lebih panjang untuk memecut kepada laju maksimumnya dan ia mengambil beberapa kilometer untuk berhenti walaupun propelernya telah diterbalikkan. Mengapa? A massive oil tanker (a very big ship) takes a long time to accelerate to its full speed and a few kilometers to come to a stop even though the engine has reversed its propeller to slow it down. Why?
Kapal minyak yang besar mempunyai jisim yang lebih besar, jadi inersianya juga lebih besar. Oleh itu, ia adalah lebih sukar untuk memberhentikan kapal minyak. The massive oil tanker has larger mass, so it has a larger inertia. So it is more difficult to stop the oil tanker.
Cadangan untuk mengurangkan kesan negatif inersia Suggestions to reduce the negative effects of inertia
1 Keselamatan dalam kereta: Safety in a car: U N I T
2
(a) Tali pinggang keledar mengekalkan pemandu pada tempat duduknya. Apabila kereta berhenti secara hadapan . mendadak, tali pinggang itu mengelakkan pemandu daripada terhumban ke A safety belt secures a driver to his seat. When the car stops suddenly, the seat belt prevents the driver from being thrown forward .
(b) Alas kepala mencegah kecederaan leher semasa perlanggaran dari belakang. Inersia kepala cenderung rehat apabila badan digerakkan secara tiba-tiba ke depan. untuk mengekalkannya keadaan A headrest prevents injuries to the neck during rear-end collisions. The inertia of the head tends to keep it in its state of rest when the body is moved forward suddenly.
(c) Beg udara dipasang di dalam stereng. Ia membekalkan kusyen untuk mengelakkan pemandu daripada terhentam pada stereng atau papan pesawat kereta semasa perlanggaran. An air bag is fitted inside the steering wheel. It provides a cushion hitting to prevent the driver from the steering wheel or dashboard during a collision.
2 Perabot yang diangkat oleh lori biasanya perlu diikat dengan tali kepada bahagian-bahagian lori yang tertentu supaya apabila lori bergerak atau berhenti dengan tiba-tiba, perabot itu tidak akan jatuh atau tidak akan terhumban ke depan.
Tali Rope
tied Furniture carried by a lorry normally is by ropes to certain fixed parts of the lorry so that when the lorry moves or stops suddenly, the furniture will not fall or will not be thrown forward.
3 Empat tangki kecil di mana jisim muatan dibahagi antara tangkiinersia tangki tersebut akan mempunyai yang lebih kecil. impak Ini akan mengurangkan pada setiap tangki yang disebabkan oleh inersia jika lori tangki itu berhenti dengan tibatiba. inertia . Four small tanks with distributed mass will have smaller impact This will greatly reduce the inertial on each tank if the tanker stops suddenly. © Nilam Publication Sdn. Bhd.
03 FIZ Form 4 Ch2 B 2F.indd 44
Kepala lori Tractor
Treler dengan 4 tangki kecil Trailer with 4 small tanks Lori tangki Tanker
44
9/10/13 4:30 PM
MODUL • Fizik TINGKATAN 4
Eksperimen Experiment
Tujuan Aim
Inersia dan Jisim / Inertia and Mass
Untuk mengkaji hubungan antara jisim dan inersia (tempoh ayunan) To study the relationship between mass and inertia (period of oscillation)
Radas
Bilah Hacksaw, pengapit-G, jam randik dan plastisin.
Apparatus
Hacksaw blade, G-clamp, stopwatch and plasticine.
Pemboleh ubah dimanipulasi/Manipulated variable Jisim plastisin/mass of plasticine Pemboleh ubah Variables
Pemboleh ubah bergerak balas/Responding variable Tempoh ayunan/period of oscillation Pemboleh ubah dimalarkan/Fixed variable: Panjang bilah Hacksaw/length of the Hacksaw blade
Pengapit-G/G-clamp Plastisin/Plasticine Bilah Hacksaw Hacksaw blade
1 Letakkan sejumlah plastisin (berbentuk sfera) dengan jisim 30 g pada hujung bilah Hacksaw. Place a lump of plasticine (sphere-shaped) with a mass of 30 g at the free end of the Hacksaw blade.
2 Sesarkan sedikit bilah Hacksaw dan lepaskannya supaya ia berayun secara mengufuk.
U N I T
2
Displace the Hacksaw blade slightly and release it so that it oscillates horizontally.
Prosedur Procedure
3 Tentukan dan rekodkan masa yang diambil untuk 10 ayunan lengkap, t saat. Determine and record the time taken for 10 complete oscillations, t seconds.
4 Hitungkan tempoh ayunan, T = t saat. 10 Calculate period of oscillation, T =
t seconds. 10
5 Ulangi langkah 1 – 4 eksperimen dengan jisim 40 g, 50 g, 60 g dan 70 g. Repeat steps 1 – 4 of the experiment with mass of 40 g, 50 g, 60 g and 70 g.
6 Lakarkan graf tempoh ayunan melawan jisim. Plot the graph of period of oscillation against mass.
Masa untuk 10 ayunan, t/s
Jisim / g Mass / g
Keputusan Results
Time for 10 oscillation, t/s
t2
t1
tmin
T= t s 10
30 40 50 60 70
45
03 FIZ Form 4 Ch2 B 2F.indd 45
© Nilam Publication Sdn. Bhd.
9/10/13 4:30 PM
MODUL • Fizik TINGKATAN 4
Lakarkan graf T melawan jisim, m. Plot the graph T against mass, m. T/s
Analisis Analysis
0
m/g
1 Nyatakan kuantiti yang digunakan untuk mewakili inersia dalam aktiviti ini. State the quantity used to represent inertia in this activity. Tempoh ayunan/Period of oscillation
2 Apakah hubungan antara tempoh ayunan suatu objek dengan inersianya? What is the relationship between the period of oscillation of an object and its inertia?
U N I T
2
Semakin panjang tempoh ayunan, semakin besar inersia. The longer the period of oscillation, the larger the inertia.
Perbincangan Discussion
3 Daripada graf, nyatakan hubungan antara From the graph, state the relationship between
(a) tempoh ayunan dengan jisim objek. period of oscillation and mass of object.
Semakin besar jisim, semakin panjang tempoh ayunan. The larger the mass, the longer the period of oscillation.
(b) inersia suatu objek dan jisimnya. inertia of an object and its mass.
Semakin besar jisim objek, semakin besar inersianya. The larger the mass of the object, the larger its inertia.
© Nilam Publication Sdn. Bhd.
03 FIZ Form 4 Ch2 B 2F.indd 46
46
9/10/13 4:30 PM
MODUL • Fizik TINGKATAN 4
MENGANALISIS MOMENTUM
2.4
ANALYSING MOMENTUM
Definisi momentum sebagai hasil darab jisim dan halaju Define momentum as the product of mass and velocity
Momentum = jisim × halaju kg m s-1
Unit SI:
kg m s-1
SI unit:
atau N s (Newton saat) or N s (Newton second)
Momentum adalah suatu kuantiti Momentum is a
vector
Contoh/Example
Momentum = mass × velocity
vektor
. Arah momentum mengikut arah
halaju
quantity. The direction of the momentum follows the direction of the
. velocity
.
1
Dalam permainan bola sepak, seorang pemain berjisim 70 kg bergerak dengan halaju 4 m s-1 dan seorang pemain yang lain yang berjisim 75 kg bergerak dengan 3 m s-1 menghala antara satu sama seperti yang ditunjukkan. Hitungkan momentum kedua-dua pemain itu masing-masing.
U N I T
2
In a football game, a player of mass 70 kg is moving with velocity of 4 m s–1 and the other player of mass 75 kg is Pemain I moving with 3 m s-1 towards each other as shown. Calculate Player I the momentum of the two players respectively. Penyelesaian/Solution Momentum pemain I/Momentum player I = m1v1 = (70 kg)(4 m s–1) = 280 kg m s-1 Momentum pemain II/Momentum player II = m2v2 = (75 kg)(–3 m s–1) = –225 kg m s-1
Contoh/Example
Pemain II Player II
2
Nenek (m = 80 kg) menggelongsor sekeliling gelanggang gelongsor dengan halaju 6 m s–1. Tiba-tiba dia berlanggar dengan Bobby (m = 40 kg) yang berada dalam keadaan rehat. Hitungkan momentum nenek dan Bobby masing-masing. Granny (m = 80 kg) whizzes around the ring with a velocity of 6 m s–1. Suddenly she collides with Bobby (m = 40 kg) who is at rest. Calculate the momentum of granny and Bobby respectively.
Nenek Granny
Bobby
Penyelesaian/Solution Momentum nenek/granny = m1v1 = (80 kg)(6 m s–1) = 480 kg m s–1 Momentum Bobby = m2v2 = (40 kg) × (0 m s–1) = 0 kg m s–1 (dalam keadaan rehat / at rest)
Nyatakan prinsip keabadian momentum/State the principle of conservation of momentum Tanpa kehadiran daya luar, jumlah momentum dalam suatu sistem kekal tidak berubah. In the absence of an external force, the total momentum of a system remains unchanged.
Jumlah momentum sebelum perlanggaran/letupan = Jumlah momentum selepas perlanggaran/letupan
Total momentum before collision/explosion = Total momentum after collision/explosion
47
03 FIZ Form 4 Ch2 B 2F.indd 47
© Nilam Publication Sdn. Bhd.
9/10/13 4:30 PM
MODUL • Fizik TINGKATAN 4
Aktiviti/Activity 1 Rajah di sebelah menunjukkan dua orang adik-beradik yang sedang menggelongsor. Abang bergerak dan berlanggar dengan adiknya yang berada dalam keadaan rehat. Apakah gerakan mereka selepas perlanggaran? The diagram on the right shows two brothers skating. The elder brother moves and collides with his younger brother who is at rest. What is their movement after the collision?
Selepas perlanggaran,/After collison, berkurang Laju abang ./The speed of the elder brother Laju adik
decreases
.
bertambah
Momentum abang Momentum adik
increases ./The speed of the younger brother . berkurang decreases ./Momentum of the elder brother
bertambah
./Momentum of the younger brother
. increases
.
Adakah jumlah momentum sebelum perlanggaran sama dengan jumlah momentum selepas perlanggaran? U N I T
2
Is the total momentum before collision equal to the total momentum after collision? Ya/Yes.
Aktiviti/Activity 2
Aktiviti/Activity 3
Menjentik sekeping duit syiling 20 sen, A secara terus kepada sekeping duit syiling 20 sen, B yang lain.
Menjentik sekeping duit syiling 20 sen, A secara terus kepada duit syiling 20 sen B dan C.
Flick a 20-cent coin, A, directly to another 20-cent coin, B.
Flick a 20-cent coin A, directly to 20-cent coins B and C.
A
A
B
What happens to the motion of both coins after collision?
Duit syiling A berhenti, duit syiling B bergerak. Coin A stops, coin B moves.
(b) Apakah yang berlaku kepada momentum duit syiling A selepas perlanggaran?
C
(a) Gambarkan gerakan semua duit syiling selepas perlanggaran.
(a) Apakah yang berlaku kepada gerakan kedua-dua duit syiling selepas perlanggaran?
B
Describe the motion of all the coins after collision. Duit syiling A/Coin A: Berhenti/Stop
Duit syiling B/Coin B: Rehat/At rest
Duit syiling C/Coin C: Bergerak ke kanan/Moves to the right
(b) Apakah yang berlaku kepada momentum duit syiling A selepas perlanggaran?
What happens to the momentum of coin A after collision?
What happens to the momentum of coin A after collision?
Momentum duit syiling A dipindahkan kepada
Momentum duit syiling A dipindahkan ke duit
duit syiling B selepas perlanggaran.
syiling B dan duit syiling C.
Momentum of coin A is transferred to coin B after
Momentum of coin A is transferred to coin B and to
collision.
coin C.
© Nilam Publication Sdn. Bhd.
03 FIZ Form 4 Ch2 B 2F.indd 48
48
9/10/13 4:30 PM
MODUL • Fizik TINGKATAN 4
Aktiviti/Activity 4 Rajah di sebelah menunjukkan sebiji bola keluli, A ditarik dan dilepaskan. The diagram on the right shows a steel ball, A is pulled and released.
(a) Bola itu akan berlanggar dengan empat biji bola yang lain. Ini akan sama menyebabkan bola terakhir, E bergerak ke ketinggian yang dengan ketinggian bola A.
The ball will collide with the other four balls. This will cause the last ball, E to move to the same height as ball A.
Adakah momentum diabadikan?
E
D
C
B
A
Is the momentum conserved? Ya/Yes
(b) Apakah yang akan berlaku jika kedua-dua bola A dan B ditarik dan kemudian dilepaskan?
What will happen if two balls A and B are pulled and then released?
Bola D dan E akan bergerak ke ketinggian yang sama dengan bola A dan B masing-masing. Bola C akan berada dalam keadaan rehat.
U N I T
Balls D and E will rise to the same heights of balls A and B respectively. Ball C is at rest.
2
Aktiviti/Activity 5 Seorang budak perempuan berdiri dalam keadaan rehat di atas papan luncur. Dia membalingkan bola ke hadapan. Bola itu bergerak ke kiri. Budak perempuan bergerak ke kanan. A girl is standing at rest on the skateboard. She throws the massive ball forward. The ball moves to the left. The girl moves to the right.
• Momentum bola sebelum balingan = 0 Momentum of the ball before the throw = 0 • Momentum budak perempuan sebelum balingan/Momentum of the girl before the throw = 0 • Jumlah momentum selepas balingan sama dengan jumlah momentum sebelum balingan = 0 Total momentum after the throw is equal to total momentum before the throw = 0
• Jumlah momentum selepas balingan = momentum bola + momentum budak perempuan = 0 Total momentum after the throw = momentum of the ball + momentum of the girl = 0
Jadi, selepas balingan, magnitud momentum budak perempuan adalah bertentangan . momentum bola tetapi dalam arah Therefore, after the throw, the magnitude of the momentum of the girl is of the ball but in the opposite direction.
49
03 FIZ Form 4 Ch2 B 2F.indd 49
equal
sama
dengan magnitud
to the magnitude of the momentum
© Nilam Publication Sdn. Bhd.
9/10/13 4:30 PM
MODUL • Fizik TINGKATAN 4
Perlanggaran kenyal/Elastic collision
Perlanggaran tak kenyal/Inelastic collision
u2
v1
v2
u1
u2
m1
m2
m1
m2
m1
m2
Sebelum perlanggaran Before collision
Both objects move separately velocities after the collision.
Jumlah momentum Total momentum
• U N I T
2
Jumlah Total Kinetic
•
diabadikan. is conserved.
•
• Tenaga
v2
is conserved.
energy is conserved.
kinetik
tidak diabadikan.
energy is not conserved.
u1
m1 m2 Selepas perlanggaran After collision
diabadikan.
tenaga diabadikan.
Kinetic v1
m1 m2 Sebelum perlanggaran Before collision
Jumlah Total
diabadikan.
u2
Jumlah momentum Total momentum
energy is conserved.
u1
Selepas perlanggaran After collision
The two objects combine and move together with a common velocity after the collision.
at their respective
energy is conserved.
kinetik
m2
• Kedua-dua objek bergabung dan bergerak bersama dengan satu halaju sepunya selepas perlanggaran.
tenaga diabadikan.
• Tenaga
m1
Sebelum perlanggaran Before collision
Selepas perlanggaran After collision
• Kedua-dua objek bergerak secara berasingan dengan halaju masing-masing selepas perlanggaran.
•
v m1 + m2
u1
u2
v
m1 m2 Sebelum perlanggaran Before collision
m1 m2 Selepas perlanggaran After collision
Tuliskan persamaan yang menghubungkaitkan jumlah momentum sebelum perlanggaran dengan jumlah momentum selepas perlanggaran:
Tuliskan persamaan yang menghubungkaitkan jumlah momentum sebelum perlanggaran dengan jumlah momentum selepas perlanggaran: Write equation which relates the total momentum before collision with the total momentum after collision:
Write equation which relates the total momentum before collision with the total momentum after collision:
m1u1 + m2u2 = m1v1 + m2v2
m1u1 + m2u2 = (m1 + m2)v Letupan/Explosion
Troli pegun Stationary trolleys
bercantum Sebelum letupan, kedua-dua objek bersama dan berada dalam keadaan rehat. Selepas letupan, kedua-dua objek bertentangan bergerak pada arah yang .
pin
m2
Before explosion, both the objects
m1
Sebelum letupan/Before explosion v2 m2
v1 m1
explosion, both objects move at
stick opposite
together and are at rest. After directions.
Jumlah momentum sebelum sifar . letupan adalah
Jumlah momentum selepas
The
Total momentum after explosion = m1v1 + m2v2
total
explosion is
momentum before zero .
letupan =
m1v1 + m2v2
Selepas letupan/After explosion
© Nilam Publication Sdn. Bhd.
03 FIZ Form 4 Ch2 B 2F.indd 50
50
9/10/13 4:30 PM
MODUL • Fizik TINGKATAN 4
(Catatan: v2 bernilai negatif)
Daripada prinsip keabadian momentum:
(Remarks: V2 has a negative value)
Apakah yang dimaksudkan dengan nilai negatif bagi v2? Why does v2 have a negative value?
Arah bertentangan
From the principle of conservation of momentum:
Jumlah momentum sebelum perlanggaran Total momentum before collision
Jumlah momentum selepas = perlanggaran Total momentum after collision
Terbitkan persamaan untuk letupan:
Opposite direction
Derive an equation for explosion:
0 = m1v1 + m2v2 m1v1 = –m2v2 Huraikan aplikasi prinsip kebadiaan momentum Describe applications of the principle of conservation of momentum m1 + m2
Pegun/Stationary u=0
(a) Sebelum letupan Before explosion v1
m1
• Apabila sepucuk senapang ditembak, peluru yang berjisim m2 bergerak dengan halaju tinggi, v2. Ini menghasilkan suatu momentum ke arah hadapan . When a rifle is fired, the bullet of mass m2 moves with a high velocity, v2. forward This creates a momentum in the direction.
Peluru Bullet v2 m2
• Daripada prinsip keabadian momentum, suatu momentum yang sama tetapi bertentangan arah dihasilkan supaya senapang itu tersentak ke belakang .
U N I T
2
From the principle of conservation of momentum, an equal but opposite momentum is produced to recoil the rifle backward .
(b) Selepas letupan After explosion
Catatan/Remarks: Jisim senapang/Mass of riffle = m1 Jisim peluru/Mass of bullet = m2 Selepas letupan/After explosion: v1 = Halaju senapang/Velocity of riffle v2 = Halaju peluru/Velocity of bullet Pelancaran roket/The launching of rocket • Campuran bahan api hidrogen dan oksigen terbakar dengan letupan dalam kebuk pembakaran. Gas panas dalam jet itu dipancutkan dengan kelajuan yang sangat tinggi melalui ekzos. Roket Rocket
Gas panas Hot gas
A mixture of hydrogen and oxygen fuels burn explosively in the combustion chamber. Jets of hot gases are expelled at very high speed through the exhaust.
• Kelajuan tinggi gas panas ini menghasilkan momentum yang besar ke bawah . This high-speed hot gas produces a large
momentum
.
• Dengan prinsip keabadian momentum, suatu momentum yang sama tetapi bertentangan arah dihasilkan dan menggerakkan roket itu ke atas . By the principle of conservation of momentum, an equal but opposite upwards . momentum is produced and propels the rocket
51
03 FIZ Form 4 Ch2 B 2F.indd 51
© Nilam Publication Sdn. Bhd.
9/10/13 4:30 PM
MODUL • Fizik TINGKATAN 4
Huraikan aplikasi prinsip kebadiaan momentum linear Describe applications of the principle of conservation of linear momentum
Aplikasi enjin jet: Gas panas Hot gas
Application in the jet engine:
• Suatu gas panas yang berkelajuan tinggi dipancut keluar dari belakang dengan momentum tinggi . A high-speed hot gas is ejected from the back with
high momentum .
• Ini menghasilkan mementum yang sama tetapi bertentangan arah untuk menolak jet bergerak ke hadapan.
Jet
This produces an equal and opposite momentum to propel the jet plane forward.
Dalam kawasan paya, suatu bot berkipas digunakan.
U N I T
In a swamp area, a fan boat is used. Gerakan udara ke belakang Movement of air backwards
2
• Kipas itu menghasilkan gerakan udara berkelajuan tinggi ke belakang . Ini menghasilkan suatu momentum yang besar ke belakang. The fan produces a high speed movement of air produces a large momentum backwards.
backwards
. This
• Dengan keabadian momentum, suatu momentum yang sama tetapi bertentangan arah dihasilkan dan ditindakkan ke atas bot itu. Jadi, bot itu akan bergerak ke hadapan .
Bot berkipas Fan boat
By conservation of momentum, an
equal
but opposite momentum is produced and acts on the boat. So the boat will move forward .
Seekor sotong bergerak dengan mengeluarkan cecair pada halaju yang tinggi. Air masuk melalui pembukaan yang besar dan keluar melalui tiub yang kecil. Air dipaksa keluar pada kelajuan tinggi ke belakang. Magnitud momentum air dan sotong adalah sama Sotong Squid
tetapi pada arah yang bertentangan. Ini menyebabkan sotong itu bergerak ke hadapan . velocity . Water enters A squid propels by expelling a liquid at high through a large opening and exits through a small tube. The water is forced speed out at a high backward. The magnitude of the momentum of water and squid are equal squid to jet forward .
© Nilam Publication Sdn. Bhd.
03 FIZ Form 4 Ch2 B 2F.indd 52
but opposite in direction. This causes the
52
9/10/13 4:30 PM
MODUL • Fizik TINGKATAN 4
Menyelesaikan masalah melibatkan momentum linear Solve problems involving linear momentum
Latihan/Exercises
1
Kereta A yang berjisim 1 000 kg bergerak pada 20 m s–1 berlanggar dengan kereta B yang berjisim 1 200 kg dan bergerak pada 10 m s–1 dalam arah yang sama. Akibatnya, kereta B, bergerak ke hadapan pada 15 m s–1. Berapakah halaju, v, bagi kereta A sebaik sahaja selepas perlanggaran?
Car A of mass 1 000 kg moving at 20 m s–1 collides with car B of mass 1 200 kg moving at 10 m s-1 in the same direction. If car B is shunted forwards at 15 m s–1 by the impact, what is the velocity, v, of car A immediately after the crash? uA = 20 m s-1 A
uB = 10 m s-1 B
m1 = 1 000 kg
m2 = 1 200 kg
Penyelesaian/Solution Jumlah momentum sebelum perlanggaran = Jumlah momentum selepas perlanggaran
U N I T
Total momentum before collision = Total momentum after collision
(1 000 kg)(20 m s–1) + (1 200 kg)(10 m s–1) = 20 000 kg m s–1 + 12 000 kg m s–1 = (1 000 kg)(v) = ∴ v =
Latihan/Exercises
(1 000 kg)v + (1 200 kg)(15 m s–1) (1 000 kg)(v) + 18 000 kg m s–1 14 000 kg m s–1 14 m s–1
2
2
Sebiji bola yang berjisim 5 kg dibalingkan pada halaju 20 km j–1 kepada Lily yang berjisim 60 kg pada keadaan rehat di atas ais. Lily menangkap bola itu dan kemudian menggelongsor dengan bola di atas ais. Tentukan halaju Lily dengan bola selepas perlanggaran.
A 5 kg ball is thrown at a velocity of 20 km h–1 towards Lily whose mass is 60 kg at rest on ice. Lily catches the ball and subsequently slides with the ball across the ice. Determine the velocity of Lily and the ball together after the collision. m1 = 5 kg u1 = 20 km j-¹
m2 = 60 kg u2 = 0 km j-¹
m1 = 5 kg
m2 = 60 kg
v=?
Penyelesaian/Solution Jumlah momentum sebelum perlanggaran = Jumlah momentum selepas perlanggaran
Total momentum before collision = Total momentum after collision
(5 kg)(20 km j–1) + (60 kg)(0 km j–1) = (100 + 0) kg km j–1 = ∴ v = (v =
(5 + 60) kg × v (65 kg)v 1.54 km j–1 1.54 km h–1)
53
03 FIZ Form 4 Ch2 B 2F.indd 53
© Nilam Publication Sdn. Bhd.
9/10/13 4:30 PM
MODUL • Fizik TINGKATAN 4
3
Latihan/Exercises
Sebuah trak yang berjisim 1 200 kg bergerak pada 30 m s–1 berlanggar dengan sebuah kereta yang berjisim 1 000 kg yang bergerak dalam arah bertentangan pada 20 m s–1. Selepas perlanggaran, kedua-dua kenderaan itu bergerak bersama. Berapakah halaju kedua-dua kenderaan itu sebaik sahaja selepas perlanggaran?
A truck of mass 1 200 kg moving at 30 m s–1 collides with a car of mass 1 000 kg which is traveling in the opposite direction at 20 m s–1. After the collision, the two vehicles move together. What is the velocity of both vehicles immediately after collision? 30 m s-¹
(a)
v
20 m s-¹
Sebelum perlanggaran Before collision
(b) Selepas perlanggaran After collision
Penyelesaian/Solution Jumlah momentum sebelum perlanggaran = Jumlah momentum selepas perlanggaran U N I T
2
Total momentum before collision = Total momentum after collision
(1 200 kg)(30 m s–1) + (1 000 kg)(–20 m s–1) = (36 000 – 20 000) kg m s–1 = (2 200 kg)v = ∴ v =
Latihan/Exercises
(1 200 + 1 000)kg × v (2 200 kg)v 16 000 kg m s–1 7.27 m s–1
4
Seorang menembak sepucuk pistol yang berjisim 1.5 kg. Jika peluru itu berjisim 10 g dan mempunyai halaju 300 m s–1 selepas tembakan, berapakah halaju sentakan pistol itu?
A man fires a pistol which has a mass of 1.5 kg. If the mass of the bullet is 10 g and it has a velocity of 300 m s–1 after shooting, what is the recoil velocity of the pistol? v
1.5 kg
Pegun/Stationary (a) Sebelum tembakan Before shooting
300 m s-¹
10 g
(b) Selepas tembakan After shooting
Penyelesaian/Solution Jumlah momentum sebelum tembakan = Jumlah momentum selepas tembakan
Total momentum before explosion = Total momentum after explosion
© Nilam Publication Sdn. Bhd.
03 FIZ Form 4 Ch2 B 2F.indd 54
0 kg m s–1 = (1.5 kg)(v) + (0.010 kg)(300 m s–1) (1.5 kg) (v) = –3.0 kg m s–1 ∴ v = –2.0 m s–1
54
9/10/13 4:30 PM
MODUL • Fizik TINGKATAN 4
5
Latihan/Exercises
Seekor ikan yang besar yang berjisim 3 m bergerak dengan 2 m s–1 bertemu seekor ikan kecil yang berjisim m dalam keadaan rehat. Ikan besar itu menelan ikan kecil dan meneruskan gerakan dengan kelajuan yang berkurang. Jika jisim ikan besar adalah tiga kali ganda jisim ikan kecil, berapakah halaju ikan besar selepas menelan ikan kecil itu?
Bayangkan anda berlegar di sebelah kapal angkasa pada orbit bumi dan rakan anda yang sama jisim bergerak dengan 4 km j–1 (dengan merujuk kepada kapal angkasa) melanggar anda. Jika dia memegang anda, berapakah kelajuan anda bergerak (dengan merujuk kepada kapal angkasa itu)? Imagine that you are hovering next to a space shuttle in earth orbit and your buddy of equal mass who is moving at 4 km/hr (with respect to the ship) bumps into you. If she holds onto you, how fast do you move (with respect to the ship)? Sebelum perlanggaran Before collison m
6
Latihan/Exercises
A large fish of mass 3 m is in motion at 2 m s–1 when it encounters a smaller fish of mass m which is at rest. The large fish swallows the smaller fish and continues in motion at a reduced speed. If the large fish has three times the mass of the smaller fish, then what is the speed of the large fish after swallowing the smaller fish?
Selepas perlanggaran After collison m
m
m
Sebelum perlanggaran Before collison m 3m Dalam gerakan In motion u1 = 4 km j–1
Dalam keadaan rehat At rest u2 = 0 km j–1
Dalam gerakan bersama pada laju yang sama In motion together at the same speed
2
Penyelesaian/Solution Jumlah momentum Jumlah momentum sebelum perlanggaran = selepas perlanggaran
Total momentum after collision
Total momentum before collision
( m kg)(4 km j ) + (m kg)(0 km j ) = (m + m) kg × v (4m) kg km j–1 + 0 = (2m) kg × v (4m) kg km j–1 ∴v= = 2 km j–1 (2m) kg –1
U N I T
Dalam gerakan/In motion Rehat u1 = 2 m s–1 At rest
Penyelesaian/Solution Jumlah momentum Jumlah momentum sebelum perlanggaran = selepas perlanggaran Total momentum before collision
Selepas perlanggaran After collison
–1
Total momentum after collision
[ (3m) kg × (2 m s–1)] + 0 = (6m) kg m s–1 = ∴ 4v = v =
(3m + m) kg × v (4m) kg × v 6 m s–1 1.5 m s–1
Huraikan apa yang dilakukan oleh penjaga gol sebelum dia menendang bola itu. Describe what the goalkeeper does before kicking the ball.
• Penjaga gol itu akan mengambil beberapa langkah ke belakang dan kemudian berlari ke hadapan untuk menendang bola itu. The goalkeeper takes a few steps backwards and then the ball.
runs
forward to kick
• Bola itu akan bergerak lebih jauh/lebih cepat apabila ditendang semasa berlari berbanding dengan tendangan dari kedudukan pegun. The ball goes further / faster from a standing position.
when kicked while running compared to kicking
• Ini adalah disebabkan seorang pemain bola sepak yang berlari mempunyai momentum yang dipindahkan momentumnya kepada bola. This is because a running football player has a ball.
large
momentum and his momentum is
55
03 FIZ Form 4 Ch2 B 2F.indd 55
besar
transferred
dan to the
© Nilam Publication Sdn. Bhd.
9/10/13 4:30 PM
MODUL • Fizik TINGKATAN 4
Eksperimen Experiment
Tujuan Aim
Radas Apparatus
Keabadian Momentum / Conservation of Momentum
Untuk menunjukkan jumlah momentum bagi suatu sistem tertutup adalah malar dalam perlanggaran tak kenyal. To show that the total momentum of a closed system is constant in an inelastic collision.
Jangka masa detik, pita detik, plastisin, pita selofan, troli, landasan, bekalan kuasa a.u. 12 V. Ticker timer, ticker tape, plasticine, cellophane tape, trolleys, runway, 12 V ac power supply.
1 Dirikan satu landasan dengan mengubah suai kecerunannya supaya landasan terpampas geseran di mana troli boleh bergerak turun landasan dengan halaju malar. Set up a runway and adjust the slope to compensate for friction where the trolley moves down the runway with constant velocity.
2 Letakkan plastisin pada troli P dan Q supaya mereka akan melekat antara satu sama lain semasa perlanggaran. Fix plasticine on trolleys, P and Q so that they can stick together upon collision. Pita detik Ticker tape
U N I T
Jangka masa detik Ticker timer
Troli P Trolley P
Plastisin Plasticine
Troli Q Trolley Q
Bekalan kuasa Power supply
2
Landasan terpampas geseran Frictioncompensated runway
Blok kayu/Wooden block
3 Pita detik diletakkan melalui jangka masa detik dan dilekatkan pada troli P. A ticker tape is passed through the ticker timer and is attached to trolley P.
4 Mulakan jangka masa detik dan tolakkan troli P supaya ia bergerak menuruni landasan dan berlanggar dengan troli Q, yang berada dalam keadaan rehat. Prosedur Procedure
Start the ticker timer and give trolley P a push so that it will move down the runway and collide with trolley Q, which is at rest.
5 Daripada pita detik yang diperoleh, tentu dan ukurkan halaju berikut. From the ticker tape obtained, determine and measure the following velocities.
(a) Halaju troli P sebelum perlanggaran, uP / Velocity of trolley P before collision, uP (b) Halaju troli Q sebelum perlanggaran, uQ / Velocity of trolley Q before collision, uQ (c) Halaju troli (P + Q) selepas perlanggaran, v Velocity of trolley (P + Q) after collision, v
6 Langkah-langkah 2 – 5 diulangi dengan jisim P dan jisim Q yang berbeza seperti ditunjukkan dalam jadual di bawah. Steps 2.5 are repeated for different masses of P and Q as shown in the table below. Sebelum perlanggaran
Selepas perlanggaran
Before collision
Jisim troli P
Mass of trolley P
kg
© Nilam Publication Sdn. Bhd.
03 FIZ Form 4 Ch2 B 2F.indd 56
Jisim troli Q Mass of trolley Q
kg
1
1
2
1
1
2
2
2
3
2
After collision
Halaju P
Jumlah momentum
m s–1
Total momentum
Velocity of P
kg m s–1
Halaju sepunya Common velocity
m s–1
Jumlah momentum Total momentum
kg m s–1
56
9/10/13 4:30 PM
MODUL • Fizik TINGKATAN 4
Contoh: Perlanggaran tak kenyal antara dua troli Example: Inelastic collision between two trolleys Perlanggaran berlaku di sini Collision occurs here
Arah gerakan/Direction of motion 18.4 cm
Sebelum perlanggaran Before collision
Result
Selepas perlanggaran After collision
Kuantiti fizik
Sebelum perlanggaran
Selepas perlanggaran
Physical quantity
Before collision
After collision
Panjang 10 detik Keputusan
9.2 cm
10-tick length
Masa diambil untuk 10 detik Time taken for 10 ticks
Halaju Velocity
Jisim troli (jisim 1 troli = 1 kg) Mass of trolley (mass of 1 trolley = 1 kg)
Momentum Momentum
18.4 cm
9.2 cm
0.2 s
0.2 s
18.4 cm = 0.92 m s–1 0.2 s
9.2 cm = 0.46 m s–1 0.2 s
1 kg
2 kg
(1 kg)(0.92 m s–1) = 0.92 kg m s–1
(2 kg)(0.46 m s–1) = 0.92 kg m s–1
U N I T
2
1 Bandingkan jumlah momentum sebelum perlanggaran dan selepas perlanggaran. Compare the total momentum before collision and after collision.
Jumlah momentum sebelum dan selepas perlanggaran adalah sama. The total momentum before collision and after collision are equal.
2 Nyatakan satu kesimpulan. State a conclusion.
Perbincangan
Tanpa kehadiran daya luar, jumlah momentum sebelum perlanggaran adalah sama
Discussion
dengan jumlah momentum selepas perlanggaran. In the absence of any external force, the total momentum before collision is equal to total momentum after collision.
3 Apakah tujuan utama mengubah suai landasan supaya landasan terpampas geseran? What is the main purpose of adjusting the runway so that it is friction-compensated?
Troli bergerak dengan halaju malar. The trolley moves with constant velocity.
57
03 FIZ Form 4 Ch2 B 2F.indd 57
© Nilam Publication Sdn. Bhd.
9/10/13 4:30 PM
MODUL • Fizik TINGKATAN 4
2.5
MEMAHAMI KESAN DAYA
UNDERSTANDING THE EFFECTS OF A FORCE
Huraikan kesan daya seimbang yang bertindak ke atas objek Describe the effect of balanced forces acting on an object
Daya seimbang/Balanced force Apabila daya-daya yang bertindak ke atas suatu objek dalam keadaan antara satu sama lain. Daya bersih adalah sifar .
seimbang , ia akan membatalkan
balanced , they cancel each other out. The net force is
When the forces acting on an object are
zero .
Kesan/Effect: Objek berada dalam keadaan rehat [halaju = 0 ] atau bergerak pada The object is at
rest
[velocity = 0] or moves at constant
velocity
malar [pecutan = 0 ]
[acceleration =
Daya dikenakan oleh meja ke atas cawan Force exerted by table on the cup
U N I T
halaju
0 ]
Daya angkat, U/Lift, U
Tujahan, F Thrust, F
Seretan, G Drag, G
2 Berat Weight
Berat, W Weight, W
Cawan itu berada dalam keadaan rehat. Daya Kapal terbang bergerak dengan halaju malar. Daya bersih yang bertindak ke atasnya adalah sifar . bersih yang bertindak ke atasnya adalah sifar . The cup stays at rest. The net force acting on it is zero . W = R di mana/where W : Berat/Weight R : Tindak balas normal/Normal reaction
The plane moves with constant velocity. The net force acting zero . on it is
W = U di mana/where F = G di mana/where W : Berat/Weight F : Tujahan/Thrust U : Daya angkat/Lift G : Seratan/Drag
Huraikan kesan daya tidak seimbang yang bertindak ke atas suatu objek Describe the effects of unbalanced forces acting on an object
Apabila daya yang bertindak ke atas objek ke atasnya.
not balanced , there must be a
When the forces acting on an object are
Daya bersih dikenali sebagai daya The net force is known as the
tidak seimbang , terdapat daya net
bersih
yang bertindak
force acting on it.
paduan yang bertindak ke atasnya.
resultant
force acting on it.
Daya tidak seimbang = daya bersih = daya yang dikenakan – daya geseran The unbalanced force = net force = force applied – frictional force Kesan: Boleh menyebabkan badan seseorang/Effect: Can cause a body to
• bertukar keadaan rehatnya (objek itu akan memecut) change its state at rest (an object will accelerate)
• bertukar keadaan gerakannya (suatu objek yang bergerak akan memecut / nyahpecut atau menukar arahnya) change its state of motion (a moving object will accelerate/decelerate or change its direction) © Nilam Publication Sdn. Bhd.
03 FIZ Form 4 Ch2 B 2F.indd 58
58
9/10/13 4:30 PM
MODUL • Fizik TINGKATAN 4
Menentukan hubungan antara daya, jisim dan pecutan (F = ma)
Determine the relationship between force, mass and acceleration (F = ma)
Hukum Gerakan Newton Kedua
Pecutan yang dihasilkan oleh daya ke atas suatu objek adalah berkadar langsung dengan magnitud daya bersih yang dikenakan dan berkadar
Newton’s Second Law of Motion
songsang
The acceleration produced by a force on an object is directly proportional to the magnitude of the net force applied and is inversely proportional to the mass of the object.
F=5N
m = 25 kg
Hubungan antara a dan F Relationship between a and F
Hubungan antara a dan m Relationship between a and m
Eksperimen Experiment
dengan jisim objek itu.
aya = Jisim × Pecutan D F = ma
Force = Mass × Acceleration F = ma
aαF Pecutan, a, berkadar langsung dengan daya yang dikenakan, F
a
The acceleration, a, is directly proportional to the applied force, F
0
1 m Pecutan, a, bagi suatu objek berkadar songsang dengan jisimnya, m
F
a
aα
U N I T
2
The acceleration, a, of an object is inversely proportional to its mass, m
0
1 m
Mencari hubungan antara daya, jisim dan pecutan
Find the relationship between force, mass and acceleration
Hubungan antara
aαF
Relationship between
aαm
A
A
B
B
Situasi Situation
Dua orang pemuda menolak jisim yang sama tetapi pemuda A menolak dengan daya yang lebih besar. Jadi dia bergerak dengan lebih cepat.
Dua orang pemuda mengeluarkan daya yang sama. Tetapi pemuda B bergerak dengan lebih cepat daripada pemuda A.
Both men exerted the same force. But man B Both men are pushing the same mass but man A moves faster than man A. pushes with a greater force. So he moves faster.
Semakin besar daya, semakin besar pecutan. Semakin besar jisim, semakin kecil pecutan. Hipotesis
The larger the force, the greater the
The greater the mass, the smaller the
Hypothesis
acceleration.
acceleration.
59
03 FIZ Form 4 Ch2 B 2F.indd 59
© Nilam Publication Sdn. Bhd.
9/10/13 4:30 PM
MODUL • Fizik TINGKATAN 4
Pemboleh ubah dimanipulasi
Daya/Force
Jisim/Mass
Pemboleh ubah bergerak balas
Pecutan/Acceleration
Pecutan/Acceleration
Pemboleh ubah dimalarkan
Jisim/Mass
Daya/Force
Manipulated variable
Responding variable
Constant variable
Jangka masa detik dan pita detik, bekalan kuasa, landasan terpampas geseran, Bahan dan radas
pembaris, troli, takal licin (dengan pengapit), tali tak kenyal, pemberat berslot
Materials and apparatus
Ticker timer and ticker tape, power supply, friction-compensated runaway, ruler, trolley, smooth pulley (with clamp), inelastic string, slotted weights Pita detik Ticker tape
U N I T
2
Jangka masa detik Ticker timer
Troli P Trolley P
Tali tak kenyal Inelastic string
Bekalan kuasa a.u. a.c. power supply
Rajah
Landasan terpampas geseran Friction-compensated runway
Blok kayu Wooden block
Diagram
Takal licin Smooth pulley
Susunan radas untuk mengkaji hubungan antara (1) daya dan pecutan (2) jisim dan pecutan
Pemberat berslot Slotted weight
Arrangement of apparatus to investigate the relationship between (1) Force and acceleration (2) mass and acceleration
1 Radas disusun seperti ditunjukkan 1 Radas disusun seperti ditunjukkan dalam rajah di atas. dalam rajah di atas. The apparatus is set up as shown in the diagram above.
The apparatus is set up as shown in the diagram above.
2 Sebuah troli berjisim 1.0 kg (jisim 2 Sebuah lori dengan jisim , m = 1.0 kg malar) diletakkan di atas landasan. Pita diletakkan di atas landasan. Pita detik detik dilekat pada troli itu. dilekat pada troli itu. A trolley of mass 1.0 kg (constant mass) is placed on the runway. A length of ticker tape is attached to the trolley.
Prosedur Procedure
A trolley of mass, m = 1.0 kg is placed on the runway. A length of ticker-tape is attached to the trolley.
3 Jangka masa detik dihidupkan dan troli 3 Jangka masa detik dihidupkan dan troli itu ditarik oleh pemberat yang mempunyai itu ditarik oleh pemberat (daya malar daya, F = 10.0 N. pemberat ini ialah 10 N) The ticker timer is switched on and the trolley is pulled by a weight of force, F = 10.0 N.
The ticker timer is switched on and the trolley is pulled by a weight of constant force, 10 N
4 Dari pita detik yang di peroleh , pecutan 4 Dari pita detik yang diperoleh, pecutan troli dihitung dengan menggunakan troli dihitung dengan menggunakan (v – u) formula, a = (v – u) . formula, a = t t From the ticker tape obtained, the acceleration of the trolley is calculated by (v – u) using the formula, a = . t
© Nilam Publication Sdn. Bhd.
03 FIZ Form 4 Ch2 B 2F.indd 60
From the ticker tape obtained, the acceleration of the trolley is calculated by (v – u) using the formula, a = . t
60
9/10/13 4:30 PM
MODUL • Fizik TINGKATAN 4
5 Langkah-langkah 2 – 4 diulangi dengan 5 Langkah-langkah 2 – 4 diulangi dengan menambahkan pemberat berslot supaya melekat pemberat berslot pada troli F = 15.0 N, 20.0 N, 25.0 N dan 30.0 N. supaya jisim troli, m = 1.5 kg, 2.0 kg, Steps 2 – 4 are repeated by adding slotted 2.5 kg dan 3.0 kg. weights to pull the trolley so that F = 15.0 N, 20.0 N, 25.0 N and 30.0 N.
Daya, F/N
Pecutan, a/cm s–2
Force, F/N
Acceleration, a/cm s
-2
10.0
Mass, m/kg
Pecutan, a/m s–2
Acceleration, a/cm s–2
1.5
20.0
Recording data
Jisim, m/kg 1.0
15.0
Merekodkan data
Steps 2 – 4 are repeated by taping up slotted weights to the trolley to give m = 1.5 kg, 2.0 kg, 2.5 kg and 3.0 kg.
25.0
2.5
30.0
2.5 3.0
Pecutan, a/cm s–2 Acceleration, a/cm s–2
U N I T
Pecutan, a/cm s–2 Acceleration, a/cm s–2
2
Menganalisis data Analysing data
Daya, F/N Force, F/N
0
Jisim, m/kg Mass, m/kg
0
Menyelesaikan masalah menggunakan F = ma Solve problems using F = ma
1 Hitungkan pecutan bagi blok di bawah:/Calculate the acceleration of the block: m = 2 kg m = 10 kg (a) (c) F = 8.0 N
a =
F = 8.0 N m 2 kg
a = (18 – 2) N = 16 N 10 kg 10 kg
= 4 m s-2
F=6N
(14 + 6) N 20 N = 8 kg 8 kg = 2.5 m s-2 a =
F = 14 N
(d)
m = 12 kg F=5N R=5N
F = 10 N
a = (10 – 5 – 5) N = 0 12 kg = 0 m s-2
61
03 FIZ Form 4 Ch2 B 2F.indd 61
= 1.6 m s-2
m = 8 kg
(b)
F = 18 N
F=2N
© Nilam Publication Sdn. Bhd.
9/10/13 4:30 PM
MODUL • Fizik TINGKATAN 4
Menyelesaikan masalah menggunakan F = ma Solve problems using F = ma
2 Seorang lelaki menolak troli yang berisi kotak (jumlah jisim 5 kg) di atas permukaan yang licin. Jika dia menggunakan daya 30 N untuk menolak troli itu, apakah magnitud dan arah pecutan troli itu? A man pushes a trolley with a box (total mass 5 kg) on a smooth surface. If he uses a force of 30 N to move the trolley, what is the magnitude and direction of the acceleration of the trolley?
F = 30 N
5 kg
Penyelesaian/Solution: F = ma a = 30 N = 6 m s-2 ke kanan / to the right. 5 kg
3 Sebuah objek yang berjisim 2 kg ditarik di atas tanah dengan daya 5 N dan halaju malar. An object of mass 2 kg is pulled on the floor by a force of 5 N and has a constant velocity. U N I T
2
(a) Berapakah daya geseran antara objek dan tanah? What is the frictional force between the object and the floor?
(b) Hitungkan pecutan objek itu jika objek itu ditarik dengan daya 17 N.
R
F1 = 5 N
Calculate the acceleration of the object if the object is pulled by a 17 N force.
Penyelesaian/Solution: (a) R ialah daya geseran/R is the frictional force F1 – R = ma ∴ R = F1 – ma Oleh kerana halaju malar/Because the velocity is constant, a = 0 ∴ R = F1 – 0 = F1 = 5 N (b) F2 – R = ma 17 N – 5 N = (2 kg) (a) a = 12 N = 6 m s–2 2 kg 4 Sebuah bas berjisim 2 000 kg bergerak dengan halaju seragam 40 m s-1 sejauh 2 500 m sebelum berehat. Hitungkan
A bus of mass 2 000 kg travels at a uniform velocity 40 m s-1 for a distance of 2 500 m before it comes to rest. Calculate (a) purata nyahpecutan bas itu./the average deceleration of the bus.
(b) purata daya yang dikenakan oleh brek itu untuk membolehkan bas itu berhenti bergerak. the average force applied by the brakes to bring the bus to a standstill.
Penyelesaian/Solution: (b) F = ma (a) v2 = u2 + 2as = (2 000 kg)(–0.32 m s–2) 0 = (40 m s–1)2 + 2a(2 500 m) = –640 N ∴ 5 000a = –1 600 m s–2 -2 (Negatif bermaksud daya untuk menentang a = –0.32 m s gerakan/Negative means force to resist the motion)
© Nilam Publication Sdn. Bhd.
03 FIZ Form 4 Ch2 B 2F.indd 62
62
9/10/13 4:30 PM
MODUL • Fizik TINGKATAN 4
MENGANALISIS IMPULS DAN DAYA IMPULS
2.6
ANALYSING IMPULSE AND IMPULSIVE FORCE
Menerangkan daya impuls/Explain what an impulsive force is Daya yang besar yang bertindak dalam tempoh masa yang singkat semasa perlanggaran atau letupan daya impuls . dikenali sebagai A large force that acts over a short period of time during a collision or explosion is known as an impulsive force .
Daripada hubungan antara daya, jisim dan pecutan: From the relationship between force, mass and acceleration:
F = ma = m ( v – u ) t F = mv – mu = perubahan momentum t t m = jisim/mass
Unit =
u = halaju awal/initial velocity
N
=
t = masa/time
kg m s-2 v = halaju akhir/final velocity U N I T
Daya impuls ialah kadar perubahan momentum dalam perlanggaran atau letupan. An impulsive force is the rate of change of momentum in a collision or explosion.
2
Mendefinisikan impuls/Define impulse Impuls didefinisikan sebagai perubahan momentum /Impulse is defined as the
change of momentum .
atau (momentum akhir – momentum awal) atau (mv – mu) or
(final momentum – initial momentum)
Unit:
kg m s-1
or
(mv – mu)
Ns
atau/or
Daripada F = mv – mu , Ft = mv – mu = perubahan momentum = impuls t From F =
mv – mu , t
Ft = mv – mu = change of momentum = impulse
Impuls ialah hasil darab antara
daya
The product of the force and the
dan
masa .
time is called the impulse.
Kesan peningkatan dan pengurangan masa perlanggaran The effects of increasing and decreasing the time of collision
Daripada formula, F = Perubahan momentum /From the formula, F = Change of momentum . Time Masa Daya impuls berkadar Impulsive force is
songsang
inversely
dengan masa sentuhan atau tindakan atau perlanggaran.
proportional to the time of contact or impact or collision.
Daya impuls kecil / Impulsive force is Daya impuls besar / Impulsive force is
Tempoh masa yang panjang/Longer period of time Tempoh masa yang pendek/Shorter period of time 63
03 FIZ Form 4 Ch2 B 2F.indd 63
small large
© Nilam Publication Sdn. Bhd.
9/10/13 4:30 PM
MODUL • Fizik TINGKATAN 4
Terangkan situasi di mana daya impuls perlu dikurangkan dan kaedah untuk mengurangkannya Explain situations where an impulsive force needs to be reduced and suggest ways to reduce it
Situasi
Penjelasan
Situations
Explanation
Peserta lompat tinggi mendarat di atas tilam yang tebal. Mengapa? A high jumper lands on the thick mattress. Why?
Masa tindakan yang panjang, maka daya impuls menjadi kecil. The time of impact is longer, so the impulsive force is smaller.
Penjaga gol memakai sarung tangan untuk menangkap bola. Mengapa? Goal keepers wear gloves to catch a ball. Why?
U N I T
Masa tindakan yang panjang, maka daya impuls menjadi kecil. The time of impact is longer, so the impulsive force is smaller.
2
Ahli lompat jauh akan membengkokkan kaki semasa mendarat. Mengapa? A long jumper will bend his legs upon landing. Why?
Masa tindakan adalah lebih panjang, maka daya impuls menjadi kecil. Ini mengurangkan kecederaan./The time of impact is longer, so the impulsive force is smaller. This reduces injury.
Seorang pemain besbol mesti menangkap bola mengikut arah pergerakan bola. Mengapa? A baseball player must catch the ball in the direction of the motion of the ball. Why?
Masa tindakan lebih panjang, maka daya impuls menjadi lebih kecil. The time of impact is longer, so the impulsive force is smaller.
Bahan yang mudah pecah seperti telur, kaca dan perkakasan elektrik mestilah dibungkus dalam bahan yang lembut dan boleh dimampatkan. Mengapa? Items that are fragile such as eggs, glass and electrical appliances must be packed in materials that are soft and compressible. Why?
Bahan yang lembut dan mudah dimampatkan menghasilkan masa perlanggaran yang panjang. Ia menyerap hentakan. Jadi ia mengurangkan daya impuls. Soft and compressible material provides longer time of impact. It absorbs the shock. So it can reduce the impulsive force.
© Nilam Publication Sdn. Bhd.
04 FIZ Form 4 Ch2 C 2F.indd 64
64
9/10/13 4:28 PM
MODUL • Fizik TINGKATAN 4
Terangkan situasi di mana daya impuls mendatangkan faedah Explain situations where an impulsive force is beneficial
Situasi
Penjelasan
Situations
Explanation
Peserta karate yang mahir boleh memecahkan kayu yang tebal dengan menggunakan sisi tangan yang bergerak dengan kelajuan yang sangat tinggi. A karate expert can break a thick wooden slab with his bare hand that moves at a very fast speed.
Tangan tersebut digerakkan pada halaju yang tinggi. Masa tindakan adalah singkat. Menghasilkan daya impuls yang besar. The hand is moved at high velocity. Time of impact is shorter. Produce large impulsive force.
Kepala penukul yang besar bergerak pada kelajuan yang tinggi untuk memukul paku. A massive hammer head moving at a fast speed is brought to rest upon hitting the nail.
Masa tindakan adalah pendek apabila kepala penukul memukul pada
U N I T
2
halaju yang tinggi. Ia akan menghasilkan daya impuls yang besar. Time of impact is shorter when the hammer head is hit at high velocity. It produces large impulsive force.
Sebiji bola sepak mestilah mempunyai tekanan udara yang cukup tinggi. A football must have an air pressure that is high enough.
Sebiji bola sepak yang mempunyai tekanan udara yang cukup tinggi akan mempunyai masa tindakan yang pendek. Maka daya impuls adalah besar. Bola akan bergerak lebih jauh. Daya impuls Impulsive force
A football which has a high-enough air pressure will have a short time of impact. So, the impulsive force is large. The ball will move further.
Antan dan lesung diperbuat daripada batu. A pestle and mortar are made of stone.
Masa tindakan adalah kecil. Maka daya impuls adalah besar.
Antan Pestle Daya impuls Impulsive force
Time of impact is shorter. So the impulsive force is large.
Lesung/Mortar
65
04 FIZ Form 4 Ch2 C 2F.indd 65
© Nilam Publication Sdn. Bhd.
9/10/13 4:28 PM
MODUL • Fizik TINGKATAN 4
Latihan/Exercises 1 Rosli yang berjisim 60 kg melompat dari tingkat pertama sebuah rumah yang terbakar. Halajunya sejurus sebelum mendarat ialah 6 m s-1.
2 Rooney menyepak bola dengan kekuatan daya 1 500 N. Masa tindakan di antara kasut dan bola ialah 0.01 s. Berapakah impuls yang dikenakan kepada bola itu? Jika jisim bola itu ialah 0.5 kg, berapakah halaju bola tersebut?
Rosli, with a mass of 60 kg, jumps from the first floor of a burning house. His velocity just before landing on the ground is 6 m s-1.
Rooney kicks a ball with a force of 1 500 N. The time of contact of his boot with the ball is 0.01 s. What is the impulse delivered to the ball? If the mass of the ball is 0.5 kg, what is the velocity of the ball?
(a) Kirakan impuls apabila kakinya mencecah tanah. Calculate the impulse when his legs hit the ground.
(b) Berapakah daya impuls yang bertindak ke atas kaki Rosli jika dia membengkokkan kaki ketika mendarat dan mengambil masa 0.5 s untuk berhenti?
Penyelesaian/Solution Impuls/Impulse = Ft = 1 500 N × 0.01 s = 15.0 N s = 15.0 kg m s-1
What is the impulsive force on Rosli’s legs if he bends upon landing and takes 0.5 s to stop?
U N I T
2
mv – mu = 15 kg m s-1 (0.5 kg)v = 15 kg m s-1 v = 30 m s-1
(c) Berapakah daya impuls yang bertindak ke atas kaki Rosli jika dia tidak membengkokkan kaki dan berhenti dalam 0.05 s?
3 Dalam satu perlawanan tenis, pemain memukul bola yang mempunyai jisim 0.2 kg yang menuju ke arahnya dengan halaju 20 m s-1. Bola itu memantul dengan halaju 40 m s-1. Masa yang diambil semasa perlanggaran antara bola dan raket tenis ialah 0.01 s.
What is the impulsive force on Rosli’s legs if he does not bend and stops in 0.05 s?
(d) Apakah kebaikan membengkokkan kaki semasa mendarat? What is the advantage of bending his legs upon landing?
In a tennis match, a player hits an on-coming ball with mass of 0.2 kg and velocity of 20 m s-1. The ball rebounds with a velocity of 40 m s-1. The time taken in the collision between the ball and the tennis racket is 0.01 s.
Penyelesaian/Solution (a) mv – mu = m (v – u) = (60 kg) (0 – 6) m s–1 = –360 kg m s–1 Impuls adalah –360 kg m s–1 kerana momentumnya dikurangkan sehingga sifar.
u = –20 m s-1
The impulse is –360 kg m s–1 because its momentum is reduced to zero.
v = 40 m s-1
–1 (b) F = mv – mu = –360 kg m s 0.5 s t = –720 N –1 mv – mu (c) F = = –360 kg m s 0.05 s t = –7 200 N (b) dan (c) : Tanda negatif bagi daya bermakna daya ini telah menyebabkan kehilangan momentum
0.2 kg t = 0.01 s
(a) Berapakah impuls yang dialami oleh bola itu? What is the impulse experienced by the ball?
(b) Berapakah daya impuls yang dikenakan ke atas bola tenis? What is the impulsive force exerted on the tennis ball?
(b) and (c) : The negative sign to the force means that the force has caused a loss of momentum
Penyelesaian/Solution (a) Impuls = m (v – u) Impulse = (0.2 kg)[(40 – (–20)] m s–1 = 12.0 kg m s-1 = 12.0 N s 12 N s = 1 200 N (b) F = 0.01 s
(d) Dengan membengkokkan kaki semasa mendarat, dia akan meningkatkan masa tindakan dan mengurangkan daya impuls. Jadi ia dapat mengurangkan kecederaan.
By bending his legs upon landing, he will increase the time of impact and reduce the impulsive force. So it will minimise the injuries.
© Nilam Publication Sdn. Bhd.
04 FIZ Form 4 Ch2 C 2F.indd 66
u=0
66
9/10/13 4:28 PM
MODUL • Fizik TINGKATAN 4
2.7
KESELAMATAN KENDERAAN VEHICLE SAFETY
Cermin hadapan kereta Windscreen
Pelapik kepala Headrest
Beg udara Air bag
Zon kemek belakang Rear crumple zone
Stering Steering
Zon remuk depan Front crumple zone
Bumper depan Front bumper
Bar hentaman sisi Side impact bar
Tali pinggang keselamatan Safety belt
Sistem brek anti kunci Anti-lock braking system (ABS)
Komponen/Component Pelapik kepala Headrest
U N I T
2 Fungsi/Function
Untuk mengurangkan kesan inersia terhadap kepala pemandu. Mengurangkan kecederaan leher apabila kereta dilanggar daripada belakang. To reduce the inertia effect on the driver’s head. Reduce neck injury when the car is hit from behind.
Beg udara Air bag
Menyerap hentakan dengan menambahkan masa perlanggaran apabila kepala pemandu terhentak ke stereng. Oleh itu daya impuls dikurangkan. Absorbing impact by increasing the collision time when the driver’s head is thrown towards the steering. So the impulsive force is reduced.
Cermin hadapan kereta/ Cermin keselamatan
Kaca tahan pecah yang tidak akan mudah pecah kepada serpihan yang kecil dengan
Windscreen/safety glass
mudah semasa perlanggaran. Mengurangkan kecederaan disebabkan oleh serpihan kaca yang berselerak. Shatterproof glass that will not break into small pieces easily during collision. This will reduce injuries caused by scattered glass.
67
04 FIZ Form 4 Ch2 C 2F.indd 67
© Nilam Publication Sdn. Bhd.
9/10/13 4:28 PM
MODUL • Fizik TINGKATAN 4
Komponen/Component
Fungsi/Function
Zone remuk (depan dan belakang)
Boleh dimampatkan ketika kemalangan. Jadi ia akan meningkatkan masa yang
Crumple zone (front and rear)
diperlukan kereta untuk berhenti sepenuhnya. Maka ia akan kaca yang mengurangkan daya impuls. Can be compressed during an accident. So it can increase the time taken by the car to come to a complete stop. So it can reduce the impulsive force.
Bumper depan Front bumper
Menyerap hentakan akibat daripada kemalangan. Diperbuat daripada keluli, aluminium, plastik, getah dan fiber komposit. Absorb the shock from the accident. Made from steel, aluminium, plastic, rubber and composite fibres.
U N I T
2
Sistem brek anti kunci Anti-lock braking system (ABS)
Membolehkan pemandu memberhentikan kereta dengan segera tanpa menyebabkan roda terkunci apabila brek ditekan secara tiba-tiba. Mengelakkan kereta daripada menggelongsor. Enables drivers to quickly stop the car without the wheels locking when the brake is applied suddenly. Prevents the car from skidding.
Bar hentaman sisi Side impact bar
Boleh dimampatkan ketika kemalangan. Jadi ia akan meningkatkan masa yang diperlukan kereta untuk berhenti sepenuhnya. Maka ia akan mengurangkan daya impuls. Can be compressed during accident. So it can increase the time the car takes to come to a complete stop. So it can reduce the impulsive force.
Tali pinggang keselamatan
Untuk mengurangkan kesan inersia dengan mengelakkan pemandu daripada
Safety belt
tercampak ke hadapan. To reduce the inertia effect by preventing the driver from being thrown forward.
Papan pesawat Dashboard
Semasa perlanggaran, papan pesawat meroboh. Ini akan menyerap kesan hentaman dengan meningkatkan masa perlanggaran antara kepala pemandu dan stereng. Jadi ia mengurangkan daya impuls. During collision, the dashboard collapses. This will absorb the impact by increasing the time of collision between the driver's head and the steering. This will reduce the impulsive force.
© Nilam Publication Sdn. Bhd.
04 FIZ Form 4 Ch2 C 2F.indd 68
68
9/10/13 4:28 PM
MODUL • Fizik TINGKATAN 4
2.8
MEMAHAMI GRAVITI
UNDERSTANDING GRAVITY
Kekuatan medan graviti/Gravitational field strength • Objek jatuh bebas kerana ditarik ke arah pusat bumi oleh daya tarikan
graviti
.
Objects experience free fall because they are pulled towards the centre of the Earth by the force of
• Kekuatan medan graviti =
gravity
.
daya tarikan graviti gravitational force . . / Gravitational field strength = mass jisim
• Dipermukaan Bumi, At the surface of the Earth, –1 gravitational field strength = 10 N kg–1 Kekuatan medan graviti = 10 N kg –2 = 10 m s = 10 m s–2 • Setiap kilogram jisim pada permukaan Bumi mengalami daya graviti sebanyak 10 N yang bertindak ke atasnya. Earth kilogram of mass at the Earth's surface has a gravitational force of 10 N acting on it.
Aktiviti 1: Pecutan disebabkan graviti Activity 1: Acceleration due gravity
U N I T
Rajah di sebelah menunjukkan gambarfoto stroboskop bagi bola yang jatuh bebas dan graf halaju lawan masa bagi gerakannya. The diagram on the right shows a stroboscopic photograph of a free falling ball and its velocity-time graph.
(a) Perhatikan gambarfoto dan terangkan halaju bola.
2
V
Observe the photograph and describe the velocity of the ball.
Halaju bola itu meningkat dengan seragam. The velocity of the ball increases uniformly.
0
t
(b) Apakah yang boleh anda simpulkan daripada graf kecerunan v – t?
What can we deduce from the gradient of the v – t graph?
Kecerunan ialah pecutan bola itu. The gradient is the acceleration of the ball.
(c) Terangkan gerakan bola tersebut.
Describe the motion of the ball.
Bola tersebut bergerak dengan pecutan seragam. The ball moves with constant acceleration.
Terangkan pecutan yang disebabkan oleh graviti, g/Explain acceleration due to gravity, g Pecutan disebabkan oleh graviti, g, ialah pecutan bagi objek yang disebabkan oleh daya tarikan graviti. Acceleration due to gravity, g, is the acceleration of an object due to the pull of the
gravitational force
.
Nilai piawai bagi pecutan graviti, g, ialah 9.81 m s-2. Nilai g yang sering digunakan ialah 10 m s-2. kekuatan medan graviti Magnitud bagi pecutan yang disebabkan oleh graviti bergantung pada . The standard value of the gravitational acceleration, g, is 9.81 m s-2. The value of g is often taken to be 10 m s-2 for simplicity. The magnitude of the acceleration due to gravity depends on the gravitational field strength .
69
04 FIZ Form 4 Ch2 C 2F.indd 69
© Nilam Publication Sdn. Bhd.
9/10/13 4:28 PM
MODUL • Fizik TINGKATAN 4
Apakah jatuh bebas?/What is free fall? Objek dikatakan 'jatuh bebas' apabila ia jatuh di bawah daya tarikan gravitational
An object is falling freely when it is falling under the
Objek hanya jatuh bebas di dalam vakum menentang pergerakan objek. An object falls freely only in the motion of the object.
vacuum
sahaja.
force only.
Sehelai kertas tidak jatuh bebas kerana kejatuhannya dipengaruhi oleh A piece of paper does not fall freely because its fall is affected by
graviti
rintangan udara
air resistance
.
.
. Ketiadaan udara bermaksud tiada rintangan udara yang
. The absence of air means there is no air resistance to
resist
Di dalam vakum, kedua-dua objek yang ringan dan berat jatuh bebas. Ia jatuh dengan pecutan graviti iaitu pecutan disebabkan oleh graviti, g. U N I T
In vacuum, both light and heavy objects fall freely. They fall with the the acceleration due to gravity, g.
2
gravitational
acceleration, that is
Aktiviti 2: Pecutan disebabkan graviti/Activity 2: Acceleration due gravity Pegang dua biji batu yang berbeza saiz pada ketinggian yang sama, kemudian kedua-dua batu itu dijatuhkan serentak daripada ketinggian yang sama. Hold two stones of different sizes at the same height and then drop both stones simultaneously from the same height.
(a) Huraikan bagaimana halaju berubah. Describe how the velocity changes. Halaju meningkat dengan seragam. The velocity increases uniformly.
(b) Bandingkan masa yang diambil untuk batu mencecah lantai.
Compare the time taken for the stones to reach the floor. Sama/same
(c) Adakah pecutan batu dipengaruhi oleh jisimnya? Is the acceleration of each stone influenced by its mass? Jisim tidak mempengaruhi pecutan. Mass does not affect the acceleration.
© Nilam Publication Sdn. Bhd.
04 FIZ Form 4 Ch2 C 2F.indd 70
70
9/10/13 4:28 PM
MODUL • Fizik TINGKATAN 4
Aktiviti 3: Yang mana satukah mencecah tanah dahulu? Activity 3: Which one reaches the ground first?
Bola golf Golf ball
Kertas Paper
Bola golf Golf ball
Kertas yang direnyukkan Paper which is crumpled
Bola golf dan sehelai kertas dipegang pada ketinggian yang sama dan dijatuhkan serentak.
Ulangi dengan bola golf dan sehelai kertas yang direnyukkan.
Hold a golf ball and a piece of paper at the same height and drop them simultaneously.
Repeat with a golf ball and a piece of paper which is crumpled.
(a) Objek yang manakah mencecah tanah dahulu?
(a) Objek yang manakah mencecah tanah dahulu?
Which object reaches the floor first?
Which object reaches the floor first?
Bola golf.
Kedua-duanya mencecah tanah pada masa
The golf ball.
yang sama.
(b) Terangkan mengapa./Explain why. Kertas mempunyai luas pemukaan yang besar.
Both reach the floor at the same time.
(b) Terangkan mengapa./Explain why. Kedua-dua objek mempunyai saiz dan luas
Jadi lebih banyak rintangan udara yang bertindak ke atasnya.
pemukaan yang sama. Jisim tidak memberi
The paper has large surface area. As such, the air
kesan kepada pecutan graviti.
resistance acting on it is big.
Both objects have same size and surface area.
U N I T
2
Mass does not affect gravitational acceleration.
Aktiviti 4: Perbezaan antara jatuh bebas di atmosfera (udara) dan jatuh bebas di dalam vakum bagi duit syiling dan bulu pelepah.
Activity 4: The difference between free fall in atmosphere and free fall in a vacuum of a coin and a feather.
Duit syiling dan bulu pelepah dilepaskan daripada ketinggian yang sama serentak di dalam makmal. A coin and a feather are released from the same height simultaneously in the laboratory.
Duit syiling dan bulu pelepah yang sama diletakkan di dalam satu tiub vakum dan kemudian dijatuhkan serentak pada ketinggian yang sama. The same coin and feather are put into a vacuum tube and then dropped simultaneously from the same height.
Pemerhatian/Observation
Pemerhatian/Observation Duit syiling jatuh lebih pelepah. The coin falls
faster
cepat
daripada bulu
than the feather.
Kedua-dua objek mencecah ke bawah silinder sama . pada masa yang Both objects reach the bottom of the cylinder at the same time.
71
04 FIZ Form 4 Ch2 C 2F.indd 71
© Nilam Publication Sdn. Bhd.
9/10/13 4:28 PM
MODUL • Fizik TINGKATAN 4
Penjelasan/Explanation
Penjelasan/Explanation Di dalam keadaan vakum, tiada rintangan udara. Hanya terdapat satu daya yang bertindak ke atas graviti . objek iaitu daya
Rintangan udara
yang besar bertindak ke atas bulu pelepah kerana ia mempunyai luas permukaan yang besar. air resistance
A bigger
Kedua-dua objek jatuh bebas dengan pecutan yang disebabkan graviti walaupun berbeza dari
graviti
pada duit syiling mampu untuk rintangan udara lebih baik daripada
mengatasi bulu pelepah. overcome U N I T
2
segi
jisim
Both objects
dan
bentuk
.
free fall
with acceleration due to mass gravity despite the differences in their and shapes .
gravitational
The
air resistance
. The only force acting on both objects is the force of gravity .
surface area .
it has a large
Daya
In vacuum, there is no
acts the feather because
force on the coin is able to air resistance better than the feather.
g sebagai kekuatan medan graviti. g sebagai pecutan yang disebabkan graviti g as gravitational field strength. g as acceleration due to gravity
1 Sebuah objek yang jatuh bebas berdekatan dengan permukaan bumi akan memecut pada 10 m s-2. An object falling freely near the earth's surface will
accelerate
at 10 m s-2.
2 Setiap kilogram jisim yang berdekatan dengan permukaan bumi mempunyai daya graviti 10 N yang bertindak ke atasnya. Each kilogram of mass near the earth’s surface has a gravitational force of 10 N acting on it.
Kekuatan medan
graviti
Gravitational
, g = 10 N kg-1/
field strength, g = 10 N kg-1
Pecutan disebabkan graviti/Ac celeration due to gravity, g = 10 m s-2
Nilai g boleh ditulis sebagai 10 m s-2 atau 10 N kg-1.
The approximate value of g can be written either as 10 m s-2 or as 10 N kg-1.
Aktiviti 5: Pecutan yang disebabkan graviti Activity 5: Acceleration due to gravity
Tujuan/Aim Menentukan pecutan disebabkan graviti/To determine the acceleration due to gravity. Radas/Apparatus Jangka masa detik, bekalan kuasa 12 V, bangku, pengapit-G, pemberat, pita detik Ticker timer, 12 V ac power supply, stool, G-clamp, slotted weight, ticker tape.
Prosedur/Procedure 1 Potong sekeping pita detik lebih kurang 2.5 m panjang dan lalukan melalui jangka masa detik yang diapit kepada kerusi oleh pengapit-G. Cut a piece of ticker tape about 2.5 m long and pass through the ticker timer which is clamped to a stool using G-clamp. © Nilam Publication Sdn. Bhd.
04 FIZ Form 4 Ch2 C 2F.indd 72
72
9/10/13 4:28 PM
MODUL • Fizik TINGKATAN 4
2 Sambungkan satu hujung pita pada pemberat 100 g. Attach one end of the tape to the 100 g slotted weight.
3 Hidupkan jangka masa detik dan pemberat dilepaskan supaya ia jatuh bebas. Switch on the ticker timer and release the slotted weight so that it falls freely.
Pita detik Ticker tape
Bekalan kuasa, 12 V
Pengapit-G G-clamp
A.C. Power supply, 12 V
4 Kaji pita itu untuk menentukan nilai bagi pecutan disebabkan oleh graviti, g.
Jangka masa detik Ticker timer
Bangku Stool
Analyse the tape to determine the value of the acceleration due to gravity, g.
Pemberat Weight
Perbincangan/Discussion
Kepingan polisterena Polystyrene sheet
1 Mengapakah sukar untuk menentukan pergerakan objek yang jatuh dengan hanya memerhatikannya jatuh? Why is it difficult to describe the motion of a falling object by just observing it fall?
Objek bergerak sangat laju. The object moves very fast.
U N I T
2 Apakah jenis pergerakan objek jika ia jatuh di bawah tarikan graviti? What is the type of motion of an object falling under the pull of gravity? Pecutan seragam./Constant acceleration.
2
3 Mengapakah pergerakan pemberat boleh diandaikan sebagai jatuh bebas? Why is it that the motion of the slotted weight can be assumed to be a free fall?
Ringtangan udara yang kecil boleh diabaikan. The small air resistance is negligible.
4 Apakah langkah yang akan anda ambil untuk mengurangkan geseran antara pita dan jangka masa detik? What steps did you take to minimise the friction between the ticker tape and the ticker timer?
Pegang pita detik dalam keadaan menegak dan lepaskannya. Pastikan ia jatuh mealui jangka masa detik dengan lancar. Hold the ticker tape vertically when releasing it. Make sure it slips through the ticker timer smoothly.
5 Terangkan mengapa perlu menjatuhkan pemberat daripada kedudukan yang tinggi. Explain the need for the slotted weight to be dropped from a high position.
Pengiraan akan menjadi lebih tepat kerana ralat eksperimen dikurangkan. The calculation will be more accurate because experimental errors are reduced.
6 Tunjukkan bagaimana anda mengira g daripada pita. Show how you would calculate g from the tape.
u = s1 v = s2 t1 t2 v – u a= t
73
04 FIZ Form 4 Ch2 C 2F.indd 73
© Nilam Publication Sdn. Bhd.
9/10/13 4:28 PM
MODUL • Fizik TINGKATAN 4
7 Bandingkan nilai pecutan disebabkan oleh graviti daripada aktiviti ini dengan nilai yang sebenar. Berikan alasan yang munasabah bagi perbezaan di antara dua nilai tersebut. Compare the value of the acceleration due to gravity from this activity with the actual value. Give possible reasons for any difference in these two values.
Nilai daripada eksperimen adalah lebih rendah berbanding dengan nilai sebenar. Sebabnya ialah rintangan akibat jangka masa detik. / The value from the experiment is lower than the actual value. The reason is the resistance caused by the ticker timer.
8 Bandingkan nilai bagi g daripada eksperimen jika anda mengulangi eksperimen dengan menggunakan pemberat 200 g dan 300 g. Compare the values of g from the experiment if you repeat the experiment using 200 g and 300 g weights.
Keputusan sepatutnya sama. The result should be the same.
9 Apakah yang boleh anda simpulkan tentang hubungan antara g dan jisim bagi objek yang jatuh? What can you conclude about the relationship between g and the mass of the falling object?
U N I T
Jisim tidak mempengaruhi pecutan graviti, g. Mass does not affect the gravitational acceleration, g.
2
Definisi berat Definition of weight
Berat, W, bagi sesuatu objek ialah daya graviti yang dikenakan ke atasnya. The weight, W, of an object is the gravitational force acting on it.
Berat ialah daya dan diukur dalam unit Weight is a force and is measured in
Newton, N . Berat ialah kuantiti
Newton, N
vector
. Weight is a
vektor
.
quantity.
Sebiji batu yang berjisim m, dilepaskan jatuh bebas pada pecutan graviti, g. A stone of mass, m, is released and free falls with a gravitational acceleration of g.
Daya yang bertindak ke atas batu hanyalah berat, W, di mana ianya menuju ke arah bawah. The only force acting on the stone is its weight, W, which is downward.
Hukum gerakan Newton kedua:
Pecutan = g Acceleration = g
Newton’s second law of motion: F = ma di mana/where F = W , a = g
Jisim/Mass = m kg
Oleh itu/Therefore: W = mg W = berat/weight m = jisim/mass g = pecutan graviti/acceleration due to gravity unit bagi g ialah = m s-2
Berat/Weight = m × 10 N Bumi/Earth
the unit of g is = m s–2
© Nilam Publication Sdn. Bhd.
04 FIZ Form 4 Ch2 C 2F.indd 74
74
9/10/13 4:28 PM
MODUL • Fizik TINGKATAN 4
Berat berubah, jisim tetap
Weight changes but mass is fixed
ringan Di Bulan, berat kita lebih daripada di Bumi, ini kerana medan graviti di Bulan adalah lebih kecil . On the Moon, our weight is
less
than that on Earth because smaller the Moon’s gravitational field is than that of the Earth.
1 Nilai bagi pecutan graviti, gB, di bulan ialah 6 daripada nilai gE di bumi.
The value of the gravitational acceleration, gB, on the Moon is 1 the value of gE on the Earth. 6
Malahan di Bumi, berat kita berbeza sedikit dari suatu tempat ke tempat yang lain, kerana kekuatan medan graviti yang berbeza. Semakin jauh dari bumi, berat kita semakin berkurang . Even on Earth, our weight can vary slightly from place to place, because the Earth’s gravitational field strength varies. Moving away from the Earth, our weight decreases .
Di Di Di angkasa permukaan permukaan lepas Deep in Bulan Bumi
Jika kita boleh pergi lebih jauh ke dalam ruang angkasa lepas dan bebas daripada sebarang tarikan graviti, berat kita akan menjadi sifar . If we could go deep into space, and be free from any gravitational pull, our weight would be zero .
Jisim
Sama ada di atas Bumi, Bulan atau di angkasa, jisim kita tetap tidak akan berubah.
Berat
Mass
Whether on the Earth, Moon or deep in space, our mass does not change.
Weight
space
On Moon’s surface
On Earth’s surface
100 kg
100 kg
100 kg
0N
1 (1 000) N 6
1 000 N
U N I T
2
Perbezaan antara berat dan jisim
Difference between weight and mass
Berat/Weight
Jisim/Mass
Definisi
Daya graviti yang bertindak ke atas objek.
Jumlah jirim di dalam objek.
Definition
The force of gravity acting on the object.
The amount of matter in the object.
Perubahan / Tiada perubahan
Berat sesuatu objek berubah dengan kekuatan medan graviti pada sesuatu tempat.
Jisim sesuatu objek tidak berubah walau di mana-mana.
Change / unchanged
Kuantiti asas atau kuantiti terbitan
The weight of an object changes with the gravitational field strength at the location.
Kuantiti
terbitan
Base quantity or derived quantity
A
Kuantiti vektor atau kuantiti skalar
Kuantiti
Vector of scalar quantity
A
Unit SI/SI unit
Newton, N/Newton, N
derived
vector
Kuantiti
quantity
A
vektor
base
Kuantiti
quantity
A
scalar
unchanged
asas quantity
skalar quantity
Kilogram, kg/Kilogram, kg 75
04 FIZ Form 4 Ch2 C 2F.indd 75
The mass of an object is anywhere.
© Nilam Publication Sdn. Bhd.
9/10/13 4:28 PM
MODUL • Fizik TINGKATAN 4
Untuk objek yang jatuh dengan pecutan, g, berikut adalah persamaan-persamaan yang berkaitan: For an object falling with acceleration g, the following equations apply: 1 v = u + at di mana/where s = sesaran/displacement 2 s = ut + ½ at2 u = halaju awal/initial velocity 3 s = ½ (u + v)t v = halaju akhir/final velocity 4 v2 = u2 + 2as t = masa/time
titik tertinggi, v = 0 highest point, v = 0
a = g, pecutan graviti/acceleration due to gravity, Nota/Notes: 1 Apabila sebuah objek jatuh bebas: a = g = 10 m s-2 (pecutan) When an object fall freely: a = g = 10 m s-2 (acceleration)
2 Apabila sebuah objek dilambung ke atas: a = –g = –10 m s-2. (nyahpecutan) When an object is thrown upwards: a = –g = –10 m s-2. (deceleration)
3 Pada kedudukan yang tertinggi, v = 0. At the highest point, v = 0.
4 Jatuh ke bawah, v adalah positif./Downward direction, v is positive. 5 Arah ke atas, v adalah negatif./Upward direction, v is negative. U N I T
2
Objek dilambung ke atas Object thrown upwards
Contoh/Example Andaikan g = 10 m s-2 dan tiada rintangan udara. / Assume g = 10 m s-2 and there is no air resistance. 1 Sebiji batu jatuh daripada ketinggian 45 m.
2 Sebiji bola dilambung ke atas daripada tanah dengan halaju 30 m s–1. Selepas beberapa lamakah bola itu akan menyentuh tanah semula? (a) Berapa lamakah masa yang diambil oleh A ball is thrown upwards from the ground with a batu itu untuk mencecah ke tanah? A rock falls from a height of 45 m.
velocity of 30 m s–1. After how many seconds will it strike the ground again?
How long does it take to reach the ground?
(b) Berapakah halaju batu itu semasa ia menghentam lantai?
v=0
Tinggi/Height h
What is its velocity as it hits the ground?
Pecutan Acceleration g
g = –10 m s–2 Masa Time t
Penyelesaian/Solution (a) u = 0, s = 45 m, g = 10 m s–2, t = ? s = ut + ½ gt2 45 m = 0 + ½ (10 m s–2)(t2) t2 = 9 s2 t = 3 s (b) v = u + gt = 0 + (10 m s–2)(3 s) = 30 m s–1
u = 30 m s–1
Penyelesaian/Solution Untuk gerakan ke atas,/For the upward motion, u = 30 m s–1, v = 0, g = –10 m s–2, v = u + gt ∴0 = 30 m s–1 + (–10 m s–2)(t) 10t = 30 s t = 3 s (gerakan ke atas/upward motion) Maka, masa untuk gerakan ke bawah juga mengambil 3 s. Oleh itu, ia mengambil masa 6 saat. The time taken for the downward motion is also 3 s. So it takes a total of 6 seconds.
© Nilam Publication Sdn. Bhd.
04 FIZ Form 4 Ch2 C 2F.indd 76
76
9/10/13 4:28 PM
MODUL • Fizik TINGKATAN 4
Latihan/Exercises Andaikan nilai g = 10 m s–2./Assume the value of g = 10 m s–2. 1 Amir menjatuhkan batu ke dalam perigi. Jika jarak antara bahagian atas perigi dan permukaan air ialah 20 m, Amir releases a stone into a well. If the distance between the top of the well and the water surface is 20 m,
(a) berapakah masa yang diambil oleh batu itu untuk sampai ke permukaan air? what is the time required for the stone to reach the surface of the water?
(b) berapakah halaju batu itu apabila ia terkena permukaan air? what is the velocity of the stone when it strikes the surface of the water?
Penyelesaian/Solution (a) u = 0, s = 20 m , g = 10 m s–2 , t = ? s = ut + ½ gt2 20 m = 0 + ½ (10 m s–2)(t2) t2 = 4 s2 t = 2 s (b) v2 = u2 + 2gs v2 = 0 + 2(10 m s–2)(20 m) ∴v = 20 m s–1
Perhatian/Note: Untuk (b), mengapa v2 = u2 + 2gs digunakan dan bukan v = u + gt? For (b), why v2 = u2 + 2gs is used and not v = u + gt? Jawapan/Answer: Semua nilai yang diperlukan diberi dalam soalan. All the values required are given in the question.
2 Suatu objek yang berjisim 5 kg dilepaskan dari sebuah bangunan setinggi 500 m. Berapakah An object of mass 5 kg is released from a tall building of height 500 m. What is the
(a) berat objek itu?/weight of the object? (b) kekuatan medan graviti?/gravitational field strength? (c) masa yang diambil untuk sampai ke tanah?/time taken to reach the ground? Penyelesaian/Solution (a) W = 5 kg × 10 m s–2 (c) u = 0, s = 500 m, g = 10 m s–2 = 50 N s = ut + ½ gt2 –1 –2 (b) g = 10 N kg atau/or 10 m s 500 m = 0 + ½(10 m s–2)(t2) t2 = 100 s2 t = 10 s
2.9
U N I T
2
MENGANALISIS KESEIMBANGAN DAYA ANALYSING FORCES IN EQUILIBRIUM
Menerangkan situasi di mana daya berada dalam keseimbangan Describe situations where forces are in equilibrium
Keseimbangan daya/Forces in equilibrium pegun 1 Apabila suatu daya dikenakan terhadap objek dan ia mengekalkan keadaan atau bergerak dengan halaju seragam, maka objek itu dikatakan berada di dalam keadaan keseimbangan. When forces act upon an object and it remains is said to be in a state of equilibrium.
stationary
or moves at a constant
velocity , the object
paduan 2 Apabila keadaan keseimbangan berlaku, daya yang bertindak ke atas objek itu sifar adalah iaitu tiada daya bersih bertindak ke atasnya. When equilibrium is reached, the is no net force acting upon it.
resultant
force acting on the object is
77
04 FIZ Form 4 Ch2 C 2F.indd 77
zero
, i.e. there
© Nilam Publication Sdn. Bhd.
9/10/13 4:28 PM
MODUL • Fizik TINGKATAN 4
Menyatakan maksud daya paduan State what a resultant force is
Daya paduan: daya tunggal yang menunjukkan kesan daripada gabungan dua atau lebih daya dalam magnitud dan arah Resultant force: a single force that represents the combined effect of two or more forces in magnitude and direction
Daya paduan ialah hasil tambah vektor bagi dua atau lebih daya yang bertindak ke atas objek. Dalam keskes berikut, jika F ialah daya paduan, maka, The resultant force is the vector sum of two or more forces which act on the object. In the cases below, if F is the resultant force, hence, F1
F2
F2
Daya paduan/Resultant force = F = F1 + F2
U N I T
2
F1
Daya paduan/Resultant force = F = F1 – F2
Latihan/Exercises 1 Hitungkan daya paduan. Ke arah manakah objek itu bergerak?
2 Hitungkan daya paduan. Ke arah manakah objek itu bergerak?
Calculate the resultant force. Which direction does the object move?
Calculate the resultant force. Which direction does the object move?
5N
5N
12 N
Daya paduan, F/Resultant force, F = 5 N + 12 N = 17 N Arah ke kanan/To the right. 3 Seekor kuda menarik kereta dengan daya 500 N. Seorang petani membantu kuda itu menolak kereta itu dengan daya 200 N. Berapakah daya paduan?
12 N
Daya paduan, F/Resultant force, F = 12 N – 5 N = 7 N Arah ke kanan/To the right. 200 N
500 N
A horse pulls a cart with a force of 500 N. A farmer helps the horse by pushing the cart with a force of 200 N. What is the resultant force? Daya paduan, F/Resultant force, F = 500 N + 200 N = 700 N ke kanan/to the right
4 Seekor kuda menarik kereta dengan daya 500 N. Seorang petani menarik kereta itu pada arah bertentangan dengan daya 200 N. Berapakah daya paduan?
200 N
500 N
A horse pulls a cart with 500 N force. A farmer pulls the cart with a force of 200 N but in opposite direction. What is the resultant force? Daya paduan, F/Resultant force, F
= 500 N – 200 N = 300 N ke kanan/to the right
© Nilam Publication Sdn. Bhd.
04 FIZ Form 4 Ch2 C 2F.indd 78
78
9/10/13 4:28 PM
MODUL • Fizik TINGKATAN 4
5 Rajah 5.1 menunjukkan sebiji ladung berjisim 0.3 kg digantung dari siling. Diagram 5.1 shows a pendulum bob of mass 0.3 kg hung from the ceiling.
Siling Ceiling
Benang itu ditarik secara mengufuk dengan daya, F, supaya sudut antara benang dengan garis mencancang adalah 40°seperti ditunjukkan dalam Rajah 5.2. The thread is then pulled horizontally by a force, F, so that the thread makes an angle of 40° with the vertical line as shown in Diagram 5.2.
40°
Benang Thread Ladung Pendulum Bob
Rajah 5.1/Diagram 5.1
(a) Dalam ruang di bawah, lukis sebuah segi tiga keseimbangan daya bagi T, F dan berat ladung itu.
T
In the space below, draw a triangle of forces in equilibrium for T, F and the weight of the bob. 40°
F
Berat Weight 3.0 N
T m = 0.3 kg
Perhatian/Note: Arah bagi tiga daya itu adalah berkitar. The directions of the three forces are cyclic.
Berat Weight
F
Rajah 5.2/Diagram 5.2
U N I T
2
(b) Hitung Daya, F Calculate the force, F
Dari segi tiga di atas,/From the triangle above, F = tan 40° 3.0 N F = 3.0 N tan 40° = 2.52 N
(c) Hitung tegangan, T, dalam benang itu Calculate the tension, T, in the string. Dari segi tiga di atas,/From the triangle above,
3.0 N = kos 40° T T = 3.0 N kos 40° = 3.92 N
Perhatian/Note: Soalan 5 di atas boleh dijawab dengan kaedah lukisan berskala.
Question 5 above can be answered by scale-drawing.
79
04 FIZ Form 4 Ch2 C 2F.indd 79
© Nilam Publication Sdn. Bhd.
9/10/13 4:28 PM
MODUL • Fizik TINGKATAN 4
Dua daya yang betindak pada sudut tertentu antara satu sama lain Two forces acting at an angle to each other
Daya paduan bagi dua daya yang bertindak ke atas dua objek pada dua arah berbeza, boleh ditentukan dengan menggunakan kaedah segi tiga daya atau kaedah segi empat selari daya. The resultant force of two forces, which act on an object in two different directions, can be determined by a triangle parallelogram of forces or a of forces.
Dua daya yang bertindak pada satu titik melalui satu sudut [kaedah segi empat selari]
F1
Two forces acting at a point at an angle [Parallelogram method]
Skala/Scale: 1 cm = U N I T
2
60 °C
N
LANGKAH 1/ STEP 1: Dengan menggunakan pembaris dan protaktor, lukis dua daya F1 dan F2 bermula dari satu titik, X.
F2
LANGKAH 2/ STEP 2: Lengkapkan rajah segi empat selari. Complete the parallelogram.
Using ruler and protractor, draw the two forces F1 and F2 from a point, X.
Draw the diagonal (from X) of the parallelogram. The diagonal represents the resultant force, F in magnitude and direction.
F1 F1 X
X
60 °C
LANGKAH 3/ STEP 3: Lukis pepenjuru (dari X) bagi segi empat selari bagi menunjukkan magnitud dan arah bagi daya paduan, F.
F1
F2
F2
Daya paduan
X
Resultant force F2
Contoh/Example Rajah di bawah menunjukkan dua daya yang bertindak ke atas objek P. The diagram below shows two forces acting on object P. 5N
F1
60° P
12 N
F2
Tentukan daya paduan yang terhasil. Determine the resultant force produced.
© Nilam Publication Sdn. Bhd.
04 FIZ Form 4 Ch2 C 2F.indd 80
80
9/10/13 4:28 PM
MODUL • Fizik TINGKATAN 4
Jawapan/Answer: Kaedah I/Method I Kaedah segi empat selari daya Parrallelogram of forces method
1 Tentukan skala. Dengan menggunakan pembaris dan protaktor, lukis dua daya F1 dan F2 bermula dari titik O. Set a scale. Using a ruler and protactor, draw the two forces, F1 and F2 from a point O.
2 Lengkapkan rajah segi empat selari./Complete the parallelogram. 3 Lukis pepenjuru bagi segi empat selari. Pepenjuru menunjukkan paduan, F.
magnitud
dan
Draw the diagonal of the parallelogram. The diagonal represents the resultant force, F in direction .
arah
daya
magnitude
and
U N I T
5 cm = 5 N
2 60° O 12 cm = 12 N
Kaedah II/Method II Kaedah segi tiga daya Triangle of forces method 1 Tentukan skala. Dengan menggunakan pembaris dan protaktor, lukis daya yang pertama, F1 dari titik O. Set a scale. Using a ruler and protractor, draw the first force, F1, from a point O.
2 Lukis daya yang kedua F2 dari hujung atas F1. Draw the second force, F2 from the head of F1.
3 Lengkapkan segi tiga daya dengan melukis garisan dari pangkal F1 ke hujung F2. Complete a triangle of forces by drawing a line from the tail of F1 to the head of F2.
4 Pepenjuru menunjukkan magnitud dan arah daya paduan, F. The diagonal represents the resultant force, F, in magnitude and direction. F2 = 12 N
F1 = 5 N F = Daya paduan Resultant force 60° O
81
04 FIZ Form 4 Ch2 C 2F.indd 81
© Nilam Publication Sdn. Bhd.
9/10/13 4:28 PM
MODUL • Fizik TINGKATAN 4
Latihan/Exercises 1 Dengan menggunakan skala dan kaedah yang sesuai, tentukan daya paduan. (Perhatian: magnitud dan arah diperlukan). By using suitable scale and method, determine the resultant force (Note: Magnitudde and direction are required).
(a)
(b)
(c)
8N
6N
5N
600
1200 10 N
8N
5N
2 Lengkapkan rajah untuk menunjukkan daya paduan. Complete the diagram to show the resultant force. U N I T
2N 120°
2
5N
Jawapan/Answers: 1 cm : 1 N
1 Skala/Scale: (a)
0.0
1 n= ua rce d pa fo ya ant Da sult Re
6N
N
37° 8N
10 N pada sudut 37° dengan daya 8 N 10 N at angle of 37° with the 8 N-force
© Nilam Publication Sdn. Bhd.
04 FIZ Form 4 Ch2 C 2F.indd 82
82
9/10/13 4:28 PM
MODUL • Fizik TINGKATAN 4
(b)
5.6
=1 an aduforce p a t Day ultan Res
8N
600
N
260 10 N
U N I T
15.6 N pada sudut 26° dengan daya 10 N 15.6 N at an angle of 26° with the 10 N-force
2
Da Res ya pad ulta uan nt f orce = 5.0 N
(c)
5N
600 600 5N
5 N pada sudut 60° dengan daya 5 N 5 N at an angle of 60° with 5 N-force
2 Skala/Scale:
2 cm : 1 N
(a) Daya
padu
an/Re
sulta
nt for ce = 6
.2 N
2N 120°
16° 5N
6.2 N pada sudut 16° dengan daya 5 N 6.2 N at an angle of 16° with 5 N-force
83
04 FIZ Form 4 Ch2 C 2F.indd 83
© Nilam Publication Sdn. Bhd.
9/10/13 4:28 PM
MODUL • Fizik TINGKATAN 4
Daya dileraikan kepada komponen berkesan Resolve a force into the effective components
Leraian Daya/Resolution Of Forces: berserenjang/bersudut tegak
Daya F boleh dileraikan kepada dua komponen yang sama lain:
perpendicular/at 90° A force F can be resolved into two components which are (a) F : komponen mengufuk / horizontal component,
to each other:
x
U N I T
2
(b) Fy: komponen
menegak
F sin θ = y F Fy = F sin θ
F kos θ = x F Fx = F kos θ
/ vertical
antara satu
θ
component, Fy
F
θ Fx
Latihan/Exercises 1 Dapatkan komponen mengufuk dan komponen menegak daya tersebut. (Perhatian: Pertimbangkan magnitud daya yang positif sahaja.) Find the horizontal component and the vertical component of the force. (Note: Consider only the positive magnitudes of the forces.)
(a)
(b)
Fy
Fx
200 N
700
700
500 75 N 500
Fy
Fx
Fx = kos 50° 200 N ∴ Fx = 200 N kos 50° = 128.6 N
FY = sin 50° 200 N ∴ FY = 200 N sin 50° = 153.2 N
(c)
Fx = sin 70° 75 N ∴ Fx = 75 N sin 70° = 70.48 N (d)
Fy
400
0
600 F=6N
F=5N
Fx = sin 40° 5N ∴ Fx = 5 N sin 40° = 3.21 N
© Nilam Publication Sdn. Bhd.
04 FIZ Form 4 Ch2 C 2F.indd 84
Fx
600
400 Fx
FY = kos 70° 75 N ∴ FY = 75 N kos 70° = 25.65 N
0
Fy
FY = kos 40° 5N ∴ FY = 5 N kos 40° = 3.83 N
Fx = kos 60° 6N ∴ Fx = 6 N kos 60° = 3.0 N
FY = sin 60° 6N ∴ FY = 6 N sin 60° = 5.20 N
84
9/10/13 4:28 PM
MODUL • Fizik TINGKATAN 4
2 Rajah menunjukkan troli yang berjisim 2 kg di atas permukaan licin, ditarik oleh daya 6 N pada sudut 60° dengan ufuk.
6N
The diagram shows a trolley of mass 2 kg on a smooth surface being pulled by a force of 6 N at an angle of 60° with the horizontal.
600
2 kg
(a) Berapakah komponen mengufuk daya itu?
Fx
Permukaan licin/Smooth surface
What is the horizontal component of the force? (b) Berapakah pecutan troli itu?/What is the acceleration of the trolley?
Penyelesaian/Solution Fx (a) = kos 60° 6N ∴ Fx = 6 N kos 60° = 3.0 N
(b) F = ma, a =
3N = 1.5 m s-2 2 kg
600
3 Rajah menunjukkan sebuah kereta sedang ditunda. Kabel itu mempunyai daya F, 5 000 N.
The diagram shows a car being towed. The cable has a force F of 5 000 N. Tunjukkan dan labelkan/Indicate and label: • daya F/the force F • daya komponen mengufuk, Fx /the horizontal component force Fx • daya komponen menegak, Fy /the vertical component force Fy Cari/Find
(a) (b)
U N I T
2
• daya mengufuk kabel yang menggerakkan kereta ke hadapan. the horizontal force of the cable which moves the car forward. • daya menegak kabel./the vertical force of the cable.
Penyelesaian/Solution (a)
Fx = 5 000 N F = 5 000 N ∴ Fx = Fy = 60° FY Fx = 5 000 N ∴ FY = =
(b)
kos 60° 5 000 N kos 60° 2 500 N sin 60° 5 000 N sin 60° 4 330 N
4 Seorang pelancong menarik begnya dengan daya 100 N pada sudut 55° dari garis mengufuk.
A tourist pulls his bag with a force of 100 N at an angle of 55° with the horizontal. Tunjukkan dan labelkan/Indicate and label: • daya F/the force F • daya komponen mengufuk, Fx /the horizontal component force Fx • daya komponen menegak, Fy /the vertical component force Fy Cari/Find
(a) (b)
• daya mengufuk beg yang menggerakkannya ke hadapan. the horizontal force of the cable which moves it forward. • daya menegak beg./the vertical force of the bag.
Penyelesaian/Solution (a)
Fx = 100 N F = 100 N ∴ F = Fy = 55° FY = Fx 100 N ∴ FY =
(b)
85
04 FIZ Form 4 Ch2 C 2F.indd 85
kos 55° 100 N kos 55° 57.36 N sin 55° 100 N sin 55° = 81.92 N © Nilam Publication Sdn. Bhd.
9/10/13 4:28 PM
MODUL • Fizik TINGKATAN 4
Masalah yang melibatkan daya paduan dan prinsip leraian daya
Problems involving resultant force and the principle of resolution of forces
A Lif/Lift Seorang budak lelaki berada di dalam sebuah lif. Dia berdiri di atas mesin penimbang. Berat budak lelaki, W, bertindak arah ke bawah budak itu dan tindak balas normal, R, bertindak arah ke atas. A boy is inside a lift. He is standing on a weighing machine. The weight of the boy, W acts downward on the boy and a normal reaction, R, acts upwards.
Lif yang pegun (atau bergerak ke bawah atau ke atas dengan halaju seragam) Stationary lift (or moves upwards or downwards with uniform velocity)
Lif bergerak ke atas dengan pecutan a m s-2
Lif bergerak ke bawah dengan pecutan a m s-2
The lift moves upwards with an acceleration of a m s-2
The lift moves downwards with an acceleration of a m s-2
R
U N I T
R
Mesin penimbang Weighing machine
Mesin penimbang Weighing machine
2
R
Mesin penimbang Weighing machine
a
a
W = mg
W = mg
W = mg
Daya paduan, F = 0
Daya paduan, F arah ke atas
Daya paduan, F arah ke bawah
Resultant force, F = 0
Resultant force, F is upwards
Resultant force, F is downward
F = R – mg = 0 R = mg
R F F R
mg > R F = ma F = mg – R = ma R = mg – ma
Bacaan pada mesin penimbang = berat budak lelaki tersebut
Bacaan pada mesin penimbang lebih besar daripada berat budak itu
The reading on the weighing scale = the weight of the boy
> = = =
mg ma R – mg = ma mg + ma
The reading on the weighing scale machine is larger than the weight of the boy
Bacaan mesin penimbang lebih kecil daripada berat budak itu The reading on the weighing scale machine is smaller than the weight of the boy
Perhatian : Dalam setiap kes di atas, R memberi bacaan pada mesin penimbang. Note
: In each of the cases above, R gives the reading on the weighing scale.
© Nilam Publication Sdn. Bhd.
04 FIZ Form 4 Ch2 C 2F.indd 86
86
9/10/13 4:28 PM
MODUL • Fizik TINGKATAN 4
Seorang budak lelaki berjisim 50 kg berada di dalam sebuah lif. A boy of mass 50 kg is inside a lift.
(a) Kirakan berat budak lelaki itu,/Calculate the weight of the boy, (b) Kirakan bacaan yang ditunjukkan oleh mesin penimbang jika lif itu:
Calculate the reading on the weighing scale if the lift:
(i) pegun
Lif Lift
m = 50 kg
is stationary
(ii) memecut ke atas dengan pecutan 2 m s-2
accelerates upwards with an acceleration of 2 m s-2
(iii) memecut ke bawah dengan pecutan 2 m s-2
accelerates downwards with an acceleration of 2 m s-2
(iv) bergerak ke atas dengan halaju seragam 1.5 m s-1
moves upwards with constant velocity of 1.5 m s-1
Mesin penimbang Weighing scale
Penyelesaian/Solution
(iii) mg – R = ma R = 500 N – (50 kg)(2 m s–2) = 400 N (iv) R – mg = ma Tetapi a = 0 (kerana halaju malar)
(a) Berat budak = W Mass of the boy = mg = 50 kg × 10 m s–2 = 500 N (b) (i) R = W = 500 N (ii) R – mg = ma R = 500 N + (50 kg)(2 m s–2) = 600 N
but a = 0 (because constant velocity)
∴ R = mg = 500 N
U N I T
2
B Takal/Pulley 1 Takal licin Smooth pulley T
Daya geseran, 2 N Frictional force, 2 N 4 kg
T
T
T
4 kg
3 kg
3 kg Berat/Weight, 40 N
Cari daya paduan, F
Berat/Weight, 30 N
Takal licin Smooth pulley
Berat/Weight, 30 N
F = 40 N – 30 N = 10 N
F = 30 N – 2 N = 28 N
m = 4 kg + 3 kg = 7 kg
m = 4 kg + 3 kg = 7 kg
F = ma a = 10 N = 1.43 m s-2 7 kg
F = ma, ∴28 N = (7 kg)(a) a = 28 N = 4 m s-2 7 kg
Find the resultant force, F
Cari jisim yang bergerak, m Find the moving mass, m
Cari pecutan, a Find the acceleration, a,
87
04 FIZ Form 4 Ch2 C 2F.indd 87
© Nilam Publication Sdn. Bhd.
9/10/13 4:28 PM
MODUL • Fizik TINGKATAN 4
Cari tegangan benang, T Find the string tension, T
Kaedah I/Method I Pertimbangkan jisim 4 kg sahaja (gerak ke bawah)
Kaedah I/Method I Pertimbangkan jisim 3 kg sahaja (yang bergerak ke bawah)
Consider only the 4 kg-mass (moving downwards)
Consider only the 3 kg-mass (moving downwards)
40 N – T = m1 a 40 N – T = (4 kg)(1.43 m s–2) ∴ T = 34.28 N Kaedah II/Method II Pertimbangkan jisim 3 kg sahaja (gerak ke atas)
30 N – T = ma 30 N – T = (3 kg)(4 m s–2) ∴ T = 30 N – 12 N = 18 N Kaedah II/Method II Pertimbangkan jisim 4 kg sahaja (yang bergerak ke kanan)
Consider only the 3 kg-mass (moving upwards)
– 30 N = m2 a T T = 30 N + (3 kg)(1.43 m s–2) T = 34.29 N U N I T
2
Consider only the 4 kg-mass (moving to the right)
T – 2 N = ∴ T = = =
2 Troli yang berjisim 2 kg disambungkan dengan tali kepada pemberat berjisim 3 kg. Tali diletakkan di atas takal licin. Pemberat kemudiannya dilepaskan. A 2 kg-trolley is connected by a rope to a 3 kg-load. The rope passes over a smooth trolley. The load is then released.
(a) (i) Jika permukaan meja adalah licin, berapakah pecutan troli?
2 kg
Meja Table
If the surface of the table is smooth, what is the acceleration of the trolley? (ii) Berapakah tegangan dalam tali?/What is the tension in the rope?
ma 2 N + (4 kg)(4 m s–2) 2 N + 16 N 18 N
T
Takal licin Smooth trolley T 3 kg 30 N
(b) (i) Jika daya geseran antara troli dan permukaan meja ialah 10 N, berapakah pecutan troli? If the frictional force between the trolley and the surface of the table is 10 N, what is the acceleration of the trolley?
(ii) Berapakah tegangan tali sekarang? What is the tension in the rope now?
Penyelesaian/Solution (b) (i) 30 N – 10 N = (3 + 2) kg × a 20 N = 5 kg × a a = 4 m s-2 (ii) dari gerakan troli from the motion of the trolley T – 10 N = ma T – 10 N = (2 kg)(4 m s-2) T = 8 N + 10 N = 18 N atau dari gerakan jisim 3 kg or from the motion of the 3 kg-mass 30 N – T = m1 a 30 N – T = (3 kg)(4 m s-2) T = 30 N – 12 N = 18 N
(a) (i) F = ma 30 N = [(2 + 3) kg] [a] a = 6 m s–2 (ii) dari gerakan troli from the motion of the trolley T = ma = (2 kg)(6 m s–2) = 12 N atau dari gerakan jisim 3 kg or from the motion of the 3 kg-mass 30 N – T = ma 30 N – T = (3 kg)(6 m s–2) T = 30 N – 18 N = 12 N
© Nilam Publication Sdn. Bhd.
04 FIZ Form 4 Ch2 C 2F.indd 88
88
9/10/13 4:28 PM
MODUL • Fizik TINGKATAN 4
C Satah condong/Inclined plane N
Sebuah bongkah kayu berjisim m kg diletakkan di atas satah condong yang membuat sudut θ dengan ufuk. Blok kayu dikenakan dengan daya-daya seperti yang disenaraikan di bawah: A block of wood of m kg which is placed on an inclined plane makes an angle θ with the horizontal. The block of wood is acted upon by the forces as listed below:
(a) Wx, komponen berat yang selari dengan satah condong.
Wx
θ
Wy
mg
θ
Wx, the weight component which is parallel to the inclined plane.
(b) Wy, komponen berat yang berserenjang dengan satah condong.
Wy, the weight component which is perpendicular to the inclined plane.
(c) tindak balas normal, N.
the normal reaction, N.
Ungkapkan setiap komponen di (a), (b), (c) dalam sebutan m, g dan θ, di mana g ialah pecutan akibat graviti. Express each of the components in (a), (b), and (c) in terms of m, g and θ, where g is acceleration due to gravity.
Penyelesaian/Solution (a) Wx = mg sin θ (b) Wy = mg kos θ (c) N = Wy = mg kos θ
U N I T
2 2.10
MEMAHAMI KERJA, TENAGA, KUASA DAN KECEKAPAN UNDERSTANDING WORK, ENERGY, POWER AND EFFICIENCY
Kerja dilakukan apabila daya membuatkan suatu objek bergerak. Semakin besar daya dan objek itu digerakkan, maka semakin besar kerja dilakukan.
jarak
Work is done whenever a force makes something move. The greater the force and the greater the moved, the more work is done.
distance
sesuatu
Definisi kerja/Definition of work Kerja yang dilakukan ialah hasil darab daya yang dikenakan dengan sesaran objek pada arah daya dikenakan. Work done is the product of an applied force and the displacement of the object in the direction of the applied force.
Kerja, W = Fs , di mana F = daya, s = sesaran Work, W = Fs , where
F = force, s = displacement
Unit SI bagi kerja ialah
joule, J
The SI unit of work is the
joule, J
F
s
Kerja 1 joule dilakukan apabila daya 1 N menggerakkan objek sejauh 1 m dalam arah daya dikenakan. 1 joule of work is done when a force of 1 N moves an object 1 m in the direction of the force.
89
05 FIZ Form 4 Ch2 D 2F.indd 89
© Nilam Publication Sdn. Bhd.
9/10/13 4:14 PM
MODUL • Fizik TINGKATAN 4
Pengiraan kerja
Calculation of work
Sesaran, s, bagi objek dalam arah sama dengan daya, F The displacement, s, of the object is in the direction of the force, F
Sesaran, s, bagi objek pada arah yang tidak sama dengan arah daya, F The displacement, s, of the object is not in the direction of the force, F
F
θ F1 s
s
F
F1 = kos θ F F1 = F kos θ ∴ W = F1 × s = (F kos θ) s
F
s
U N I T
2
W = Fs Contoh/Example
W = Fs
1
Contoh/Example
2
Seorang budak menolak basikalnya dengan daya 25 N melalui jarak 3 m. Kira kerja yang dilakukan oleh budak itu.
Seorang budak perempuan mengangkat sebuah pasu berjisim 3 kg pada ketinggian 0.4 m. Berapakah kerja yang dilakukan oleh budak perempuan itu?
A boy is pushing his bicycle with a force of 25 N through a distance of 3 m. Calculate the work done by the boy.
A girl lifts up a 3 kg flower pot steadily to a height of 0.4 m. What is the work done by the girl?
25 N
Penyelesaian/Solution W = 3 kg × 10 m s–2 × 0.4 m = 12 J Atau/Or W = 3 kg × 10 N kg–1 × 0.4 m = 12 J
Penyelesaian/Solution W = Fs = 25 N × 3 m = 75 J (Perhatian/Note: 1 N m = 1 J) Contoh/Example
30 N
3
Seorang pekebun menolak mesin rumput dengan daya 50 N pada sudut 60 °C dengan ufuk. Berapakah kerja yang dilakukan untuk menolak mesin rumput pada jarak 100 m? A gardener pushes a lawn mower with a force of 50 N at an angle of 60 °C from horizontal. What is the work done in pushing the lawn mower through a distance of 100 m?
Penyelesaian/Solution F = kos 60° 50 N F = 50 N kos 60° ∴ W = F × 100 m = (50 N kos 60°) × (100 m) = 2 500 N © Nilam Publication Sdn. Bhd.
05 FIZ Form 4 Ch2 D 2F.indd 90
50 N
F 60°
600
50 N
60°
90
9/10/13 4:14 PM
MODUL • Fizik TINGKATAN 4
Tiada kerja dilakukan apabila:/No work is done when:
Objek berada dalam keadaan pegun . stationary
The object is
.
Arah gerakan objek adalah berserenjang dengan daya yang dikenakan. The direction of motion of the object is force.
perpendicular
to that of the applied
Seorang pelayan membawa dulang makanan dan berjalan. A waiter is carrying a tray of food and walking.
Latihan/Exercises 1 Berapalah kerja dilakukan oleh daya 80 N itu?
U N I T
F = 80 N
How much work is done by the 80 N force?
2
Penyelesaian/Solution W = 80 N × 0.5 m = 40 J
Sesaran/Displacement = 50 cm
2 Ali menolak sebiji batu dengan mengenakan daya 200 N. Berapakah kerja yang dilakukannya?
Pegun/Stationary
Ali pushes a big rock by applying a force of 200 N. How much work has he done?
F = 200 N
Penyelesaian/Solution W = sifar (pegun)/zero (stationary) 3 Seorang lelaki menarik satu beban dengan menggunakan satah condong. Tinggi landasan condong itu ialah 80 cm. Berapakah kerja yang dilakukan oleh lelaki itu untuk menarik beban itu? A man pulls up a load using an inclined plane. The height of the inclined plane is 80 cm. How much work is being done by the man to lift the load?
F = 320 N
80 cm 30°
Penyelesaian/Solution s 30°
80 cm
80 cm = sin 30° s 80 cm ∴s = sin 30° 0.8 m = sin 30°
W = Fs
= 320 N × (
= 512.0 J
0.8 m ) sin 30°
91
05 FIZ Form 4 Ch2 D 2F.indd 91
© Nilam Publication Sdn. Bhd.
9/10/13 4:14 PM
MODUL • Fizik TINGKATAN 4
4 Berapakah kerja yang dilakukan oleh Raju untuk mengangkat beban melalui jarak 4 m?
F = 150 N
How much work is done by Raju to lift the load through the displacement of 4 m?
Penyelesaian/Solution W = 150 N × 4 m = 600 J
F
Sesaran Displacement =4m
5 Satu daya 25 N digunakan untuk mengangkat sebuah beg. Encik Rahim berjalan sejauh 20 m dengan memegang beg itu. Berapakah kerja yang dilakukan oleh Encik Rahim terhadap beg itu? Terangkan jawapan anda. A force of 25 N is used to lift a bag. Encik Rahim walks a distance of F = 25 N 20 m holding the bag. How much work is being done by Encik Rahim with respect to the bag? Explain your answer.
Penyelesaian/Solution Kerja dilakukan adalah sifar kerana daya adalah berserenjang dengan sesaran.
U N I T
Work done is zero because the force is perpendicular to the displacement.
2
Sesaran/Displacement = 20 m
Menyatakan bahawa apabila kerja dilakukan, tenaga dipindahkan dari satu objek ke satu objek yang lain State that when work is done, energy is transferred from one object to another
• Tenaga boleh ditakrifkan sebagai kebolehan untuk melakukan kerja. Energy can be defined as the ability to do work.
• Satu objek yang boleh melakukan kerja dikatakan mempunyai tenaga./An object that can do work has energy. • Tenaga boleh wujud dalam pelbagai bentuk, misalnya/Energy exists in many forms, for example: (a) Tenaga keupayaan graviti/Gravitational potential energy (b) Tenaga kinetik/Kinetic energy (c) Tenaga keupayaan kenyal/Elastic potential energy (d) Tenaga keupayaan elektrik/Electric potential energy (e) Tenaga bunyi/Sound energy (f) Tenaga mekanik/Mechanical energy (g) Tenaga nuklear/Nuclear energy • Kerja dilakukan apabila daya dikenakan ke atas objek dan objek itu bergerak. Ini diikuti dengan pemindahan tenaga dari satu objek ke objek lain. Work is done when a force is applied on an object and the object moves. This is followed by the transference of energy from one object to another.
• Oleh itu, apabila kerja dilakukan, Therefore, when work is done,
© Nilam Publication Sdn. Bhd.
05 FIZ Form 4 Ch2 D 2F.indd 92
tenaga
energy
dipindahkan dari satu objek ke satu objek yang lain.
is transferred from one object to another.
92
9/10/13 4:14 PM
MODUL • Fizik TINGKATAN 4
Definisi tenaga keupayaan graviti/Define gravitational potential energy Seorang budak perempuan membuat kerja apabila dia memanjat tangga sebuah papan gelongsor. Dia mempunyai tenaga keupayaan gravity apabila dia berada pada kedudukan tertinggi papan gelongsor itu. A girl does work when she climbs up the stairs of a sliding board. She has gravitational potential energy when she is at the top of the sliding board.
Tenaga keupayaan gravity ialah tenaga yang tersimpan dalam objek ketinggian (kedudukan) nya daripada permukaan disebabkan bumi. The gravitational potential energy is the energy stored in the object because height (position) above the earth’s surface. of its
Tenaga keupayaan graviti adalah sama dengan kerja dilakukan untuk menaikkan satu objek kepada satu ketinggian tertentu. Daya diperlukan untuk menaikkan objek adalah sama dengan berat objek, F = mg. work done The gravitational potential energy is equal to the to raise an object to a particular height. The force required to raise the object is the same weight as the of the object, F = mg.
Bola Ball Jisim/Mass = m kg
Kedudukan akhir Final position h
Kedudukan awal Initial position
Jika jarak menegak yang dilalui oleh objek ialah h,/If the distance moved by the object is h, Tenaga keupayaan graviti, Work done, W = F × s Kerja dilakukan, W = F × s Gravitational potential Energy, = mg × h = mg × h = mgh EP = W = mgh
U N I T
2
Contoh/Example Kirakan tenaga keupayaan graviti bagi setiap keadaan berikut. Calculate the gravitational potential energy for each of the following.
(a)
(b)
(c)
Kotak Box m = 0.3 kg
Berat Weight m = 0.3 kg
m = 2 kg
h = 1.2 m
h = 20 cm
h = 0.8 m Pendulum Bob
Penyelesaian/Solution E = mgh E = mgh E = mgh = (2 kg) (10 m s–2) (0.8 m) = (0.3 kg) (10 m s–2) (0.2 m) = 0.3 kg × 10 m s–2 × 1.2 m = 16 J = 0.6 J = 3.6 J
93
05 FIZ Form 4 Ch2 D 2F.indd 93
© Nilam Publication Sdn. Bhd.
9/10/13 4:14 PM
MODUL • Fizik TINGKATAN 4
Nota/Notes: 1 Tenaga keupayaan graviti bagi objek bergantung pada The gravitational potential energy of an object depends on the
A
A
A
hm
(a) jisim objek, m mass of the object, m
B
B
B
(b) kekuatan medan graviti, g gravitational field strength, g
(c) perubahan ketinggian, h change in height, h 2 "Kehilangan" tenaga keupayaan graviti tidak bergantung pada kecerunan tetapi bergantung pada jarak tegak . U N I T
2
ia bergerak. The "loss" of gravitational potential energy does not depend on the gradient of the slope but depends on the vertical distance traversed. Tenaga keupayaan graviti 100 J Gravitational potential energy 100 J Kerja dilakukan sepanjang landasan Work done along this path = 100 J
Kerja dilakukan Work done in this path = mgh = 100 J
Definisi tenaga kinetik Definition of kinetic energy
Seorang budak lelaki yang sedang mengayuh basikal mempunyai tenaga kinetik. Apabila dia mengayuh lebih pantas, maka dia mempunyai tenaga kinetik yang lebih besar. Pada keadaan pegun, dia tidak mempunyai tenaga kinetik. A boy riding a bicycle possesses kinetic energy. When he rides faster, he will have more kinetic energy. When he is stationary, he does not have any kinetic energy.
Tenaga kinetik ialah tenaga yang diperoleh sesuatu objek disebabkan oleh gerakannya. Kinetic energy is the energy of an object due to its motion.
Tenaga kinetik = 1 mv2 2 di mana m = jisim v = halaju
© Nilam Publication Sdn. Bhd.
05 FIZ Form 4 Ch2 D 2F.indd 94
1 mv2 2 where m = mass v = velocity
Kinetic energy =
94
9/10/13 4:14 PM
MODUL • Fizik TINGKATAN 4
Contoh/Example 1 Sebiji bola berjisim 0.5 kg bergerak dengan halaju 4 m s-1. Hitungkan kerja dilakukan. A ball of mass 0.5 kg moves with velocity of 4 m s-1. Calculate the work done.
Penyelesaian/Solution Kerja = Tenaga kinetik = 1 mv2 2 = 1 (0.5 kg) (4 m s–1)2 2 = 4.0 J
Work done = Kinetic energy 1 mv2 = 2 1 (0.5 kg) (4 m s–1)2 = 2 = 4.0 J
2 Sebuah kereta berjisim 950 kg bergerak dengan halaju malar 20 m s-1 selama satu minit. Hitungkan kerja yang dilakukan oleh kereta itu dalam tempoh yang tersebut.
A car of mass 950 kg moves at a constant velocity of 20 m s-1 for one minute. Calculate the work done by the car during this period. m = 950 kg
U N I T
–1
v = 20 m s
2
Penyelesaian/Solution Kerja dilakukan = Tenaga kinetik Work done
= Kinetic energy
= 1 mv2 2 = 1 (950 kg) (20 m s–1)2 2 = 190 000 J
Menyatakan prinsip keabadian tenaga/State the principle of conservation of energy berubah Prinsip keabadian tenaga menyatakan bahawa tenaga boleh dicipta bentuk yang lain, tetapi tidak boleh atau dimusnahkan. The principle of conservation of energy states that energy can be form, but it cannot be created or destroyed.
Jumlah tenaga di dalam sistem
diabadikan
dari satu bentuk ke satu
changed
from one form to another
/ The total energy in a system is
conserved
Jumlah tenaga sebelum berubah kepada bentuk tenaga yang lain = Jumlah tenaga berubah kepada bentuk tenaga yang lain. Total energy of energy.
before
conversion to other forms of energy = Total energy
95
05 FIZ Form 4 Ch2 D 2F.indd 95
after
.
selepas
conversion to other forms
© Nilam Publication Sdn. Bhd.
9/10/13 4:14 PM
MODUL • Fizik TINGKATAN 4
Aktiviti: Keabadian Tenaga/Activity: Conservation of Energy 1 (a) Sebiji bola dipegang pada ketinggian tertentu di atas lantai. Hold a ball at a certain height above the floor.
Apakah jenis tenaga yang terdapat pada bola? What is the energy gained by the ball? Tenaga keupayaan graviti/Gravitational potential energy
(b) Bola dilepaskan. Release the ball. (i) Apakah jenis tenaga yang dipunyai oleh bola sejurus sebelum ia menghentam lantai? What is the energy gained by the ball just before it hits the floor? Tenaga kinetik/Kinetic energy
U N I T
Ketinggian Height
(ii) Dari manakah tenaga pada bola itu berasal? Where does the energy of the ball originate? Tenaga keupayaan graviti/Gravitational potential energy
Bola Ball
Lantai/Floor
(iii) Apakah hubungan antara tenaga di (a) dengan tenaga di (b)? What is the relationship between the energy in (a) and the energy in (b)? Sama/Equal
2
2 (a) Troli ditolak ke arah dinding untuk memampatkan spring. Push a trolley against a wall to compress a spring.
Apakah tenaga yang tersimpan di dalam spring? What is the energy stored in the spring?
Tenaga keupayaan kenyal Elastic potential energy
(b) Troli dilepaskan supaya ia bergerak menjauhi dinding. Release the trolley so that it moves away from the wall.
Apakah yang berlaku kepada tenaga yang tersimpan di dalam spring? What happens to the energy stored in the spring?
Tenaga keupayaan kenyal ditukarkan kepada tenaga kinetik. The elastic potential energy is transferred to kinetic energy.
© Nilam Publication Sdn. Bhd.
05 FIZ Form 4 Ch2 D 2F.indd 96
96
9/10/13 4:14 PM
MODUL • Fizik TINGKATAN 4
Contoh/Example
(g = 10 m s–2)
1 Sebiji kelapa berjisim 1.2 kg jatuh dari satu ketinggian. Abaikan rintangan udara.
Sebelum kelapa jatuh/Before the coconut falls, Tenaga keupayaan graviti = mgh Gravitational potential energy = mgh Tenaga kinetik/Kinetic energy = 0
A coconut of mass 1.2 kg drops from a height. Ignore air resistance.
(a) (i) Tentukan jarak buah kelapa itu jatuh dalam masa 2.0 s. Determine the distance the coconut falls in 2.0 s.
h
(ii) Tentukan halajunya, selepas 2.0 s. Determine its velocity after 2.0 s.
(b) (i) Berapakah jumlah kehilangan tenaga keupayaan graviti selepas 2.0 s? What is the loss of its gravitational potential energy after 2.0 s?
(ii) Berapakah tenaga kinetiknya?/What is its kinetic energy then? (c) Apakah yang boleh dikatakan tentang kehilangan tenaga keupayaannya dan tenaga kinetik yang diperoleh? What can be said about the loss of its gravitational potential energy and the kinetic energy gained?
U N I T
Penyelesaian/Solution (a) (i) s = ut + 1 at2 2 1 = 0 + (10 m s–2)(2.0 s)2 2 = 20.0 m (ii) v = u + gt v = 0 + (10 m s–2)(2.0 s) = 20.0 m s–1 (b) (i) E = mgh = (1.2 kg) (10 m s–2) (20.0 m) = 240 J (ii) Tenaga kinetik/Kinetic energy = 240 J (c) Tenaga keupayaan graviti yang hilang telah ditukarkan kepada tenaga kinetik.
2
Lost of gravitional potential energy has been changed to kinetic energy.
2 Sebiji durian jatuh dari ketinggian 15 m. Berapakah halaju buah durian itu sejurus sebelum ia menghentam tanah? (Andaikan bahawa g = 10 m s-2) A durian falls from a height of 15 m. What is the velocity of the durian just before it hits the ground? (Assume that g = 10 m s-2)
Penyelesaian/Solution mgh = ½ mv2 v2 = 2gh = 2 × 10 m s–2 × 15 m v = 17.32 m s–1 3 Sebiji bola dilepaskan pada titik A dari ketinggian 0.8 m dengan menggunakan landasan licin. Berapakah halaju bola itu pada titik B? A ball is released at point A from a height of 0.8 m using a smooth inclined plane. What is the velocity of the ball at point B?
Penyelesaian/Solution Jumlah tenaga di A = Jumlah tenaga di B
A
0.8 m
Total energy at A = Total energy at B
mgh = v2 = = ∴ v =
½ mv2 2gh 2 × 10 m s–2 × 0.8 m = 16 m2 s–2 4 m s–1 97
05 FIZ Form 4 Ch2 D 2F.indd 97
B
© Nilam Publication Sdn. Bhd.
9/10/13 4:14 PM
MODUL • Fizik TINGKATAN 4
Definisi kuasa Define power
kadar kerja dilakukan
Kuasa ditakrifkan sebagai
Power is defined as the rate of work done or
Kuasa, P =
atau
kadar tenaga ditukarkan .
the rate of energy transformed .
Kerja Tenaga = Masa Masa
Power, P =
Work Energy = Time Time
J s–1 / S.I unit of power is
Unit SI bagi kuasa ialah watt, W atau
watt, W
or
J s–1 .
Kuasa 1 W dihasilkan apabila 1 J kerja dilakukan dalam masa 1 saat. A power of 1 W is generated when 1 J of work is done in 1 second.
U N I T
2
Menerangkan kecekapan peralatan
Kecekapan =
Explain what efficiency of a device is
Efficiency =
Contoh/Example
Kuasa output × 100% Kuasa input
Output power × 100% Input power
(g = 10 m s–2)
1 Seorang murid yang berjisim 45 kg mengambil masa 6 s untuk menaiki tangga yang mempunyai 36 anak tangga. Jika tinggi setiap anak tangga ialah 12 cm, kirakan A student of mass 45 kg takes 6 s to climb a flight of stairs that has 36 steps. If each step is 12 cm high, calculate the
(a) kerja yang dilakukan oleh murid itu work done by the student
(b) kuasa murid itu power of the student
12 cm
Penyelesaian/Solution (a) W = mgh = (45 kg) (10 m s–2) × (36 anak tangga × 0.12 m setiap anak tangga) (45 kg) (10 m s–2) × (36 steps × 0.12 m each step)
= 1 944 J
(b) P = 1 944 J 6s = 324 W 2 Sebuah motor mengangkat pemberat yang berjisim 1.5 kg pada ketinggian 1.0 m dalam masa 4.0 s. Berapakah kuasa motor itu? A motor lifting a weight having a mass of 1.5 kg up to a height of 1.0 m in 4.0 s. What is the power of the motor?
Penyelesaian/Solution mgh P = t 1.5 kg × 10 m s–2 × 1 m = 4.0 s = 3.75 W © Nilam Publication Sdn. Bhd.
05 FIZ Form 4 Ch2 D 2F.indd 98
98
9/10/13 4:14 PM
MODUL • Fizik TINGKATAN 4
3 Sebuah enjin petrol mempunyai kuasa output 96 kJ per minit. Berapakah kuasa input jika kecekapan enjin itu ialah 20%? A petrol engine has an output power of 96 kJ per minute. What is the input power if the engine efficiency is 20%?
Penyelesaian/Solution 3 Kuasa output/Output power = 96 × 10 J 60 s = 1 600 W Kecekapan = Kuasa output × 100% Kuasa input ∴ 20% = 1 600 W × 100% Kuasa input Kuasa input = 1 600 W × 100% 20% = 8 000 W
Efficiency =
Output power × 100% Input power
∴ 20% = 1 600 W × 100% Input power
Input power = 1 600 W × 100%
20%
= 8 000 W
U N I T
Latihan/Exercises
(g = 10 m s–2)
1 Sebuah troli dilepaskan dari keadaan rehat pada titik X. Berapakah halaju troli di titik Y?
2
2 kg
X
A trolley is released from rest at point X. What is the velocity of the trolley at point Y?
Penyelesaian/Solution Dari X ke Y, jarak tegak = 1.5 m From X to Y, the vertical distance = 1.5 m ∴ h = 1.5 m mgh = 1 mv2 2 v2 = 2gh = 2 × (10 m s–2)(1.5 m) ∴ v = 5.48 m s–1
2.5 m
Y 1.0 m Z
vB = ?
2 Sebiji bola bergerak di sepanjang permukaan mengufuk yang licin dengan halaju 6 m s-1. Bola itu kemudiannya bergerak naik ke atas satu satah condong licin. Ketinggian satah condong itu ialah 1.5 m. Berapakah halaju bola itu di titik B?
B
A ball is moving along a smooth horizontal surface at a velocity of 6 m s-1. The ball then moves up a smooth inclined plane. The height of the inclined plane is 1.5 m. What is its velocity at point B?
Penyelesaian/Solution Jumlah tenaga di A = Jumlah tenaga di B
1.5 m vA = 6 m s–1 A
Total energy at A = Total energy at B
1 mv 2 = A 2 1 (6 m s–1)2 = 2 1 v 2 = 2 B vB =
mgh + 1 mB2 2 (10 m s–2)(1.5 m) + 1 vB2 2 (18 – 15) m2 s–2 2.45 m s–1
99
05 FIZ Form 4 Ch2 D 2F.indd 99
© Nilam Publication Sdn. Bhd.
9/10/13 4:14 PM
MODUL • Fizik TINGKATAN 4
3 Sebiji bola tenis dilontar ke atas dengan halaju awal 20 m s-1. Berapakah tinggi maksimum yang boleh dicapai oleh bola tersebut? A tennis ball is thrown upwards with an initial velocity 20 m s-1. What is the maximum height that the ball can achieve?
Penyelesaian/Solution 2 ½ mv2 = mgh, h = v 2g –1 2 = (20 m s –2) 2(10 m s ) = 20 m 4 Nyatakan bentuk tenaga di titik/State the form (s) of energy at point (a) P = Tenaga keupayaan graviti Gravitational potential energy
U N I T
2
(b) Q = Tenaga kinetik/Kinetic energy (c) R = Tenaga kinetik + Tenaga keupayaan graviti
S
P
Kinetic energy + Gravitational potential energy
Ketinggian maksimum Maximum height
R Q
(d) S = Tenaga keupayaan graviti
Bandul ringkas Simple pendulum
Gravitational potential energy
5 Seorang budak lelaki berjisim, m, sedang berlari menaiki sebuah tangga. Dia mengambil masa, t, untuk sampai ke puncak. Berapakah kuasa budak itu? Beri jawapan anda dalam sebutan m, g, Y dan t.
Ketinggian minimum Minimum height
X
A boy of mass, m. runs up the stairs. He takes time, t, to reach the top. What is the power of the boy? Give your answer in terms of m, g, Y and t.
Y
Penyelesaian/Solution mgY mgY Kuasa = Power = t t
6 Sebuah kereta dengan kecekapan 25% menghasilkan 3 000 J tenaga mekanikal setiap saat. Berapakah kuasa output enjin itu? A car engine with an efficiency of 25% produces 3 000 J of mechanical energy per second. What is the output power of the engine?
Penyelesaian/Solution Kuasa output × 100% Kuasa input Kuasa output 25% = × 100% 3 000 W 25% Kuasa output = × 3 000 W 100% = 750 W Kecekapan =
© Nilam Publication Sdn. Bhd.
05 FIZ Form 4 Ch2 D 2F.indd 100
Output power × 100% Input power Output power × 100% 25% =
Efficiency =
3 000 W
25% Output power = × 3 000 W 100% = 750 W
100
9/10/13 4:14 PM
MODUL • Fizik TINGKATAN 4
7 Sebuah kren mengangkat beban 500 kg ke ketinggian 12 m dalam masa 8 s. Kuasa input ialah 45 000 W, berapakah kecekapan motor yang digunakan oleh kren itu? A crane lifts a load 500 kg to a height of 12 m in 8 s. The power input is 45 000 W, what is the efficiency of the motor used in the crane?
12 m
Penyelesaian/Solution Kecekapan =
Kuasa output × 100% Kuasa input
500 kg × 108ms s
–2
=
× 12 m
45 000 W
Efficiency =
Output power × 100% Input power
× 100%
7 500 W × 100% 45 000 W = 16.67% =
U N I T
8 Seorang budak lelaki berjisim 30 kg sedang duduk di atas puncak satu papan gelongsor condong pada ketinggian 2.5 m dari tanah. Apabila budak lelaki itu menggelongsor menuruni papan gelongsor, kerja yang dilakukan untuk mengatasi geseran ialah 510 J.
2
A boy of mass 30 kg sitting on the top end of an inclined sliding board at a height of 2.5 m from the ground. When the boy slides down the inclined board, the work done to overcome friction is 510 J.
2.5 m
Berapakah halaju pelajar itu sejurus sebelum dia menyentuh tanah? What is the velocity of the student just before he touches the ground?
Penyelesaian/Solution
1 mv2 + 510 J 2 1 (30 kg)(10 m s–2)(2.5 m) = (30 kg)v2 + 510 J 2 ∴ v = 4 m s–1 mgh =
101
05 FIZ Form 4 Ch2 D 2F.indd 101
© Nilam Publication Sdn. Bhd.
9/10/13 4:14 PM
MODUL • Fizik TINGKATAN 4
9 Abu bersama basikalnya menuruni satu cerun bukit yang mempunyai ketinggian 3 m pada halaju awalnya 2 m s-1 tanpa mengayuh. Pada kaki bukit, halajunya ialah 6 m s-1. Abu and his bicycle go down the slope of a hill of 3 m high at an initial velocity of 2 m s-1, without pedaling. At the foot of the hill, the velocity is 6 m s-1. u = 2 m s–1
v = 6 m s–1 3m
Diberi bahawa jisim Abu dengan basikalnya ialah 75 kg, cari Given that the mass of Abu with his bicycle is 75 kg, find U N I T
2
(a) tenaga kinetik awal dipunyai oleh Abu dengan basikal. the initial kinetic energy of Abu and his bicycle. (b) tenaga keupayaan graviti awal yang dipunyai oleh Abu dan basikalnya. the initial gravitational potential energy of Abu and his bicycle.
(c) kerja dilakukan menentang geseran sepanjang cerun. the work done against friction along the slope.
Penyelesaian/Solution (a) Tenaga kinetik/Kinetic energy 1 = mv2 2 1 = (75 kg)(2 m s–1)2 2 = 150 J (b) mgh = (75 kg)(10 m s–2)(3 m) = 2 250 J (c) Tenaga di puncak bukit = Tenaga di kaki bukit Energy at the top of the hill = Energy at the foot of the hill
1 (75 kg)(6 m s–1)2 + geseran/friction 2 Geseran/Friction = 1 050 J 2 250 J + 150 J =
© Nilam Publication Sdn. Bhd.
05 FIZ Form 4 Ch2 D 2F.indd 102
102
9/10/13 4:14 PM
MODUL • Fizik TINGKATAN 4
10 Rajah menunjukkan atlit lompat bergalah berjisim 60 kg melompat melepasi palang pada ketinggian 5.0 m. J, K, L, M, N, O, P dan Q menunjukkan beberapa peringkat lompatan yang dibuat oleh atlit. Pada titik N, ketinggian atlit dari paras palang ialah 0.2 m The diagram shows a pole vault jumper of mass 60 kg jumping over the bar of height 5.0 m. J, K, L, M, N, O, P and Q show the different stages of the jump made by the athlete. At point N, the athlete clears the bar with 0.2 m to spare. Galah Pole N
O
M P
L J
Q
K Tilam getah yang tebal A thick rubber maltress
U N I T
2
(a) Mengapakah atlit diperlukan untuk memecut kepada halaju tertentu pada peringkat J ke K sebelum dia mula melompat? Why is the athlete required to accelerate from J to K to a certain velocity before he begins to jump?
Untuk menambahkan tenaga kinetik. Apabila halaju bertambah, maka tenaga kinetik bertambah. To increase the kinetic energy. Kinetic energy increases with velocity.
(b) Terangkan mengapa galah itu perlu dibengkokkan di L. Explain why the pole has to be bent at L.
Untuk mendapatkan tenaga keupayaan kenyal yang maksimum. To get maximum elastic potential energy.
(c) Kirakan tenaga keupayaan graviti bagi atlit pada titik N. Calculate the gravitational potential energy of the athlete at point N.
E = mgh = (60 kg) × (10 m s–2) × (5.2 m) = 3 120 J
(d) Berapakah pecutan menegak atlit di peringkat P? What is the vertical acceleration of the athlete at stage P?
10 m s-2 (e) Mengapa sebuah tilam getah yang tebal diletakkan di kawasan di mana atlit mendarat? Why is a thick rubber mattress placed in the area where the athlete lands?
Menambahkan masa hentaman untuk mengurangkan daya impuls. Increase the time of collision to reduce impulsive force.
103
05 FIZ Form 4 Ch2 D 2F.indd 103
© Nilam Publication Sdn. Bhd.
9/10/13 4:14 PM
MODUL • Fizik TINGKATAN 4
2.11
MEMAHAMI KEKENYALAN
UNDERSTANDING ELASTICITY
Aktiviti 1: Idea tentang kekenyalan/Activity 1: Idea of elasticity Definisi kekenyalan Define Elasticity
Sifat bahan yang membolehkan objek kembali ke panjang dan bentuk asal apabila daya yang dikenakan ke atasnya dialihkan. A property of matter that enables an object to return to its original length and shape when the force that acts on it is removed.
Tiada daya luar dikenakan. Molekul berada pada kedudukan antara molekul adalah sifar. No external force is applied. The molecules are at their Intermolecular force is equal to zero. U N I T
asalnya . Daya
original
positions.
Memampatkan pepejal menyebabkan molekulnya bergerak lebih rapat antara tolakan satu sama lain. Daya antara molekul bertindak untuk menolak
2
molekul kembali kepada kedudukan asalnya. closer Compressing a solid causes its molecules to be moved to each other. Repulsive intermolecular forces act to push the molecules back to their original positions.
Meregangkan pepejal menyebabkan molekulnya bergerak menjauhi antara satu sama lain. Daya tarikan antara molekul bertindak untuk menarik kembali molekul kepada kedudukan asalnya. further from each other. Stretching a solid causes its molecules to be moved Attractive intermolecular forces act to pull back the molecules to their original positions.
Meregang wayar dengan daya luar Stretching a wire by an external force:
• Molekulnya akan
menjauhi further
Its molecules will move
antara satu sama lain. away from one another.
• Daya tarikan yang kuat akan bertindak di antara molekul untuk menentang regangan yang dikenakan. Strong attractive forces act between the molecules to
oppose
the stretching.
Apabila daya luar dialih/When the external force is removed: • daya tarikan antara molekul membawa molekul kembali ke kedudukan asalnya . the attractive intermolecular forces bring the molecules back to their positions.
• Wayar itu The wire © Nilam Publication Sdn. Bhd.
05 FIZ Form 4 Ch2 D 2F.indd 104
kembali returns
original
ke kedudukan asalnya.
to its original position.
104
9/10/13 4:14 PM
MODUL • Fizik TINGKATAN 4
Aktiviti 2: Hubungan antara daya dan pemanjangan spring
Activity 2: Relationship between force and extension of a spring
Tujuan
Untuk menyiasat hubungan antara daya dan pemanjangan spring
Aim
To investigate the relationship between a force and the extension of a spring
Radas
Spring keluli, lima pemberat berjisim 50 g dan pemegang, pembaris meter, kaki retort
Apparatus
Steel spring, five 50 g slotted weights and holder, metre rule, retort stand
Hipotesis
Pemanjangan spring bertambah dengan penambahan daya
Hypothesis
Extension of a spring increases as the force increases.
Pemboleh ubah dimanipulasi
Daya/berat/jisim
Manipulated variable
Force/weight/mass
Pemboleh ubah bergerak balas
Pemanjangan spring
Responding variable
Extension of a spring
Pemboleh ubah dimalarkan
Diameter spring
Fixed variable
U N I T
2
Diameter of the spring
1 Tandakan kedudukan awal pin pada pembaris meter apabila tiada pemberat dilekatkan kepada spring, l0.
Pembaris meter Metre rule Pin/Pin
Mark the initial position of the pin on the metre rule when no weight is attached to the spring, l0.
Prosedur Procedure
2 Gantungkan pemberat berjisim 50 g di bahagian hujung spring dan bandingkan kedudukan baru pin sekarang dengan kedudukan asalnya.
Pemberat Weight
Attach a slotted weight of 50 g to the end of the spring and compare the new position of the pin Spring/Spring with its initial position. 3 Ukur pemanjangan spring, x = l – l0./Measure the extension of the spring, x = l – l0 .
4 Ulangi eksperimen dengan jisim 100 g, 150 g, 200 g dan 250 g. Repeat the experiment with mass 100 g, 150 g, 200 g and 250 g.
Jadualkan data bagi m, F, l dan x./Tabulate data for m, F, l and x. l0 =
cm Jisim/kg Mass/kg
Daya/Force, F F = mg / N
l / cm
Pemanjangan/Extension, x x = l – l0 / cm
Merekodkan data Recording data
Plotkan graf daya, F melawan pemanjangan spring, x. Plot a graph of Force, F against extension of the spring, x.
105
05 FIZ Form 4 Ch2 D 2F.indd 105
© Nilam Publication Sdn. Bhd.
9/10/13 4:14 PM
MODUL • Fizik TINGKATAN 4 Daya/Force, F/N
Menganalisis data Analysis data Pemanjangan/Extension, x/cm
0
1 Daripada graf, apakah hubungan antara daya dengan pemanjangan spring, x? From the graph, what is the relationship between a force and the extension of the spring, x?
Daya, F, berkadar langsung dengan pemanjangan spring, x The force, F, is directly proportional to the extension of the spring, x
2 Kira kecerunan graf. Tunjukkan bagaimana anda mendapat kecerunan daripada graf. Calculate the gradient of the graph. Show how you get the gradient from the graph.
Kecerunan =
Perbincangan Discussion U N I T
Daya Pemanjangan spring
Force Gradient = Extension of spring
3 Langkah berjaga-jaga perlu diambil supaya spring itu tidak diregangkan dengan berat yang berlebihan. Terangkan mengapa. Precaution should be taken so that the spring is not stretched by excessive weights. Explain this.
2
Jika spring diregangkan dengan berat yang berlebihan, ia mungkin tidak akan kembali ke panjang asal kerana telah melebihi had kenyal. If the spring is stretched by too large a weight, it might not return to its original length due to its exceeding the elastic limit.
Nyatakan Hukum Hooke State Hooke’s Law
terus Pemanjangan spring berkadar kenyalnya had tidak dilebihi. directly The extension of a spring is elastic limit is not exceeded.
dengan daya yang dikenakan asalkan proportional to the applied force provided the
F = kx di mana/where daya force F = ke atas spring / on the spring x = pemanjangan spring / extension of the spring k = Graf dayapemanjangan spring Force-extension graph Daya/Force, F/N
pemalar spring
/
spring constant
Berdasarkan graf/Based on the graph: • F berkadar terus kepada x. / F is directly proportional to x. pemalar spring • Kecerunan graf = bagi spring, k. The gradient of the graph =
spring constant
of the spring, k.
• Luas di bawah graf adalah sama dengan kerja yang dilakukan untuk memanjangkan spring = tenaga keupayaan elastik Area under the graph is equal to the work done to extend the spring = elastic potential energy
0
Pemanjangan/Extension, x/cm
© Nilam Publication Sdn. Bhd.
05 FIZ Form 4 Ch2 D 2F.indd 106
• Kerja yang dilakukan = tenaga keupayaan kenyal Work done = elastic potiential energy ½ Fx = ½ kx2 106
9/10/13 4:14 PM
MODUL • Fizik TINGKATAN 4
Had kenyal bagi spring The elastic limit of a spring F/N Had kenyal Elastic limit x (cm)
0
Berat boleh
maksimum yang boleh dikenakan kepada spring supaya spring itu masih kembali ke panjang asalnya apabila daya dialihkan.
The maximum weight that can be applied to a spring such that the spring will be able to return to its original length when the force is removed.
Jika daya regangan spring melebihi had kenyal, spring tidak kembali ke panjang asal walaupun tiada lagi daya dikenakan ke atasnya. If a force stretches a spring over its elastic limit, the spring cannot return to its original length even though the force no longer acts on it.
Hukum Hooke tidak dipatuhi lagi./The Hooke’s Law is not obeyed anymore. Pemalar daya spring, k Force constant of a spring, k Spring keras F (N) Stiff spring 75 Spring lembut Soft spring x (m)
12.5 0
Pemalar daya spring, k, ialah daya yang diperlukan untuk menghasilkan satu unit pemanjangan spring. The force constant, k, of a spring is the force required to produce one unit of extension of the spring.
F unit: N m-1 x k ialah pengukuran kekerasan spring./k is a measurement of the stiffness of the spring. tinggi • Spring dengan pemalar daya, k, yang adalah lebih susah untuk k=
dipanjangkan dan dikatakan lebih keras.
U N I T
2
The spring with a high force constant, k, is harder to extend and is said to be more stiff.
kecil • Spring dengan pemalar daya, k, yang adalah lebih mudah untuk dipanjangkan dan dikatakan lebih lembut atau kurang keras. A spring with a or less stiff.
small
force constant, k, is easier to extend and is said to be softer
Faktor-faktor yang mempengaruhi kekenyalan/Factors that affect elasticity Faktor Factor
Perubahan faktor/Change in factor
Bagaimana ia mempengaruhi kekenyalan How it affects the elasticity
Spring lebih pendek/Shorter spring
Kurang kenyal/Less elastic
Spring lebih panjang/Longer spring
Lebih kenyal/More elastic
Diameter dawai spring
Dawai lebih tebal/Thicker wire
Kurang kenyal/Less elastic
Diameter of spring wire
Dawai lebih nipis/Thinner wire
Lebih kenyal/More elastic
Diameter lebih kecil/Smaller diameter
Kurang kenyal/Less elastic
Diameter lebih besar/Larger diameter
Lebih kenyal/More elastic
Panjang Length
Diameter spring Diameter spring
Jenis bahan Type of material
Spring diperbuat daripada pelbagai bahan. Perubahan kekenyalan mengikut jenis bahan yang digunakan. Springs are made of different materials. Elasticity changes according to the type of material.
107
05 FIZ Form 4 Ch2 D 2F.indd 107
© Nilam Publication Sdn. Bhd.
9/10/13 4:14 PM
MODUL • Fizik TINGKATAN 4
Susunan spring yang serupa Arrangement of identical springs
Secara bersiri/In series
Secara selari/In parallel Spring serupa Identical springs
Spring serupa Identical springs Beban Load
Beban yang sama dikenakan kepada setiap spring
Beban dikongsi bersama sesama spring
The same load is applied to each spring.
The load is shared equally among the springs.
Tegangan dalam setiap spring = T
Tegangan dalam setiap spring =
Tension in each spring = T
Pemanjangan spring = x Extension of each spring = x
Jumlah pemanjangan = 2x
U N I T
2
Beban Load
Tension in each spring = T 2
Pemanjangan setiap spring =
Total extension = 2x
Extension of each spring = x 2
Jika bilangan spring digunakan = n, Jumlah pemanjangan = nx
If number of springs used = n,
If number of springs used = n, The total extension = nx
Contoh/Example
T 2
x 2
Jika bilangan spring digunakan = n, x Jumlah pemanjangan = n x The total extension =
n
1
Panjang asal spring ialah 5 cm. Dengan beban berjisim 20 g, spring memanjang kepada 7 cm. The original length of a spring is 5 cm. With a load of mass 20 g, the length of the spring is extended to 7 cm. Tentukan/Determine
(a) pemanjangan spring dengan beban 40 g.
the extension of the spring with a load 40 g.
(b) panjang spring dengan beban 60 g.
the length of the spring with a load of 60 g.
Penyelesaian/Solution (a) 20 g → 7 cm – 5 cm = 2 cm 40 g → 4 cm (b) 20 g menghasilkan pemanjangan 2 cm
20 g gives an extension of 2 cm ∴ 60 g → pemanjangan/extension 6 cm
∴ panjang spring dengan beban 60 g = 5 cm + 6 cm = 11 cm
length of spring with 60-g load = 5 cm + 6 cm = 11 cm
© Nilam Publication Sdn. Bhd.
05 FIZ Form 4 Ch2 D 2F.indd 108
108
9/10/13 4:14 PM
MODUL • Fizik TINGKATAN 4
Contoh/Example
2
Rajah/Diagram (a)
(a) Panjang asal spring ialah 10.0 cm. Apabila ia diregangkan dengan daya 6 N, ia memanjang kepada 13.0 cm. Apakah pemalar spring?
F = 6.0 N 13.0 cm
(b)
F = 12.0 N
(c)
F = 12.0 N
The original length of a spring is 10.0 cm. When it is stretched by a force of 6 N, it extends to 13.0 cm. What is the spring constant?
x = 13 cm – 10 cm = 3 cm 6N F k= = = 2 N cm–1 3 cm x
(b) Dua spring yang serupa disambungkan secara bersiri seperti ditunjukkan dalam rajah (b). Two identical springs are connected in series as shown in diagram (b).
(i) Berapakah jumlah panjang spring jika diregangkan oleh daya 12.0 N? What is the total length of the spring if stretched by 12.0 N force? 6 N → 3 cm Jumlah panjang/Total length
12 N → 6 cm = 10 cm + 10 cm + 6 cm + 6 cm = 32 cm (ii) Berapakah pemalar spring bagi sistem spring di (b)?/What is the spring constant of the spring system in (b)? 12 N F = = 1 N cm–1 k= 12 cm x (c) 2 spring yang serupa disambungkan secara selari seperti ditunjukkan dalam rajah (c).
U N I T
2
2 identical springs are connected in parallel as shown in diagram (c).
(i) Berapakah jumlah panjang sistem spring itu?/What is the total length of the spring system? 4 N → 1 cm Jumlah panjang/Total length ∴ 12 N → 3 cm = 10 cm + 3 cm = 13 cm
(ii) Berapakah pemalar spring bagi sistem spring di (c)?/What is the spring constant of the spring system in (c)? 12 N = 4 N cm–1 k= 3 cm Contoh/Example
3
Spring A memanjang 2 cm apabila ia digantung dengan pemberat 10 g. Spring B memanjang 4 cm apabila ia digantung dengan pemberat 10 g. Cari jumlah regangan pada setiap sistem spring seperti ditunjukkan dalam rajah sebelah. Spring A extends by 2 cm when a 10 g weight is hung on it. Spring B extends by 4 cm when a 10 g weight is hung on it. Find the total extension in each of the spring systems shown in the diagrams on the right.
Penyelesaian/Solution (a) A: 10 g → 2 cm B: 10 g → 4 cm 20 g → 4 cm 20 g → 8 cm Jumlah pemanjangan/Total extension = 4 cm + 8 cm = 12 cm (b) 10 g → 1 cm 50 g → 5 cm Pemanjangan sistem/Extension in the system = 5 cm (c) Sistem B/System B : 10 g → 2 cm ∴ 40 g → 8 cm A : 10 g → 2 cm ∴ 40 g → 8 cm ∴ Pemanjangan sistem/Extension in the system = 8 cm + 8 cm = 16 cm 109
05 FIZ Form 4 Ch2 D 2F.indd 109
A
A
A
B
B
A
B 50 g 20 g 40 g (a)
(b)
(c)
© Nilam Publication Sdn. Bhd.
9/10/13 4:14 PM
MODUL • Fizik TINGKATAN 4
Contoh/Example
4
Panjang asal spring ialah 12 cm. Dengan beban 20 g, panjang spring dipanjangkan kepada 15 cm. Berapakah tenaga keupayaan kenyal yang tersimpan di dalam spring? The original length of a spring is 12 cm. With a load of 20 g, the length of the spring is extended to 15 cm. What is the elastic potential energy stored in the spring?
Penyelesaian/Solution 1 E = Fx 2 1 = × (0.02 kg × 10 m s–2) × 0.03 m 2 = 0.003 J Contoh/Example
5
Rajah menunjukkan graf daya, F, melawan pemanjangan, x, bagi spring. Berapakah tenaga keupayaan kenyal yang tersimpan apabila spring diregangkan sebanyak 0.4 m? U N I T
2
The diagram shows a graph of force, F, against extension, x, for a spring. What is the elastic potential energy stored when the spring is extended by 0.4 m?
F/N 20
Penyelesaian/Solution 1 E = Fx 2 1 = (20 N) (0.4 m) 2 = 4.0 J Contoh/Example
0
0.4
x/m
6
Rajah menunjukkan sebiji bebola keluli berjisim 10 g ditolak pada penghujung satu spring di sepanjang permukaan licin. Panjang asal spring ialah 14 cm dan pemalar spring ialah 200 N m-1. The diagram shows a steel ball of mass 10 g being pushed against one end of a spring along a smooth surface. The original length of the spring is 14 cm and its spring constant is 200 N m-1. 14 cm
10 cm Daya Force
Tentukan/Determine (a) tenaga keupayaan kenyal tersimpan di dalam spring./the elastic potential energy stored in the spring. 1 E = kx2 2 1 = (200 N m–1) (0.04 m)2 2 = 0.16 J (b) halaju maksimum bola itu selepas daya mampatan pada spring itu dialihkan. the maximum velocity of the ball after the force of compression on the spring is removed.
1 mv2 = 0.16 J 2 2 × 0.16 J v 2 = 0.01 kg = 32 m2 s–2 v = 5.66 m s–1
© Nilam Publication Sdn. Bhd.
05 FIZ Form 4 Ch2 D 2F.indd 110
J Nm = kg kg (kg m s–2)(m) = kg 2 –2 =m s
Perhatian/Note:
110
9/10/13 4:14 PM
MODUL • Fizik TINGKATAN 4
Latihan/Exercises Rajah di bawah menunjukkan susunan radas dalam eksperimen untuk menentukan hubungan di antara pemanjangan spring, x, dengan berat, W. Hubungan di antara x dengan W ditunjukkan dalam graf di bawah. The diagram shows the arrangement of an apparatus in an experiment to determine the relationship between the extension, x, of the spring with weight W. The relationship between x and W is shown in the graph below. Pemanjangan, x/cm Extension, x/cm 7 Spring Spring
6 5 4
Pemberat berslot Slotted mass
3 2 1 0
0
2
4
6
8
10 12 14 Berat, W/N Weight, W/N
(a) (i) Nyatakan hubungan antara x dan W.
U N I T
2
State the relationship between x and W.
x berkadar langsung dengan W, asalkan had kenyal tidak dilebihi. x is directly proportional to W, provided the elastic limit is not exceeded.
(ii) Namakan hukum saintifik yang terlibat dalam hubungan yang dinyatakan di (a)(i). Name the scientific law involved in the relationship stated in (a)(i). Hukum Hooke/Hooke’s Law
(b) Tandakan dengan tanda pangkah (×) had kenyal spring di dalam graf.
Mark a cross (×) at the elastic limit of the spring on the graph.
(c) Berdasarkan graf, tentukan pemalar daya bagi spring, k (dalam unit N m–1).
Based on the graph, determine the force constant of a spring, k (in N m–1).
k=
10 N = 250 N m–1 0.04 m
(d) Tenaga keupayaan kenyal tersimpan dalam spring apabila ia diregangkan. Kirakan tenaga ini dalam spring apabila ia memanjang sebanyak 4 cm. Elastic potential energy is stored in the spring when it is extended. Calculate this energy in the spring when it is extended by 4 cm.
E = 1 Fx 2 = 1 (10 N) (0.04 m) 2 = 0.2 J
111
05 FIZ Form 4 Ch2 D 2F.indd 111
© Nilam Publication Sdn. Bhd.
9/10/13 4:14 PM
MODUL • Fizik TINGKATAN 4
UNIT
3
DAYA DAN TEKANAN FORCES AND PRESSURE
3.1 MEMAHAMI KONSEP TEKANAN/UNDERSTANDING PRESSURE • Mendefinisikan konsep tekanan dan menyatakan p =
F F /Define pressure and state that p = A A
• Menghuraikan aplikasi tekanan/Describe applications of pressure • Menyelesaikan masalah berkaitan dengan tekanan/Solve problems involving pressure
3.2 Memahami tekanan di dalam cecair/UnderstaNding pressure in liquids • Mengaitkan kedalaman dengan tekanan di dalam cecair/Relate depth to pressure in a liquid • Mengaitkan ketumpatan dengan tekanan di dalam cecair/Relate density to pressure in a liquid • Menerangkan konsep tekanan di dalam cecair dan nyatakan P = h ρ g Explain pressure in a liquid and state that P = h ρ g
• Menghuraikan aplikasi tekanan di dalam cecair/Describe applications of pressure in liquids • Menyelesaikan masalah berkaitan tekanan di dalam cecair/Solve problems involving pressure in liquids
3.3 Memahami tekanan gas dan tekanan atmosfera
UnderstaNding gas pressure and atmospheric pressure
U N I T
3
• Menerangkan tekanan gas/Explain gas pressure • Menerangkan tekanan atmosfera/Explain atmospheric pressure • Menghuraikan aplikasi tekanan atmosfera/Describe applications of atmospheric pressure • Menyelesaikan masalah berkaitan tekanan atmosfera dan tekanan gas Solve problems involving atmospheric pressure and gas pressure
3.4 Aplikasi Prinsip Pascal/Applying Pascal’s principle • Menyatakan Prinsip Pascal/State Pascal’s Principle • Menerangkan sistem hidraulik/Explain hydraulic system • Menghuraikan aplikasi Prinsip Pascal/Describe applications of Pascal’s Principle • Menyelesaikan masalah berkaitan Prinsip Pascal/Solve problems involving Pascal’s Principle 3.5 Aplikasi Prinsip Archimedes/Applying Archimedes’ principle • Menerangkan daya apung (atau tujah ke atas)/Explain buoyant force (or upthrust) • Mengaitkan daya tujah ke atas dengan berat cecair yang disesarkan Relate buoyant force to the weight of the liquid displaced
• Menyatakan Prinsip Archimedes/State Archimedes’ Principle • Menghuraikan aplikasi Prinsip Archimedes/Describe applications of Archimedes' Principle • Menyelesaikan masalah berkaitan Prinsip Archimedes/Solve problems involving Archimedes’ Principle 3.6 Memahami Prinsip Bernoulli/Understanding Bernoulli’s principle • Menyatakan Prinsip Bernoulli/State Bernoulli’s Principle • Menerangkan bahawa halaju mempengaruhi tekanan di dalam bendalir Explain that velocity affects the pressure in a fluid
• Menghuraikan aplikasi Prinsip Bernoulli/Describe applications of Bernoulli’s Principle
© Nilam Publication Sdn. Bhd.
06 FIZ Form 4 Ch3 2F.indd 112
112
9/10/13 4:33 PM
MODUL • Fizik TINGKATAN 4
3.1
MEMAHAMI KONSEP TEKANAN UNDERSTANDING PRESSURE
Tekanan
daya ialah yang bertindak secara normal per unit luas permukaan sentuhan.
Definisi Tekanan Definition of Pressure
Nyatakan formula tekanan State the formula of pressure F
Pressure is the
force
surface per unit
area
acting perpendicularly to a of contact.
Tekanan =
Daya F = Luas permukaan A
Pressure =
Force F = Area A
Unit SI: N m-2 = Pascal = Pa SI unit: N m-2 = Pascal = Pa
A
Nyatakan hubungan antara tekanan, daya dan luas.
Tekanan meningkat apabila daya berkurang .
State the relationship between pressure, force and area.
meningkat
dan luas permukaan
The pressure increases when the force increases and the surface area decreases .
Aplikasi berkaitan Tekanan Tinggi Applications involving High Pressure
mengurangkan
• Tekanan ditingkatkan dengan Pressure can be increased by
reducing
Pisau yang tajam mempunyai luas permukaan yang kecil pada bilahnya supaya tekanan tinggi dapat dihasilkan untuk memotong daging. A sharp knife has a very small surface area on its cutting edge so that high pressure meat.
can be exerted to cut the
the surface area of contact (where force is a constant).
Tapak kasut yang berpaku pada kasut bola hanya mempunyai luas permukaan sentuhan yang kecil pada permukaan padang. Tekanan yang cukup tinggi pada tapak kasut yang berpaku membuatkan kasut pemain mendap ke dalam padang sedikit, untuk memberikan lebih cengkaman. The studs on the football boots have only a
U N I T
luas permukaan sentuhan (di mana daya dimalarkan).
small area of contact with the
field . The pressure under the studs is high enough
for the boots to sink into the field. This gives extra grip.
Paku, jarum dan pin mempunyai hujung yang tajam dengan luas permukaan yang kecil suatu
daya
3
. Apabila
dikenakan pada tekanan tersebut
kepala paku, akan menyebabkan hujungnya yang tajam itu menembusi sekeping papan dengan mudah. Nails, needles and pins have very sharp ends with very areas
small surface
force . When a applied to the head of a nail, the
is
pressure
will drive its sharp end into a piece of wood easily. Ibu jari Thumb Kepala paku tekan Head of thumbtack Kayu wood
113
06 FIZ Form 4 Ch3 2F.indd 113
Hujung tajam paku tekan Tip of thumbtack
© Nilam Publication Sdn. Bhd.
9/10/13 4:33 PM
MODUL • Fizik TINGKATAN 4
Aplikasi berkaitan Tekanan Rendah/Application involving Low Pressure menambahkan • Tekanan dikurangkan dengan luas permukaan sentuhan (di mana daya dimalarkan). Pressure is reduced by
increasing
the surface area of contact (where force is a constant).
Ski mempunyai luas permukaan yang besar untuk mengurangkan tekanan ke atas salji supaya
Traktor yang bergerak di atas tanah yang lembut mempunyai tayar yang
pemakainya tidak terbenam ke dalam salji itu.
ke atas tanah supaya tidak terbenam ke dalam tanah itu.
Skis have a large
surface
area to
pressure on the snow reduce the so that they do not sink into the snow.
lebar untuk mengurangkan tekanan
A tractor moving on a soft ground has broad tires to reduce the pressure on the ground so that they will not sink into the ground.
Pad bahu yang lebar
pada beg yang berat akan mengurangkan tekanan yang dikenakan pada bahu orang yang menyandang beg tersebut. wide shoulder pad A of a heavy bag will reduce the pressure exerted on the shoulder of the person carrying the bag. Pad bahu Shoulder pad
Latihan/Exercises U N I T
3
(g = 10 m s–2)
1 Terdapat sekotak barang runcit di atas meja yang Penyelesaian/Solution: F beratnya 160 N. Luas tapak permukaan kotak itu (a) PMaksimum = A 2 Minimum ialah 0.2 m . Berapakah jumlah tekanan yang mg = dikenakan oleh kotak tersebut ke atas permukaan Luas permukaan minimum atas meja itu? –2
0.5 kg × 10 m s -2 There is a full box of groceries which has a weight of = 2 = 1 000 N m (0.05 × 0.10) m 160 N on a table. The area of the base of the box is F 0.2 m2. What is the pressure exerted by the box on the (b) PMinimum = A Maksimum surface of table? mg = Penyelesaian/Solution: Pressure, P Luas permukaan maksimum Weight Berat = –2 0.5 kg × 10 m s Tekanan, P = Surface area = = 250 N m-2 Luas permukaan (0.2 × 0.10 m)2 160 N = = 800 N m-2 3 Jisim badan seorang budak lelaki ialah 60 kg. 0.2 m2
Tekanan yang dikenakan ke atas lantai oleh budak itu ialah 2 × 104 Pa. Berapakah luas permukaan antara 2 Suatu bongkah besi berukuran 0.05 m × 0.10 m tapak kasut budak lelaki tersebut dengan lantai? × 0.20 m diletakkan di atas lantai. Jika jisim The mass of a boy is 60 kg. The pressure exerted by bongkah besi tersebut ialah 0.5 kg, berapakah the boy on the floor is 2 × 104 Pa. What is the area A metal block with the dimensions of 0.05 m × 0.10 m of contact between the soles of the boy's shoes and × 0.20 m is placed on a floor. If the mass of the metal the floor? block is 0.5 kg, what is Penyelesaian/Solution: (a) nilai tekanan maksimum, dan mg P = the maximum pressure, and A 60 kg × 10 m s–2 (b) nilai tekanan minimum yang boleh dikenakan 2 × 104 Pa = A oleh bongkah besi itu ke atas permukaan (60 × 10) N lantai? A = (Perhatian/Note: the minimum pressure that can be exerted by the 2 × 104 Pa 1 Pa = 1 N m–2) metal block on the floor? = 3.0 × 10-2 m2 © Nilam Publication Sdn. Bhd.
06 FIZ Form 4 Ch3 2F.indd 114
114
9/10/13 4:33 PM
MODUL • Fizik TINGKATAN 4
3.2
MEMAHAMI TEKANAN DI DALAM CECAIR UNDERSTADING PRESSURE IN LIQUIDS
Tekanan dalam cecair Pressure in liquid
beratnya
Cecair di dalam bekas mengenakan tekanan disebabkan A liquid in a container exerts pressure because of its
weight
.
.
(a) Tekanan pada sebarang titik dalam cecair, pada kedalaman tertentu, sama dalam semua arah bertindak dengan magnitud yang . The pressure at any point in a liquid, at a particular depth, acts equally in all directions .
Ciri-ciri tekanan dalam cecair Characteristics of pressure in a liquid.
(b) Tekanan dalam cecair tidak dipengaruhi oleh
luas
permukaannya. area
The pressure in a liquid does not depend on its surface
.
(c) Tekanan dalam cecair bertindak dengan magnitud yang sama dalam semua arah dan tidak dipengaruhi oleh bentuk bekas. The pressure in a liquid acts equally in all directions and does not depend on the shape
Hubungan antara tekanan cecair dan kedalamannya Relationship between pressure of a liquid and its depth
of the container.
• Tekanan cecair kedalamannya. Liquid pressure is depth.
berkadar langsung
dengan
directly proportional
bertambah • Tekanan dalam cecair kedalamannya bertambah. The pressure in a liquid
increases
Tekanan cecair, P Liquid pressure, P
to its
apabila with depth.
0
Kedalaman cecair, h Depth of liquid, h
Tekanan cecair, P Liquid pressure, P
Hubungan antara ketumpatan cecair dan tekanannya Relationship between the density of a liquid and its pressure
• Tekanan ceair adalah berkadar langsung ketumpatannya. Liquid pressure is density.
directly proportional
Derive the formula for liquid pressure.
3
dengan
to its 0
Terbitkam persamaan untuk tekanan cecair.
U N I T
Ketumpatan, ρ Density, ρ
Bekas di sebelah kiri mempunyai luas tapak A. Ianya dipenuhi dengan cecair sehingga kedalaman h yang berketumpatan ρ. Hitungkan: The container on the left has a base area A. It is filled to a depth h with a liquid of density ρ. Calculate:
(a) isi padu cecair/volume of liquid = Ah (b) jisim cecair/mass of liquid = ρV = ρ(Ah) Ketumpatan, p Density, p
Luas tapak, A Base area, A
Kedalaman, h Depth, h
(c) berat cecair/Weight of liquid = mg = (ρAh)g (d) daya pada tapak/force on base = ρAhg F ρAhg (e) tekanan/pressure = = = ρgh A A (f) tekanan cecair/liquid pressure = ρgh
115
06 FIZ Form 4 Ch3 2F.indd 115
© Nilam Publication Sdn. Bhd.
9/10/13 4:33 PM
MODUL • Fizik TINGKATAN 4
Aplikasi tekanan dalam cecair/Applications of pressure in liquids (a) Tangki air/Water tank • Kebiasaannya tangki air ditempatkan di tempat yang tinggi supaya tekanan air yang dihasilkan tinggi adalah lebih . Normally a water tank is placed at
higher
higher .
level so that the water pressure produced is
(b) Empangan/Dam
• Dinding sebuah empangan adalah lebih tebal di bahagian dasar berbanding dengan bahagian atas supaya dapat menampung lebih tekanan di bahagian bawah. Ini adalah kerana tekanan bertambah apabila kedalaman air bertambah. thicker at the bottom than at the top so that it can withstand the pressure at the bottom. This is because water pressure increases with depth. The walls of a dam is much
greater
• Dinding tersebut dibina menggunakan bahan/konkrit yang lebih kuat untuk meningkatkan ketahanan dinding itu. The wall is constructed using
stronger
materials/concrete to increase the strength of the wall
(c) Kapal selam/Submarine
• Kapal selam dibina dengan lautan dalam A submarine is built with
U N I T
3
Latihan/Exercises
dinding yang tebal
thick walls
to withstand
untuk menahan tekanan yang sangat high
tinggi
di
pressure at greater depth.
(g = 10 m s–2)
1 Jika ketumpatan air laut ialah 1 150 kg m–3, kira tekanan air laut pada kedalaman 40 m. If the density of sea water is 1 150 kg m–3, calculate the water pressure at a depth of 40 m.
Penyelesaian/Solution:
P = ρgh = (1 150 kg m–3) (10 m s–2) (40 m) = 460 000 Pa
2 Rajah menunjukkan sebuah silinder mengandungi merkuri. Berapakah tekanan yang disebabkan oleh merkuri pada titik P? [Ketumpatan merkuri ialah 1.36 × 104 kg m-3] The diagram shows a cylinder containing mercury. What is the pressure caused by the mercury at the point P? [Density of mercury is 1.36 × 104 kg m-3]
Merkuri Mercury
0.8 m
Penyelesaian/Solution:
P
P = ρgh = (1.36 × 104 kg m–3) (10 m s–2) (0.8 – 0.2)m = 8.16 × 104 Pa
0.2 m
3 Rajah menunjukkan permukaan air di dalam tangki air rumah adalah 5 m di atas paip air sejuk di tingkat bawah. Berapakah tekanan air dalam paip air itu? [Ketumpatan air = 1 000 kg m-3] The diagram shows the surface of the water in a domestic water tank is 5 m above the water tap downstairs. What is the water pressure in the tap? [Density of water = 1 000 kg m-3]
Penyelesaian/Solution:
Tangki air Water tank
5m Pili Tap
P = ρgh = (1 000 kg m–3) (10 m s–2) (5 m) = 50 000 Pa
© Nilam Publication Sdn. Bhd.
06 FIZ Form 4 Ch3 2F.indd 116
116
9/10/13 4:33 PM
MODUL • Fizik TINGKATAN 4
3.3
MEMAHAMI TEKANAN GAS DAN TEKANAN ATMOSFERA
UNDERSTANDING GAS PRESSURE AND ATMOSPHERIC PRESSURE
Definisi tekanan gas Definition of gas pressure
Tekanan gas ialah
daya per unit luas
berlanggar dengan
dinding bekas
Gas pressure is the
force per unit area
the
Menerangkan tekanan atmosfera Explain atmospheric pressure
walls of container
yang dikenakan oleh
ketika
. gas molecules
exerted by the
as they collide with
.
berat udara
• Tekanan atmosfera ialah tekanan yang disebabkan oleh
equally in all directions
a. Tekanan atmosfera dipengaruhi oleh ketinggian Atmospheric pressure is influenced by the
height
upon us.
sama pada semua arah
• Tekanan atmosfera bertindak dengan magnitud yang Atmospheric pressure acts
ke atas kita.
weight of the air
Atmospheric pressure is the pressure caused by the
b. Ia
molekul-molekul gas
.
.
sesuatu objek di atas paras laut (altitud). of an object above sea level (altitude).
menurun
decreases
apabila altitud menurun./It when the altitude drops. rendah c. Semakin tinggi altitud, semakin ketumpatan dan suhu udara. At higher altitudes, the density and the temperature of the air are
d. Jadi
lower .
frekuensi perlanggaran
antara molekul-molekul dengan dinding bekas adalah kecil rendah dan ianya menyebabkan tekanan atmosfera menjadi . So the frequency of collisions between the molecules and the walls of the container is lower and causes atmospheric pressure to be lower .
Aktiviti untuk menunjukkan kewujudan tekanan atmosfera/Activities to show the existence of atmospheric pressure Kadbod Cardboard
Gelas/Glass
Terbuka Open
Air/Water
• Kadbod itu tidak terjatuh dan air tetap berada di dalam gelas. The cardboard does not fall and the water remains in the glass.
Daya paduan
yang terhasil disebabkan oleh tekanan atmosfera yang menekan pada
permukaan kadbod adalah lebih besar daripada berat air di dalam gelas itu.
the
weight
• Tin tersebut telah dikemekkan dan direnyukkan. The can is crushed and crumpled.
berkurang • Tekanan di dalam tin logam akan dan tekanan atmosfera di luar yang lebih tinggi akan menyebabkan tin tersebut dimampatkan . The pressure
inside the metal can decreases
and
the external atmospheric pressure , which is higher
than
will compress
of the water in the glass.
117
06 FIZ Form 4 Ch3 2F.indd 117
Air Water
Panaskan Heat
resultant force caused by the atmospheric pressure
acting on the surface of the cardboard is greater
Tertutup Closed
Air Water
Tekanan atmosfera Atmospheric pressure
The
3
Stim Steam
Tin logam Metal can
•
U N I T
the metal can. © Nilam Publication Sdn. Bhd.
9/10/13 4:33 PM
MODUL • Fizik TINGKATAN 4
Aplikasi tekanan atmosfera/Application of atmospheric pressure Gelas air/Glass of water
Penyedut getah/Rubber sucker
Tekanan atmosfera Atmospheric pressure
Pam getah/Rubber dish Air keluar Water removed
Penyedut minuman Straw Air/Water
• Apabila kita menyedut menggunakan penyedut minuman, tekanan udara di dalam penyedut itu berkurangan . akan When we suck through a straw, the straw
air pressure
penyedut itu. When the sucker is pressed, most of the air behind it is
in the
is lowered .
squeezed out and creates a inside the sucker.
partially vacuumed region
tinggi • Tekanan atmosfera yang pada luar permukaan getah akan menolak penyedut tersebut supaya ia boleh melekat pada permukaan licin itu. higher The atmospheric pressure on the outside surface of the rubber will push the sucker so that it will stick onto the smooth surface.
atmospheric pressure Therefore, the acting on the surface of the water in the glass pushes the water up the straw and into our mouth.
Picagari/Syringe
Separa vakum Partially vacuumed
• Apabila penyedut tersebut ditekan, kebanyakan udara di bahagian belakang ditekan keluar dan ia ruang separa vakum menghasilkan di dalam
tekanan atmosfera • Jadi, yang bertindak pada permukaan air di dalam gelas akan menolak air masuk ke dalam penyedut minuman dan kemudian ke dalam mulut.
U N I T
Tekanan atmosfera Atmospheric pressure
Permukaan licin Smooth surface
Sifon/Siphon Sifon sangat berguna untuk mengeluarkan cecair daripada tangki air atau bekas. A siphon is very useful for removing liquids from a water tank or a fixed container.
3
Piston Piston Tekanan atmosfera Atmospheric pressure
A
h
B
C H D
Cecair Liquid
• Menarik piston ke atas.
Tekanan atmosfera + pgH Atmospheric pressure + pgH
Pulling up the piston.
• Tekanan udara di dalam silinder akan berkurang . The air pressure inside the cylinder will decrease .
• Tekanan atmosfera pada permukaan cecair kemudian menolak cecair tersebut ke dalam picagari. The atmospheric pressure on the liquid surface then pushes the liquid up into the syringe.
© Nilam Publication Sdn. Bhd.
06 FIZ Form 4 Ch3 2F.indd 118
Aras tekanan atmosfera Atmospheric pressure at this level
• Perbezaan paras air
akan menyebabkan tekanan yang berbeza.
The difference in water levels
will cause a difference in pressure.
Tekanan air di C adalah sama dengan tekanan atmosfera di A dan B . The water pressure at C is the same as the atmospheric pressure at
A
and
B .
• Tekanan air di D = tekanan atmosfera + ρgH . The water pressure at D =
• Tekanan air yang tinggi The
higher
atmospheric pressure + ρgH .
menyebabkan air mengalir keluar di D.
water pressure causes water to flow out at D.
kedua-dua hujung tiub tersebut sama • Proses ini akan berterusan sehingga dengan paras air. (tekanan atmosfera sama di kedua-dua hujung tiub.) This process will continue until both ends of tube are equal (atmospheric pressure are equal at both ends of the tube.)
to the water level.
118
9/10/13 4:33 PM
MODUL • Fizik TINGKATAN 4
Mengukur tekanan atmosfera/Measuring atmospheric pressure Barometer Fortin/Fortin barometer • Tekanan atmosfera ialah “h cm merkuri”. Vakum h cm Mangkuk Dish
Vacuum
The atmospheric pressure reading is “h cm mercury”.
Merkuri Mercury
• Jika/If h =
76 cm
,
• Tekanan atmosfera/The atmospheric pressure =
Barometer Aneroid/Aneroid Barometer berkurang • Apabila tekanan atmosfera
76 cm Hg
Skala Scale Penunjuk Pointer
, kotak tersebut akan
mengembang. When the atmospheric pressure
• Apabila tekanan atmosfera akan termampat. When the atmospheric pressure
decreases
Sistem tuas Lever system
, the box will expand.
meningkat increases
, kotak tersebut
Spring Spring
Kotak separa vakum Semi-vacuum box
, the box will compress.
• Pergerakan kotak tersebut dikawal oleh sistem tuas yang disambungkan pada penunjuk. The slight movement of the box is magnified by a lever system which is connected to a pointer.
• Barometer Aneroid boleh digunakan sebagai altimeter untuk pendaki gunung atau di dalam kapal terbang untuk menentukan altitud (ketinggian atas paras laut). The Aneroid Barometer can be used as an altimeter by mountaineers or in an aeroplane to determine its altitude.
3
Mengukur tekanan gas/Measuring gas pressure Manometer/Manometer • Manometer terdiri daripada tiub kaca berbentuk U A manometer consists of a U-shaped
U N I T
yang berisi cecair.
glass tube filled with a liquid.
• Air digunakan di dalam manometer untuk mengukur tekanan gas yang rendah. Water is used in a manometer to measure low gas pressures.
• Sebelah kiri bahagian manometer didedahkan pada atmosfera disalurkan dengan bekalan gas . One arm of the manometer is exposed to the atmosphere
manakala bahagian yang sebelah lagi
whereas another arm is connected to the gas supply .
• Terdapat tiga cara untuk membaca tekanan gas dengan menggunakan manometer apabila pili dibuka. There are three possible methods to read the pressure of a gas by using the manometer when the tap is opened.
i. Pgas = Patm
ii. Pgas = Patm + h cm Hg
Tekanan atmosfera Atmospheric pressure Pili ditutup Tap is closed
Merkuri Mercury
Gas
Tekanan atmosfera Atmospheric pressure Pili dibuka Tap is opened
h
Merkuri Mercury
119
06 FIZ Form 4 Ch3 2F.indd 119
iii. Pgas = Patm – h cm Hg
Tekanan atmosfera Atmospheric pressure
Gas
h
Pili dibuka Tap is opened
Merkuri Mercury
Gas
© Nilam Publication Sdn. Bhd.
9/10/13 4:33 PM
MODUL • Fizik TINGKATAN 4
Tolok Bourdon/Bourdon Gauge
Tiub melengkung diratakan Flattened curved tube
tekanan • Apabila gas disalurkan ke Tolok Bourdon, di dalam tiub yang melengkung itu akan cuba meluruskan tiub tersebut. pressure When the gas supply is connected to a Bourdon Gauge, the in the curved tube will try to straighten it. • Dengan itu penunjuk akan berputar./Hence the pointer will rotate.
•
Magnitud tekanan gas
The
Sistem tuas Lever system
Skala scale
Gas Gas
boleh dibaca pada penimbang tolok itu.
magnitude of the gas pressure
Latihan/Exercises
Penunjuk Pointer
can be read from the scale of the gauge.
(g = 10 m s–2)
1 Tekanan atmosfera ialah 76 cm Hg. Hitung tekanan atmosfera di dalam unit Pa. The atmospheric pressure is 76 cm Hg. Calculate the atmospheric pressure in Pa.
[Ketumpatan merkuri = 1.36 × 104 kg m-3] [Density of mercury = 1.36 × 104 kg m-3]
Penyelesaian/Solution: P = hρg = (0.76 m) × (1.36 × 104 kg m-3) × (10 m s–2) = 1.03 × 105 Pa U N I T
3
2 Rajah menunjukkan sebuah barometer. Ruang vakum telah diisi dengan gas X. Gas X
The diagram shows a barometer. The vacuum space is filled with a gas X.
Berapakah tekanan gas X? What is the pressure of the gas X?
40 cm
[Gunakan tekanan atmosfera = 76 cm Hg]
Merkuri Mercury
[Use atmospheric pressure = 76 cm Hg]
Penyelesaian/Solution: Tekanan gas X
= (76 cm Hg) – (40 cm Hg)
(Pressure of gas X) = 36 cm Hg
3 Rajah menunjukkan barometer merkuri diletakkan di makmal sekolah di mana tekanan atmosferanya adalah 75 cm Hg. The diagram shows a mercury barometer placed in a school laboratory where the atmospheric pressure is 75 cm Hg. 10 cm
Vakum Vacuum
Tiub kaca Glass tube Merkuri Mercury
h
Mangkuk Dish
© Nilam Publication Sdn. Bhd.
06 FIZ Form 4 Ch3 2F.indd 120
120
9/10/13 4:33 PM
MODUL • Fizik TINGKATAN 4
(a) Berapakah nilai h?
What is the value of h?
(b) Berapakah nilai h apabila tiub kaca
What is the value of h when the glass tube is
(i) dinaikkan sebanyak 5 cm?/lifted up by 5 cm? (ii) diturunkan ke dalam mangkuk sedalam 4 cm?
lowered into the dish by 4 cm?
(c) Jika ketumpatan merkuri ialah 1.36 × 104 kg m-3 dan ketumpatan air ialah 1 × 103 kg m-3, tentukan If the density of mercury is 1.36 × 104 kg m-3 and the density of water is 1 × 103 kg m-3, determine
(i) tekanan atmosfera dalam unit Pa./the atmospheric pressure in Pa. (ii) nilai h jika merkuri tersebut digantikan dengan air.
the value of h if the mercury is replaced by water.
(iii) tekanan jika barometer merkuri itu dimasukkan ke dalam air sebanyak 40.0 cm. (Beri jawapan dalam m air.)
the pressure if the mecury barometer is submerged in water at depth of 40.0 cm. (Give your answer in m water.)
Penyelesaian/Solution: (a) 75 cm (b) (i) 75 cm (ii) 75 cm (c) (i) P = hρg = (0.75 m) × (1.36 × 104 kg m–3) × (10 m s–2) = 1.02 × 105 Pa (ii) Tekanan atmosfera, P = hmerkuri × ρmerkuri × g,
U N I T
Atmospheric pressure, P = hmercury × ρmercury × g,
Tetapi Tekanan atmosfera, P = hair × ρair × g, But Atmospheric pressure, P = hwater × ρwater × g, ∴ hmerkuri × ρmerkuri × g = hair × ρair × g hmercury × ρmercury × g = hwater × ρwater × g ∴ 1.02 × 105 Pa = (hair/water) × (1 × 103 kg m–3) × (10 m s–2) ∴ hair/water = 10.2 m
3
(iii) Tekanan = (10.2 + 0.40) m air (tekanan meningkat, jadi panjang h meningkat)
= 10.6 m air Pressure = 10.6 m water
(pressure increases, so the length of h increases)
4 Rajah menunjukkan sebuah manometer merkuri disambungkan dengan bekalan gas. The diagram shows a mercury manometer that is connected to a gas supply.
Hitung tekanan gas yang dibekalkan dalam unit/Calculate the pressure of the gas supply in (a) cm Hg (b) Pa [Ketumpatan merkuri = 1.36 × 104 kg m-3 dan tekanan atmosfera = 76 cm Hg]
[Density of mercury = 1.36 × 104 kg m-3 and atmospheric pressure = 76 cm Hg] Penyelesaian/Solution: (a) Pgas = Patm + h cm Hg = 76 cm Hg + 15 cm Hg = 91 cm Hg (b) Pgas = hρg = (0.91 m) × (1.36 × 104 kg m–3) × (10 m s–2) = 1.24 × 105 Pa
121
06 FIZ Form 4 Ch3 2F.indd 121
15 cm Merkuri Mercury
Gas
© Nilam Publication Sdn. Bhd.
9/10/13 4:33 PM
MODUL • Fizik TINGKATAN 4
3.4
APLIKASI PRINSIP PASCAL
APPLYING PASCAL'S PRINCIPLE
Nyatakan Prinsip Pascal State Pascal's Principle
Prinsip Pascal menyatakan bahawa apabila tekanan yang dikenakan pada cecair yang tertutup, tekanan akan dipindahkan secara seragam ke seluruh bahagian cecair itu. Pascal’s Principle states that when a pressure is applied to an enclosed fluid, the pressure will be transmitted equally throughout the whole fluid.
Eksperimen untuk mengesahkan Prinsip Pascal Experiment to verify Pascal’s Principle
• Apabila piston ditekan pada kelalang sfera kaca, air di dalam jet akan dipancutkan daripada lubang-lubang kecil kelajuan yang sama pada sfera kaca dengan . When the piston is pushed into the round-bottom flask, the jet
Equation
3
Menjelaskan sistem hidraulik Explain a hydraulic system
.
tekanan yang dikenakan pada air dipindahkan secara seragam
• Ini adalah kerana ke seluruh bahagian air.
pressure acting on the water is transferred uniformly
throughout the water.
Dari Rajah 3.4.2, From Diagram 3.4.2., Daya, F1 Force, F1 = Tekanan P1 = Pressure P1 Luas keratan rentas pistonx, A1 Cross-sectional area, A1 of pistonx Mengikut Prinsip Pascal, Tekanan P2 (pada pistonY) = P1 Daya, F2 Tetapi P2 = Luas keratan rentas pistonY, A2 F F ∴ 2 = 1 A2 A1
U N I T
same speed
of water spurts from the holes in the flask with the
This is because the
Persamaan
Jet air Jets of water
From Pascal's Principle, Pressure P2 (at pistonY) = P1 Force, F1 But P2 = Cross-sectional area, A2 of pistonY F F ∴ 2 = 1 A2 A1
Suatu daya input F1, yang kecil telah dikenakan pada piston kecil yang menyebabkan terhasilnya daya output F2 yang besar.
F1
A small input force, F1, is applied to the small pistonInput piston, resulting in a large output force, F2. A
F2 A1
X
• Apabila daya F1, dikenakan pada piston A, tekanan dihasilkan dan disebarkan
M Y
Output piston, B
Cecair Fluid
Rajah 3.4.2/Diagram 3.4.2
secara seragam
ke dalam cecair dan seterusnya kepada piston B. When force, F1, is applied to piston A, a throughout the liquid towards piston B.
A2
pressure
is produced and
transmitted uniformly
• Tekanan di X = Tekanan di Y/Pressure at X = pressure at Y • Tekanan didarabkan dengan luas permukaan piston B akan menghasilkan daya output, F2 yang mengangkat beban M. Pressure multiplied by the surface area of piston B will produce the up load M.
output force, F2
to lift
• Luas keratan rentas piston A adalah lebih kecil daripada luas keratan rentas piston B untuk menghasilkan daya output yang besar . The cross-sectional area of piston A is output force.
© Nilam Publication Sdn. Bhd.
06 FIZ Form 4 Ch3 2F.indd 122
smaller
than that of piston B to produce a
large
122
9/10/13 4:33 PM
MODUL • Fizik TINGKATAN 4
Aplikasi Prinsip Pascal/Application of Pascal’s Principle Brek Hidraulik/Hydraulic Brake • Tekanan yang dikenakan pada pedal brek boleh menghasilkan tekanan yang sama ke semua tayar untuk memberhentikan kereta. Pressure
acting an the brake pedal can transmit an
equal pressure to all wheels simultaneously to stop the car.
daya
• Tekan pedal brek dengan mengenakan
force
Press on the brake pedal by applying a
.
Pressure is transmitted uniformly
• Ia menyebabkan This causes the
•
Geseran The
piston
Spring Spring
pressure on the brake fluid
The piston of the master cylinder applies a
•
Piston Piston Gegendang brek Brake drum
Kasut brek Brake shoe
.
• Piston di silinder utama mengenakan tekanan pada minyak brek . Tekanan bertindak secara seragam
Piston Piston
Pedal brek Brake pedal
Silinder utama Master cylinder Silinder hamba Slave cylinder
.
pada silinder hamba di tayar.
to the slave cylinders at the wheel.
pada tayar menolak kasut brek untuk menekan pada permukaan gegendang brek.
pistons
at the wheels to push the
brake shoes
to press against the surface of the brake drums.
antara brek dan kasut brek menyebabkan kenderaan menjadi perlahan dan akhirnya berhenti.
friction
between the brakes and brake shoes causes the vehicle to slow down and finally stop. U N I T
Ciri-ciri/Characteristics (a) Cecair brek menggunakan The brake fluid used is
oil
minyak
kerana minyak tidak boleh dimampatkan.
because it cannot be compressed.
(b) Cecair itu perlu mempunyai takat didih yang The fluid must have a
high
tinggi
supaya tidak mudah berubah menjadi gas.
3
boiling point so that it does not easily change to gas.
(c) Bahan untuk paip saluran diperbuat daripada keluli kerana ianya kuat dan mampu menghalang daripada kebocoran. The material for the transmission pipe is made from steel because it is strong and can prevent the pipe from leakage.
Faktor mengapa brek tidak berfungsi secara berkesan A factor why the brake does not work effectively
Apabila terdapat gelembung udara di dalam cecair, sebahagian daripada daya yang dikenakan akan digunakan untuk memampatkan gelembung udara tersebut. Ianya akan menambahkan masa untuk tindak balas brek tersebut. When there are air bubbles in the liquid, part of the force applied will be used up in compressing the air bubbles. It will increase
the response time of the brake.
Latihan/Exercises 1 Untuk menghasilkan daya sebanyak 2 500 N ke sistem hidraulik yang berkeluasan 20 m2, hitung jumlah daya yang harus digunakan untuk keluasan 50 cm2.
Penyelesaian/Solution: F1 F2 = A1 A2
To produce a 2 500 N force over a 20 m2 area of a hydraulic system, calculate the magnitude of the force that must be applied to a 50 cm2 area.
F1 = 50 cm2 ×
2 500 N 20 × 104 cm2
= 0.625 N
123
06 FIZ Form 4 Ch3 2F.indd 123
2 500 N F1 = 20 m2 50 cm2
© Nilam Publication Sdn. Bhd.
9/10/13 4:33 PM
MODUL • Fizik TINGKATAN 4
2 Di dalam sebuah sistem hidraulik, suatu piston yang besar mempunyai luas keratan rentas A2 = 200 cm2 dan piston yang kecil pula mempunyai luas keratan rentas A1 = 5 cm2. Jika daya sebanyak 250 N dikenakan pada piston kecil, berapakah daya F yang dikenakan pada piston yang besar? In a hydraulic system, the large piston has a crosssectional area A2 = 200 cm2 and the small piston has cross-sectional area A1 = 5 cm2. If a force of 250 N is applied to the small piston, what is the force, F, on the large piston?
Penyelesaian/Solution:
F1 F2 = A1 A2
250 N F = 5 cm2 200 cm2
The diagram shows a basic hydraulic system which has a small piston and a large piston with crosssectional areas of 0.005 m2 and 0.1 m2 respectively. A force of 20 N is applied to the small piston. 20 N 0.005
U N I T
3
Beban/Load
0.1 m2 Piston kecil Small piston
Piston besar Large piston
Tentukan/Determine (a) tekanan yang disebarkan dalam cecair hidraulik the pressure transmitted in the hydraulic fluid. (b) jisim beban tersebut/the mass of the load. Penyelesaian/Solution:
F = 10 000 N
3 Rajah menunjukkan sistem hidraulik asas yang mempunyai piston kecil dan besar masingmasing dengan luas keratan rentas 0.005 m2 dan 0.1 m2. Daya sebanyak 20 N dikenakan pada piston kecil. 3.5
m2
(a) tekanan yang dipindahkan/the pressure transmitted, 20 N P= = 4 000 N m-2 0.005 m2 mg (b) P = A2 jisim/mass × 10 m s–2 –2 4 000 N m = 0.1 m2 ∴ jisim/mass = 40 kg
APLIKASI PRINSIP ARCHIMEDES
APPLYING ARCHIMEDES’ PRINCIPLE
Definisi tujah ke atas atau daya apung/Definition of upthrust or buoyant force Tujah ke atas atau daya apung yang bertindak pada sesuatu objek bersamaan dengan berat bendalir yang tersesar apabila objek ini terendam dalam berdalir ini sepenuhnya atau separanya. The upthrust or buoyant force acting on an object is equal to the weight of fluid displaced when the object is immersed completely or partly in the fluid.
Menghubungkaitkan daya apung ke atas dengan kehilangan berat/Relate buoyant force to the loss of weight (a) Berat objek di udara ialah berat sebenarnya = 10 N The weight of an object in air is its actual weight = 10 N
(b) Berat diukur apabila objek direndam sepenuhnya di dalam air dan berat ketaranya = 7N The weight measured when the object is immersed totally in water is its apparent weight =
7N
10 N
Udara K Air
Air Water
K
7N
(c) Daya apung menjadikan objek kelihatan lebih ringan . The Buoyant force makes the object seem to be lighter .
K tergantung di udara K hanging in the air
Daya apung Buoyant force
K tenggelam sepenuhnya di dalam air K immersed totally in water
(d) Daya apung bertindak ke arah ke atas ./The buoyant force acts upwards . (e) Daya apung ke atas adalah bersamaan dengan berat ketara yang hilang . The buoyant force is equal to the apparent loss in weight.
(f)
Daya apung
= berat sebenar – berat ketara = 10 N – 7 N = 3 N
Buoyant force
= actual weight – apparent weight = 10 N – 7 N = 3 N
(g) Berat ketara yang hilang = 3 N
© Nilam Publication Sdn. Bhd.
06 FIZ Form 4 Ch3 2F.indd 124
/The apparent loss in weight =
3N
Nota: Tujah ke atas = Daya apung = Daya julangan
124
9/10/13 4:33 PM
MODUL • Fizik TINGKATAN 4
Menghubungkaitkan daya apung dengan berat air yang disesarkan dan isi padu air yang disesarkan Relate buoyant force to the weight of the water displaced and volume of the water displaced
0 N
3
N
N N
2
1
Air disesarkan Water displaced
berat cecair yang disesarkan
(a) Daya apung adalah bersamaan dengan
Buoyant force is equal to the weight of the liquid displaced
(b) Daya apung = 3 N – 1 N = 2 N . 2N Berat air yang disesarkan = Buoyant force =
3N–1N=2N
Weight of the water displaced =
.
.
. .
2N
.
(c) Apabila berat air yang disesarkan itu bertambah, daya apung juga turut As the weight of the water displaced increases, the buoyant force also
26 24 22 20 18 16 14 12 10
(d)
Isi padu air yang disesarkan The
26 24 22 20 18 16 14 12 10
bertambah
increases
.
.
U N I T
3
Isi padu air yang disesarkan Volume of water displaced
bersamaan dengan isi padu objek yang direndam di dalam air.
volume of the water displaced
is equal to the volume of the object immersed in the water.
Prinsip Archimedes/Archimedes’ Principle Prinsip Archimedes menyatakan bahawa apabila suatu objek yang direndamkan sepenuhnya atau sebahagian di dalam suatu bendalir, daya apung (tujah ke atas) adalah sama dengan berat bendalir yang disesarkan oleh objek tersebut.
Archimedes' Principle states that when a body is fully or partially immersed in a fluid, the buoyant force on the object (upthrust) is equal to the weight of the fluid displaced by the object.
125
06 FIZ Form 4 Ch3 2F.indd 125
© Nilam Publication Sdn. Bhd.
9/10/13 4:33 PM
MODUL • Fizik TINGKATAN 4
Persamaan daya apung/Equation of buoyant force (a) daya apung = berat bendalir yang disesarkan/Buoyant force = weight of fluid displaced (b) daya apung = ρVg/buoyant force = ρVg (c) daya apung = berat objek di udara – berat objek di dalam air Buoyant force = weight of object in air – weight of object in water Daya apung dan keapungan/Buoyant force and flotation (a) Jika objek terapung/If the object floats Daya apung = berat objek/Buoyant force = weight of object (b) Jika objek dalam seturus cecair bergerak ke atas/If the object moves upward in a liquid column Daya apung > berat objek/Buoyant force > weight of the object (c) Jika objek dalam seturus cecair bergerak ke bawah/If the object moves downward in a liquid column Daya apung < berat objek/Buoyant force < weight of the object Aplikasi Prinsip Archimedes/Application of Archimedes’ Principle
U N I T
3
Kapal diperbuat daripada besi akan terapung di atas air
Sebuah bot akan tenggelam lebih dalam di sungai daripada air laut
Ship made of steel will float on water
A boat will submerge deeper in the river than in the sea
• Sebuah kapal yang terapung di permukaan laut menyesarkan isi padu air yang sangat besar
• Ketumpatan air sungai adalah lebih daripada ketumpatan air laut.
.
A ship floating on the surface of the sea displaces a very large volume of water
•
Di laut/At sea
Berat
air yang disesarkan adalah sangat daya apung besar, oleh itu yang disesarkan sangat besar pada kapal tersebut. weight
The
Berat
•
sama dengan
kapal adalah bersamaan dengan
daya apung
The
, jadi ia terapung.
The weight of ship is equal to the and therefore it floats.
buoyant force
the ,
• Disebaliknya, sebongkah logam akan menyesarkan isi kecil padu air yang sahaja. On the other hand, a block of steel will displace a small
• Jadi,
volume of water only.
itu adalah lebih ia tenggelam.
buoyant force So the acting on it is than its weight, and therefore it sinks.
bot yang terapung itu adalah berat air yang disesarkan .
buoyant force
on the floating boat is equal to
weight of water displaced
.
• Semakin rendah ketumpatan air, semakin besar isi padu air yang disesarkan. The lower the density of the water, the volume of water displaced.
•
daya apung
yang bertindak pada logam kecil daripada beratnya; maka
Rajah (b)/Diagram (b)
Daya apung
•
acting on the ship is also
than
Di sungai/In the river
Rajah (a)/Diagram (a)
of water displaced is very large,
buoyant force so the very large.
lower
The density of the fresh river water is that of sea water.
.
kecil
Ketumpatan =
jisim isi padu
Density =
larger
the
mass volume
• Apaila bot terapung di permukaan laut, isipadu air laut yang tersesar adalah kecil kerana ketumpatan air laut adalah besar.
When the boat is at sea, the volume of sea water displaced is small because the density of sea water is large.
smaller
• Jadi bot akan tenggelam lebih di dalam air sungai berbanding dengan air laut. So the boat will submerge deeper in river water compared to sea water.
© Nilam Publication Sdn. Bhd.
06 FIZ Form 4 Ch3 2F.indd 126
126
9/10/13 4:33 PM
MODUL • Fizik TINGKATAN 4
Sesebuah kapal akan tenggelam lebih dalam ke dalam air jika berat yang lebih diletakkan di dalamnya. The hull of the ship will sink deeper in the water if extra weight is put into it.
• Jika berat beban bertambah, isi padu air yang disesarkan juga turut bertambah . If the weight of load increases, the volume of the water displaced will also increase
Badan kapal Hull
.
daya apung
• Apabila isi padu air yang disesarkan bertambah, bertambah . As the volume of the water displaced increases, the
juga akan
Kapal A/Ship A
buoyant force also increases
.
• Badan kapal tersebut perlu tenggelam lebih dalam untuk menyesarkan lebih banyak air supaya daya apung yang lebih besar dihasilkan untuk menampung berat yang lebih.
Kapal B (muatan yang lebih ) Ship B (extra load)
The hull of the ship must sink lower to displace more water so that there is larger buoyant force
to support the extra weight.
Peranan simbol Plimsoll pada kapal laut The purpose of the Plimsoll symbol on a ship
TF F
TF F T S F W WNA
T S W WNA
: : : : : : :
Air tropika tawar/Tropical fresh water Air tawar/Fresh water Air tropika masin/Tropical salt water Lautan musim panas/Salt water in summer Air tawar/Fresh water Lautan musim sejuk/Salt water in winter Musim sejuk di Lautan Atlantik Utara/Winter in North Atlantic
lokasi
• Ketumpatan air laut berbeza mengikut
dan
location
The density of sea water varies with
and
musim season
U N I T
3
. .
• Untuk memastikan kapal tersebut diisi dengan muatan yg sesuai beratnya, tanda garis plimsoll pada kapal berfungsi sebagai penanda. To ensure that a ship is loaded within safe limits, the plimsoll line marked on the body of the ship acts as a guide.
Ciri-ciri sebuah kapal Characteristics of a ship
(a) Bentuk 'streamline' untuk mengurangkan rintangan air
Streamlined shape to reduce water resistance
(b) Logam mempunyai kekuatan
The
high
tinggi
untuk menahan tekanan air yang tinggi
strength of metal withstands high water pressure
lebar
(c) Luas keratan rentas badan kapal adalah
The cross-sectional area of the hull is
wide
supaya lebih stabil so that the ship is more stable
(d) Ruang udara di dalam badan kapal adalah (membolehkan kapal terapung)
besar
untuk menghasilkan daya apung yang besar
high
in order to produce a large buoyant force (which
The volume of space in the hull of the ship is enables the ship to float)
127
06 FIZ Form 4 Ch3 2F.indd 127
© Nilam Publication Sdn. Bhd.
9/10/13 4:33 PM
MODUL • Fizik TINGKATAN 4
Kapal selam/Submarine Kapal selam tenggelam/Submarine sinks • Kapal selam mempunyai tangki-tangki balast yang besar yang digunakan untuk mengawal dan kedalaman kapal selam tersebut dari permukaan laut. A submarine has large ballast tanks, which are used to control its
kedudukan
position and depth from the surface of the sea.
• Apabila tangki diisi dengan air, berat kapal selam tersebut adalah lebih besar daripada daya apung. When the ballast tanks are filled with water, the
• Kapal selam tersebut akan
weight
of the submarine is larger than the buoyant force.
tenggelam ./The submarine will
sink .
Kapal selam timbul semula/Submarine rise up • Tangki balast diisi dengan udara (semua air dikeluarkan)./The ballast tanks are filled with air (all water is removed). Permukaan laut • Berat kapal selam tersebut adalah lebih kecil Tangki balast kosong Sea surface
daripada daya apung.
Ballast tank empty
The weight of the submarine is smaller than the buoyant force.
timbul semula .
• Kapal selam akan
will rise .
The submarine
U N I T
3
Udara Air
Tangki balast penuh dengan air Ballast tank full of water
Air Water
Ciri-ciri kapal selam/Characteristics of a submarine (a) Bentuk 'streamline' untuk mengurangkan rintangan air/Streamline shape to reduce water resistance (b) Bahan yang tebal dan kuat untuk menahan tekanan tinggi kerana tekanan meningkat mengikut kedalaman Thick and
strong materials to withstand high pressure because pressure increases with depth
tangki-tangki balast
(c) Mempunyai
ballast tanks
Has
periscopes
Has
to sink or float the submarine
periskop untuk memantau objek di luar permukaan air
(d) Mempunyai
untuk membolehkan kapal selam tenggelam atau terapung
to observe objects above the water surface
(e) Dilengkapi tangki oksigen untuk
pernafasan
/Equipped with oxigen tanks for
respiration
Belon udara panas/Hot air balloons • Belon ditindak oleh dua daya utama ( daya apung dan
berat
belon).
Balloon acted by two main forces ( buoyant force and weight of the balloon).
• Ketumpatan gas helium adalah
lebih kecil
smaller
The density of helium gas is
dari ketumpatan udara di sekeliling.
than the density of the surrounding air.
• Sebuah belon udara panas menyesarkan isi padu udara yang A hot air balloon displaces a
Daya apung
•
The
• Apabila daya apung
lebih besar
jumlah berat total weight
© Nilam Publication Sdn. Bhd.
06 FIZ Form 4 Ch3 2F.indd 128
.
of air.
is equal to the weight of the air displaced.
When the buoyant force is
When the
besar
Daya apung Upthrust/Buoyant force
adalah sama dengan berat udara yang disesarkan.
buoyant force
• Apabila
large volume
Daya apung Upthrust/ Buoyant force
greater
Berat Weight
Berat Weight
daripada berat belon tersebut, belon akan naik ke atas. than the total weight of the balloon, the balloon will rise.
belon sama dengan daya apung, ia akan kekal terapung di udara. of the balloon is equal to the buoyant force, it will remain floating in the air.
128
9/10/13 4:33 PM
MODUL • Fizik TINGKATAN 4
Ciri-ciri belon udara panas/Characteristics of hot air balloons (a) Belon besar – untuk menghasilkan daya apung yang lebih besar // meningkatkan isi padu udara yang disesarkan
bigger
Large balloon – to produce a
buoyant force // increase the volume of air displaced
(b) Gunakan pembakar yang lebih – untuk menghasilkan api yang belon dengan lebih cepat
Use more burners – to produce
bigger
lebih besar // memanaskan gas di dalam
flames // heat up the gas in the balloon faster
(c) Nilon sintetik – lebih ringan, jisim kecil, kuat dan kalis udara
Synthetic nylon – lighter, small mass, strong and air proof material
mengurangkan
(d) Suhu udara yang tinggi dalam belon – untuk
High temperature of the air in the balloon – to
reduce
ketumpatan dan berat udara di dalam belon
density and weight of the air in the balloon
(e) Bentuk sfera di atas – untuk mengurangkan rintangan udara , supaya dapat naik ke atas dengan lebih cepat
Spherical top shape – to reduce
air resistance , so that can move up faster
Hidrometer/Hydrometer Skala Scale
Batang halus Thin stem
• Di dalam cecair yang lebih kecil ketumpatannya , hidrometer akan tenggelam lebih dalam
Kelalang kaca dipenuhi udara Air-filled glass bulb
In a liquid of
, the hydrometer is
more submerged Butiran plumbum Lead shots
• Hidrometer adalah satu peralatan untuk mengukur ketumpatan relatif cecair seperti susu dan asid di dalam bateri kereta
Characteristics of a hydrometer
(a) Letakkan bebola plumbum di dalam kelalang kaca untuk memastikan hidrometer dalam keadaan tegak
kecil (b) Gunakan batang yang lebih mendapatkan selang yang lebih besar
• Kelalang kaca yang besar penuh dengan udara itu digunakan untuk menyesarkan lebih cecair dan ianya menyebabkan daya apung (tujah ke atas) The large air-filled glass bulb is used to displace more liquid and it causes the
buoyant force
dan batang
supaya hidrometer tersebut tidak tenggelam sepenuhnya dalam cecair rendah yang berketumpatan
more sensitive
menjadikan hidrometer terapung
besar
(c) Gunakan kelalang yang lebih panjang yang
It has long narrow neck with a scale marked on it which makes the hydrometer
untuk
Use a small stem to have bigger intervals on the scale
It has a glass bulb which contains some lead shots to make it float upright
• Ia mempunyai leher yang panjang dan sempit dengan tanda pengukur padanya menyebabkan hidrometer menjadi lebih sensitif
3
lead shots Put in some in the glass bulb to ensure the hydrometer is upright
relative density of liquids such as milk and acid in accumulator
• Ianya mempunyai kelalang kaca yang mengandungi butiran plumbum untuk membuat hidrometer itu terapung ke atas
U N I T
Ciri-ciri sebuah hidrometer
A hydrometer is an instrument used to measure the
bertambah
lower density
Use a
big
bulb and
longer
stem so that the
hydrometer does not sink all the way in a density liquid
(d) Diperbuat daripada mudah terhakis
(upthrust) to
Made from
glass
kaca
low
supaya ianya tidak
so it does not corrode easily
increase , hence the hydrometer floats
129
06 FIZ Form 4 Ch3 2F.indd 129
© Nilam Publication Sdn. Bhd.
9/10/13 4:33 PM
MODUL • Fizik TINGKATAN 4
Latihan/Exercises
(g = 10 m s–2)
1 Sebuah blok yang keras digantung di udara dengan menggunakan benang dari neraca spring. Neraca spring tersebut memberikan bacaan 65 N. Apabila blok itu tenggelam sepenuhnya di dalam air, neraca spring memberikan bacaan 30 N. A solid block is suspended in air by a thin thread from a spring balance. The spring balance gives a reading of 65 N. When the block is completely submerged in water, the spring balance gives a reading of 30 N.
(a) Tentukan daya apung yang dikenakan oleh air pada blok tersebut. Determine the buoyant force exerted by the water on the block.
(b) Berapakah berat air yang disesarkan oleh blok tersebut? What is the weight of water displaced by the block?
(c) Tentukan isi padu air yang disesarkan oleh blok. [Ketumpatan air = 1 000 kg m-3] Determine the volume of water displaced by the block. [Density of water = 1 000 kg m-3]
Penyelesaian/Solution: (a) Daya apung/Buoyant force = berat di udara/weight in the air – berat ketara/apparent weight = 65 N – 30 N = 35 N (b) Berat air yang disesarkan/Weight of water displaced = daya apung/buoyant force = 35 N (c) Daya apung/Buoyant force = mg = (ρV)g Perhatian/Note: 35 N = (1 000 kg m–3)(V)(10 m s–2) ρ = m 35 N V 3 = 0.0035 m V = m = ρV (1 000 kg m–3 × 10 m s–2)
Blok tenggelam sepenuhnya, maka isi padu blok = isi padu air yang disesarkan = 0.0035 m3 U N I T
3
The block is completely submerged, so volume of the block = volume of water displaced = 0.0035 m3
2 Rajah menunjukkan sebiji belon berjisim 200 kg sedang terapung pada kedudukan pegun di udara. Berapakah daya apung belon itu? The diagram shows a balloon of mass 200 kg floating in a stationary position in the air. What is the buoyant force on the balloon?
Tanah Ground
Penyelesaian/Solution: Apabila sesuatu objek terapung dalam keadaan pegun, daya apungan = berat objek When an object floats in a stationary position, the buoyant force = weight of object
Oleh itu, F(daya apung)/Therefore, F(buoyant force) = F = F = =
W mg (200 kg) (10 m s–2) 2 000 N
3 Suatu jasad mempunyai berat 20 N di udara dan 15 N di dalam cecair. Jika isi padu cecair yang tersesar adalah 5 × 10-4 m3, berapakah ketumpatan cecair tersebut? A body has a weight of 20 N in air and 15 N in a liquid. If the volume of the liquid displaced is 5 × 10-4 m3, what is the density of the liquid?
Penyelesaian/Solution: Daya apung/Buoyant force = Berat sebenar/Actual weight – Berat ketara/Apparent weight = 20 N – 15 N = 5 N Jadi, daya apung/Hence, buoyant force = Berat cecair yang disesarkan/Weight of the liquid displaced = 5 N Jisim cecair yang disesarkan/Mass of the liquid displaced = 0.5 kg Oleh itu/Then, ρ = m Perhatian/Note: V Berat/Weight = mg 0.5 kg -3 5 N = m(10 kg m–2) ρ = = 1 000 kg m 5 × 10-4 m3 m = 0.5 kg
© Nilam Publication Sdn. Bhd.
06 FIZ Form 4 Ch3 2F.indd 130
130
9/10/13 4:33 PM
MODUL • Fizik TINGKATAN 4
4 Sebuah blok logam mempunyai isi padu 0.5 m3 diikat pada tali. Blok itu ditenggelamkan di dalam air. Berapakah tegangan tali itu? [ketumpatan blok logam = 8 × 103 kg m-3, ketumpatan air = 1 × 103 kg m-3], A metal block with a volume 0.5 m3 is tied to a string. The block is immersed in water. What is the tension of the string? [density of metal block = 8 × 103 kg m-3, density of water = 1 × 103 kg m-3]
Penyelesaian/Solution: Daya apung + Tegangan = Berat blok di udara
ρL = VL = ρa = VL =
Buoyant force + Tension = Weight of block in air
Tegangan = Berat blok di udara – Daya apung Tension = Weight of block in air – Buoyant force T (tegangan/tension) = ρLVLg – ρaVag
Ketumpatan logam/Density of metal Isi padu blok logam/Volume of metal block Ketumpatan air/Density of water Isi padu air tersesar/Volume of water displaced
= (8 000 kg m–3)(0.5 m3)(10 m s–2) – (1 000 kg m–3)(0.5 m3)(10 m s–2) = 35 000 N
5 Rajah menunjukkan satu tabung uji berjisim 0.012 kg dengan diameter seragam dan luas keratan rentas 4 × 10-4 m2, dan diisi dengan pasir, supaya ianya terapung menegak di dalam bikar yang berisi air dengan kedalaman 0.08 m. [Ketumpatan air ialah 1 000 kg m-3] The diagram shows a test-tube of mass 0.012 kg with uniform diameter and cross-sectional area 4 × 10-4 m2, and it is filled with sand, so that it floats vertically in a beaker containing water at a height of 0.08 m. [Density of water is 1 000 kg m-3]
Air Water Pasir Sand
0.08 m Bikar Beaker
Tentukan/Determine (a) daya apung terhadap tabung uji. the buoyant force on the test tube.
U N I T
(b) berat pasir di dalam tabung uji. the weight of the sand in the test tube.
3
Penyelesaian/Solution:
berat air tersesar/weight of water displaced = mg = (ρV)g (1 000 kg m–3) × (0.08 m) (4 × 10-4) m2 × 10 m s–2 = 0.32 N berat tabung uji/weight of test tube + berat pasir/weight of sand Daya apung/buoyant force – berat tabung uji/weight of test tube = 0.32 N – (0.012 kg) (10 m s–2) = 0.32 N – 0.12 N = 0.2 N
aya apung/Buoyant force = D = Daya apung/Buoyant force = Berat pasir/Weight of sand =
6 Sebuah objek mempunyai isi padu 5 × 10-4 m3 digantung pada neraca spring di udara. Bacaan pada neraca spring ialah 8 N. Jika objek itu ditenggelamkan sepenuhnya ke dalam cecair yang berketumpatan 600 kg m-3, tentukan bacaan neraca spring sekarang. An object with a volume of 5 × 10-4 m3 is hung from a spring balance in the air. The reading of the spring balance is 8 N. If the object is immersed fully in a liquid with a density of 600 kg m-3, determine the reading of the spring balance now.
Penyelesaian/Solution:
Jisim cecair yang disesarkan/Mass of liquid displaced = Vcecair/liquid × ρcecair/liquid m =ρ m = 5 × 10-4 m3 × 600 kg m-3 v = 0.3 kg ∴Berat cecair yang disesarkan/Weight of liquid displaced = (0.3 kg) (10 m s–2) = 3.0 N Jadi, daya apung/Hence, buoyant force = Berat cecair yang disesarkan/Weight of liquid liquid displaced = 3 N Jadi, daya apung = Berat sebenar objek di udara – Berat ketara objek (di dalam cecair)
Hence, buoyant force = Real weight of object in air – Apparent weight of the object (in the liquid)
3 N = 8 N – Berat ketara/Apparent weight ∴ Berat ketara = Bacaan neraca spring bagi objek dalam cecair = 5 N
Apparent weight = Reading of spring balance for object in liquid = 5 N
131
06 FIZ Form 4 Ch3 2F.indd 131
© Nilam Publication Sdn. Bhd.
9/10/13 4:33 PM
MODUL • Fizik TINGKATAN 4
MEMAHAMI PRINSIP BERNOULLI
3.6
UNDERSTANDING BERNOULLI’S PRINCIPLE
Prinsip Bernoulli/Bernoulli’s Principle Prinsip Bernoulli menyatakan bahawa tekanan dalam bendalir berkurang apabila laju bendalir tersebut bertambah. Bernoulli’s Principle states that the pressure of a fluid decreases as the speed of the fluid increases.
Aktiviti yang menerangkan daya paduan yang wujud akibat tekanan yang berbeza pada bendalir Activities that explain that a resultant force exists due to a difference in fluid pressure
Aktiviti/Activity 1
Aktiviti/Activity 2 Kedudukan akhir Final position
Udara ditiup Blowing air
X
Kertas Paper
Z
Kedudukan asal Original position
U N I T
3
(a) Apabila udara ditiup kuat merentasi sebelah atas permukaan sehelai kertas, didapati bahawa kertas terangkat ke atas itu akan
(a) Apabila udara ditiup dengan kuat dengan penyedut minuman, kedua-dua bola ping pong akan bergerak mendekati antara satu sama lain
When the air is blown across the surface of a piece of paper, the paper
Benang String Bola ping pong Y Ping-pong ball Penyedut minuman Straw Udara ditiup Blowing air
When air is blown hard through the straw, the two ping-
moves up
pong balls will
(b) Ini berlaku kerana udara bergerak pada laju yang tinggi, maka tekanan menjadi rendah di
move closer to each other
(b) Udara bergerak pada halaju yang tinggi di antara bola-bola tersebut, maka tekanan menjadi rendah
sebelah atas kertas
The air moves at a very high velocity between the balls,
This happens because as the air above the paper moves at a very high velocity, the pressure decreases above the paper
(c) Mengikut Prinsip Bernoulli, tekanan udara yang bergerak menjadi rendah apabila laju udara tinggi
and so the pressure becomes low
(c) Mengikut Prinsip Bernoulli, tekanan pada udara yang bergerak berkurang apabila laju udara bertambah . According to Bernoulli’s Principle, the pressure of the
According to Bernoulli’s Principle, the pressure of the
moving air decreases as the speed of the air increases
moving air decreases as the speed of the air increases
(d) Tekanan atmosfera yang bertindak di sebelah bawah kertas adalah lebih tinggi daripada tekanan The atmospheric pressure which acts at the bottom of
tekanan udara di Z di antara bola ping pong menyebabkan perbezaan pada tekanan .
higher the paper is surface of the paper.
balls at X and Y is higher than the air pressure at Z
udara di sebelah atas permukaan kertas tersebut.
The atmospheric pressure which acts on the ping-pong
than the air pressure at the top
(e) Terdapat perbezaan tekanan, maka daya paduand yang menghala ke atas akan menolak kertas tersebut ke atas.
between the balls causing a difference in pressure .
(e) Daya paduan dihasilkan dan menolak dua biji bola ping pong bergerak mendekati antara satu sama lain.
There is a difference in pressure, so a resultant force acting upwards will push the paper up.
© Nilam Publication Sdn. Bhd.
06 FIZ Form 4 Ch3 2F.indd 132
.
(d) Tekanan atmosfera yang bertindak pada bola ping pong di X dan Y adalah lebih tinggi daripada
resultant force A is produced and pushes the ping-pong balls to move closer to each other.
132
9/10/13 4:33 PM
MODUL • Fizik TINGKATAN 4
Aktiviti/Activity 3 Udara ditiup/Blowing air
Aktiviti/Activity 4 (1) P
Aliran air Water flow Corong turas Filter funnel
A B
Bola ping pong Ping-pong ball
Halaju rendah Tekanan tinggi Low velocity High pressure
C
Kedudukan asal/Original position
(a) Apabila udara ditiup lebih kuat, bola tidak akan jatuh. When the air is blown harder, the ball does not fall down.
(b) Ini kerana aliran udara pada halaju tinggi yang antara bola dan corong turas, jadi tekanan menjadi rendah di kawasan A dan B.
R
Q Halaju sederhana Tekanan sederhana Moderate velocity Moderate pressure
• Tekanan bendalir berkurang secara dalam tiub. The pressure of fluid decreases
• Halaju bendalir The velocity of fluid
Halaju tinggi Tekanan rendah High velocity Low pressure
seragam
uniformly
bertambah
Air Water
dari P ke R
from P to R in the tube.
dari P ke R.
increases
from P to R.
• PP > PQ > PR , VP < VQ < VR (2)
This is because the air moves at a very high
velocity between the ball and the wall of the filter funnel, the pressure becomes
low
Aliran air Water flow
in regions A and B.
(c) Mengikut Prinsip Bernoulli, tekanan udara yang bergerak menjadi rendah apabila kelajuan udara meningkat. According to Bernoulli’s Principle, the pressure of the moving air decreases as the speed of the air increases.
(d) Tekanan atmosfera di kawasan C yang lebih tinggi daripada tekanan udara di kawasan A dan B menyebabkan perbezaan tekanan.
Air Water Halaju sederhana Tekanan sederhana Moderate velocity Moderate pressure
Q Halaju rendah Halaju tinggi Tekanan tinggi Tekanan rendah Low velocity High velocity High pressure Low pressure
• Bahagian tengah Q adalah paling sempit, jadi halaju pada Q adalah paling tinggi dan tekanannya adalah paling rendah .
U N I T
3
The middle part of Q is the narrowest, so the velocity at Q is the
highest
and its pressure is the
lowest .
• PP > PR > PQ , VP < VR < VQ (3)
Halaju rendah Tekanan tinggi Low velocity High pressure
The atmospheric pressure in region C, being higher than the air pressure in region A and B.
(e)
R
P
Aliran udara Air flow
Halaju tinggi Tekanan rendah High velocity Low pressure
Halaju sederhana Tekanan sederhana Moderate velocity Moderate pressure
Q
R
P
Daya paduan
ke atas akan terhasil dan menahan bola tersebut daripada jatuh ke bawah. resultant force A is produced upwards and holds the ball thus preventing it from falling down.
Air Water
• Bahagian tengah Q adalah paling sempit, jadi halaju pada Q adalah paling tinggi dan tekanannya adalah paling rendah . The middle part of Q is the narrowest, so the velocity at Q is the
highest
and its pressure is the
lowest .
• PP > PR > PQ , VP < VR < VQ
133
06 FIZ Form 4 Ch3 2F.indd 133
© Nilam Publication Sdn. Bhd.
9/10/13 4:33 PM
MODUL • Fizik TINGKATAN 4
Aplikasi Prinsip Bernoulli Applications of Bernoulli’s Principle
Aerofoil/Aerofoil (a) Rajah di sebelah menunjukkan bentuk aerofoil bagi keratan rentas sayap kapal terbang. The figure here shows an aerofoil shape for a cross-sectional area of the wing of an aeroplane.
Laju udara lebih tinggi, tekanan rendah Speed of air is higher, lower pressure
(b) Ia kelihatan aliran udara lebih panjang di sebelah permukaan atas daripada di bawah dan menyebabkan halaju yang lebih tinggi pada permukaan atas tetapi atas.
tekanan yang lebih rendah
di
It will be seen that the path of air is longer over the upper surface than the lower, and therefore the speed is greater on the upper surface but the
pressure is lower
Aerofoil Aerofoil Laju udara lebih rendah, tekanan tinggi Speed of air is lower, higher pressure
at the upper surface.
(c) Daya angkat (daya paduan) dihasilkan oleh perbezaan tekanan di antara kedua-dua permukaan, yang membantu kapal terbang untuk berlepas (mula naik ke udara). lift (resultant force) is produced by the difference in pressure The plane to take off. U N I T
3
between the two surfaces, which helps the
(d) Selain digunakan di kapal terbang, aerofoil juga digunakan pada kereta lumba. Dalam kes ini, aerofoil diterbalikkan, dan daya ke bawah membantu menstabilkan kereta pada kelajuan yang tinggi. In addition to its use in airplanes, aerofoil is also used in racing cars. In this case, the aerofoil is upside down and the downward force helps to stabilise the car at high speeds.
Penyembur racun serangga Insecticide spray
(a) Apabila omboh ditolak, udara akan dipaksa keluar melalui pada kelajuan yang tinggi dan menyebabkan tekanan udara menjadi rendah.
Omboh Jet udara Jet of air Piston
When the piston is pushed, a jet of air is forced out through the jet of gas at a
high
(b) Mengikut Prinsip Bernoulli, tekanan udara yang bergerak itu berkurang apabila laju udara meningkat. According to Bernoulli’s Principle, the pressure of the moving air decreases
(c)
Tiub logam sempit Narrow metal tube
speed which causes the air pressure to be low.
Cecair racun serangga Insect poison liquid
as the speed of the air increases.
Tekanan atmosfera
di dalam bekas racun serangga adalah lebih tinggi daripada tekanan di luar menyebabkan wujudnya perbezaan tekanan . The atmospheric pressure in the insecticide container is higher than the pressure outside, thus causing the difference in pressure .
(d) Ini menghasilkan daya paduan yang menolak cecair racun serangga ke atas melalui tiub logam yang sempit. This produces a
© Nilam Publication Sdn. Bhd.
06 FIZ Form 4 Ch3 2F.indd 134
resultant force
which pushes the insecticide up through the narrow metal tube.
134
9/10/13 4:33 PM
MODUL • Fizik TINGKATAN 4
Penunu Bunsen Bunsen burner
(a) Apabila gas mengalir keluar melalui jet X dengan halaju yang tinggi , tekanan di X di dalam penunu Bunsen menjadi rendah (Prinsip Bernoulli). When a gas jet flows out from the nozzle, X, with high velocity, the pressure at X in the Bunsen burner becomes low (Bernoulli’s Principle).
Tekanan atmosfera
(b) di luar yang lebih tinggi akan menyebabkan udara disedut ke dalam liang udara dan dicampur dengan gas tersebut untuk pembakaran gas yang lebih sempurna.
Udara disedut masuk Air sucked in
X
Jet gas Gas jet Gas masuk Gas in
A higher external atmospheric pressure will cause air to be sucked into the air hole and be mixed with the gas for a more complete combustion of the gas.
(c)
Tekanan atmosfera adalah lebih tinggi daripada tekanan di dalam jet gas tersebut menyebabkan wujudnya perbezaan tekanan . The
atmospheric pressure difference in pressure
is higher than the pressure inside the jet of gas and this causes a
.
U N I T
3
135
06 FIZ Form 4 Ch3 2F.indd 135
© Nilam Publication Sdn. Bhd.
9/10/13 4:33 PM
MODUL • Fizik TINGKATAN 4
UNIT
4
HABA HEAT
4.1 MEMAHAMI KESEIMBANGAN TERMA/UNDERSTANDING THERMAL EQUILIBRIUM • Menerangkan keseimbangan terma/Explain thermal equilibrium • Menerangkan bagaimana termometer cecair-dalam-kaca bekerja Explain how a liquid–in-glass thermometer works
4.2 MEMAHAMI MUATAN HABA TENTU/UNDERSTANDING SPECIFIC HEAT CAPACITY
• Definisi muatan haba tentu (c)/Define specific heat capacity (c) Q • Menyatakan/State that c = mθ • Menyelesaikan masalah berkaitan muatan haba tentu/Solve problems involving specific heat capacity • Menghuraikan aplikasi muatan haba tentu/Describe applications of specific heat capacity • Menentukan muatan haba tentu cecair/Determine the specific heat capacity of a liquid • Menentukan muatan haba tentu pepejal (blok aluminium atau blok kuprum) Determine the specific heat capacity of a solid (aluminium block or copper block)
4.3 MEMAHAMI HABA PENDAM TENTU/UNDERSTANDING SPECIFIC LATENT HEAT
• Menyatakan bahawa pemindahan haba semasa perubahan fasa tidak melibatkan perubahan suhu State that transfer of heat during a change of phase does not cause a change in temperature
U N I T
4
• Mentakrifkan haba pendam tentu (L)/Define specific latent heat (L) Q • Nyatakan L/State that L = m • Menentukan haba pendam tentu pelakuran/Determine the specific latent heat of fusion • Menentukan haba pendam tentu pengewapan/Determine the specific latent heat of vaporisation • Menyelesaikan masalah yang melibatkan haba pendam tentu/Solve problems involving specific latent heat
4.4 MEMAHAMI HUKUM-HUKUM GAS/UNDERSTANDING THE GAS LAWS
• Menerangkan tekanan, suhu dan isi padu gas dari segi kelakuan molekul gas Explain gas pressure, temperature and volume in terms of behaviour of gas molecules
• Menentukan hubungan antara tekanan dan isi padu pada suhu malar suatu gas berjisim tetap iaitu PV = malar
Determine the relationship between pressure and volume at constant temperature for a fixed mass of gas i.e. PV = constant
V • Menentukan hubungan antara isi padu dan suhu pada tekanan malar bagi jisim tetap gas iaitu T = malar Determine the relationship between volume and temperature at constant pressure for a fixed mass of gas i.e. V = constant T
• Menentukan hubungan antara tekanan dan suhu pada isi padu malar bagi suatu gas berjisim tetap P = pemalar iaitu T Determine the relationship between pressure and temperature at constant volume for a fixed mass of gas i.e. P = constant T • Menerangkan sifar mutlak/Explain absolute zero
• Jelaskan skala mutlak/skala Kelvin bagi suhu Explain the absolute/Kelvin scale of temperature
• Menyelesaikan masalah yang melibatkan tekanan, suhu dan isi padu suatu gas berjisim tetap Solve problems involving pressure, temperature and volume of a fixed mass of gas
© Nilam Publication Sdn. Bhd.
07 FIZ Form 4 Ch4 2F.indd 136
136
9/10/13 4:36 PM
MODUL • Fizik TINGKATAN 4
4.1
MEMAHAMI KESEIMBANGAN TERMA
UNDERSTANDING THERMAL EQUILIBRIUM
Haba (Q)/Heat (Q)
Haba, Q/Heat, Q: • Haba adalah suatu bentuk tenaga Heat is a form of energy
• Haba boleh memanaskan menyejukkan objek
atau
Suhu, T/Temperature, T: • Definisi/Definition: Ditakrifkan sebagai darjah kepanasan objek
Heat can heat up or cool down an object
Is defined as a degree of hotness of an object
• Untuk mengetahui suatu objek panas atau sejuk, ukur suhu objek tersebut To know whether an object is hot or cool, measure the temperature of the object
• Haba mengalir dari kawasan panas ke kawasan sejuk Heat flows from a hot region to a cool region
• Haba boleh mengubah keadaan fizikal bahan Heat can change the physical state of a substance
• Unit S.I.: S.I. Unit:
Kelvin(K) Kelvin(K)
• Menukar unit suhu darjah celsius (°C) kepada Kelvin (K) Change unit of temperature from degree celcius (°C) to kelvin (K)
• θ ºC = (θ + 273) K
Contoh/Example: • Perubahan pepejal kepada cecair Solid changes to liquid
• Unit S.I. bagi haba: Joule(J) S.I. Unit of heat: Joule(J)
Alat untuk mengukur suhu ialah
termometer
The measuring instrument to measure temperature is
thermometer
Jenis-jenis termometer Types of thermometers: (a) Cecair dalam termometer cecair-dalam-kaca
Liquid-in-glass thermometer
(b) Termometer termogandingan
Thermocouple thermometer
(c) Termometer rintangan
Resistance thermometer
(d) Termometer gas
Gas thermometer
Haba membolehkan sesuatu objek menukar suhu atau mengubah keadaan fizikal: Heat makes an object to change its temperature or change its physical state: (a) Jika keadaan fizikal tidak berubah, suhunya akan berubah If the physical state is unchanged, the temperature changes (b) Jika suhu tetap, keadaan fizikal akan berubah If the temperature is constant, the physical state changes Pertukaran Unit Suhu: Change Unit of Temperature: θ ºC = (θ + 273) K Contoh: Example: (i) 0ºC = = (ii) 27ºC = = (iii) 100ºC = =
(0 + 273) K 273 K (27 + 273) K 300 K (100 + 273) K 373 K
Ciri-ciri fizikal (sifat termometri) termometer yang berubah dengan haba / suhu Physical characteristics (Thermometric properties) of thermometer that change with heat/ temperature: (a) Termometer berjenis cecair-dalam-kaca – panjang / isi padu turus merkuri berubah dengan haba Liquid-in-glass thermometer – the length / volume of mercury column changes with heat (b) Termometer termogandingan – d.g.e berubah terhadap haba Thermocouple thermometer – the e.m.f/current changes with heat (c) Termometer rintangan – rintangan berubah terhadap haba. Resistance thermometer – the resistance changes with heat (d) Termometer gas – tekanan gas berubah terhadap haba. Gas thermometer – the gas pressure changes with heat.
U N I T
4
Jenis termometer cecair-dalam-kaca Liquid-in-glass thermometer: (a) Termometer merkuri/Mercury thermometer (b) Termometer alkohol/Alcohol thermometer
137
07 FIZ Form 4 Ch4 2F.indd 137
© Nilam Publication Sdn. Bhd.
9/10/13 4:36 PM
MODUL • Fizik TINGKATAN 4 Merkuri digunakan dalam termometer kerana: Mercury is used in the thermometer because:
Penentukuran Termometer/Themometer Calibration: Termometer Merkuri/Mercury Thermometer:
Ciri-ciri Penerangan Characteristics Explanation
Termometer merkuri Mercury thermometer
Takat ais Ice point
Takat stim Steam point
Ais Ice
Air dari ais yang melebur Water from melting ice
Stim Steam
Kaki retort Retort stand Xxxxxxxxxxxxxxxxxx
Kasa dawai Wire gauze
Legap Opaque
Senang untuk mengambil bacaan Easy to take a reading
Mengembang seragam dengan haba Expands uniformly with heat
Skala seragam Uniform scale
Daya lekitan yang tinggi High cohesive force
Tidak membasah tiub dan tidak melekat pada dinding kaca Does not wet the tube and does not stick to the glass wall
Takat didih yang tinggi High boiling point
Boleh mengukur suhu yang tinggi Can measure high temperature
Penunu Bunsen Bunsen burner 100 0
0°c 100 °c
Kepekaan termometer boleh ditingkatkan dengan menggunakan: The sensitivity of the thermometer can be increased by using: (a) tiub kapilari yang kecil narrow capillary tube (b) dinding bebuli kaca yang nipis thin glass wall bulb (c) saiz bebuli yang kecil small bulb
θ
Perubahan dalam panjang turus merkuri, Δl = l100 – l0 The change in length of mercury column, Δl = l100 – l0
Perubahan suhu, ΔT1 = 100 ºC – 0 ºC = 100 ºC
The change in temperature, ΔT1 = 100 ºC – 0 ºC = 100 ºC U N I T
4
Perubahan dalam panjang pada suhu θ = lθ – l0 The change in length at temperature θ = lθ – l0
Oleh itu, suhu/ Therefore, the temperature, θ =
lθ – l0 × 100 ºC l100 – l0
Hubungan antara Haba dan Suhu/Relationship between Heat and Temperature: (a) Apabila suatu objek menyerap haba, suhu meningkat .
When an object absorbs heat, the temperature
increases .
(b) Apabila suatu objek membebaskan haba, suhu
When an object releases heat, the temperature
menurun .
decreases .
Keseimbangan Terma/Thermal Equilibrium: Apabila dua objek terdapat perbezaan suhu, haba dipindahkan di antara dua objek itu. When there is a temperature difference between two objects, heat is transferred between the two objects.
membebaskan Objek yang panas mencapai keseimbangan terma.
haba manakala objek yang sejuk
The hot object releases heat whereas the cold object © Nilam Publication Sdn. Bhd.
07 FIZ Form 4 Ch4 2F.indd 138
absorbs
heat until it
menyerap reaches
haba sehingga
thermal equilibrium.
138
9/10/13 4:36 PM
MODUL • Fizik TINGKATAN 4 Dalam keseimbangan terma At thermal equilibrium
A
Panas Hot
B
Sejuk Cold
A
B
Haba yang dibebaskan = Haba yang diserap (Kadar bersih pemindahan haba adalah SIFAR) Heat released = Heat absorbed (Rate of net heat transferred is ZERO)
Dua objek bersentuhan yang berlainan suhu akan mencapai keseimbangan terma apabila: Two objects in contact with each other but at different temperatures will reach thermal equilibrium when:
(a)
kadar bersih
the net rate
haba yang dipindahkan antara dua objek ialah of heat transfer between the two objects is
(b) suhu dua objek itu adalah
sama
zero
sifar
, dan
, and
.
the temperature of the two objects is the
same
.
Pada keseimbangan terma/At thermal equilibrium, haba yang diserap oleh objek yang sejuk adalah yang panas. the heat absorbed by the cool object is
equal
sama
dengan haba yang dibebaskan oleh objek
to the heat released by the hot object.
Pada keseimbangan terma, Kadar pemindahan tenaga = Kadar pemindahan tenaga dari B ke A dari A ke B
At thermal equilibrium, Rate of transfer of = Rate of transfer of energy from A to B energy from B to A
Latihan/Exercises 1 Panjang turus merkuri pada takat ais dan pada takat stim masing-masing ialah 5.0 cm dan 40.0 cm. Apabila termometer itu direndam dalam cecair P, panjang turus merkuri adalah 23.0 cm. Berapakah suhu cecair P?
2 Panjang turus merkuri pada takat stim dan pada takat ais masing-masing ialah 65.0 cm dan 5.0 cm. Apabila termometer itu direndam dalam cecair Q, panjang turus merkuri adalah 27.0 cm. Berapakah suhu cecair Q?
The lengths of the mercury column at ice point and steam point are 5.0 cm and 40.0 cm respectively. When the thermometer is immersed in liquid P, the length of the mercury column is 23.0 cm. What is the temperature of liquid P? l – l0 × 100 ºC Suhup/Temperaturep = θ l100 – l0 (23.0 – 5.0) cm = × 100 ºC (40.0 – 5.0) cm 18.0 cm = × 100 ºC 35.0 cm
The lengths of the mercury column at the steam point and ice point are 65.0 cm and 5.0 cm respectively. When the thermometer is immersed in liquid Q, the length of the mercury column is 27.0 cm. What is the temperature of liquid Q? (27.0 – 5.0) cm θ = × 100 ºC (65.0 – 5.0) cm 22 cm = × 100 ºC 60 cm = 36.67 ºC
4
= 51.43 °C
139
07 FIZ Form 4 Ch4 2F.indd 139
U N I T
© Nilam Publication Sdn. Bhd.
9/10/13 4:36 PM
MODUL • Fizik TINGKATAN 4
3 Jarak pada turus merkuri antara 0 ºC dan 100 ºC adalah 28.0 cm. Apabila termometer itu dimasukkan ke dalam bikar yang berisi air, panjang turus merkuri adalah 24.5 cm di atas titik ais. Berapakah suhu air tersebut? The distance between 0 ºC and 100 ºC on the mercury column is 28.0 cm. When the thermometer is put into a beaker of water, the length of the mercury column is 24.5 cm above the ice point. What is the temperature of the water? 24.5 cm × 100 ºC = 87.5 ºC θ= 28.0 cm
4 Jarak pada turus merkuri antara 0 ºC dan 100 ºC adalah 25 cm. Apabila termometer diletakkan dalam bikar yang berisi air, panjang turus merkuri adalah 16 cm di atas titik ais. The distance between 0 ºC and 100 °C on the mercury coloumn is 25 cm. When the thermometer is put into a beaker of water, the length of mercury column is 16 cm above the ice point.
(a) Berapakah suhu air tersebut? What is the temperature of the water?
(b) Berapakah panjang turus merkuri dari bebuli kaca pada suhu 30 ºC? What is the length of the mercury column from the bulb at 30 ºC? 16 cm × 100 ºC (a) θ = 25 cm = 64 ºC l – l0 (b) 30 °C = θ × 100 ºC 25 cm 30 °C × 25 cm lθ – l0 = 100 °C = 7.5 cm
U N I T
4
4.2
MEMAHAMI MUATAN HABA TENTU
UNDERSTANDING SPECIFIC HEAT CAPACITY
Definisi/Definition: Muatan haba tentu ialah haba (Q) yang diperlukan oleh sesuatu bahan berjisim 1 kg untuk menaikkan suhunya sebanyak 1 ºC.
1 2
The specific heat capacity is the heat (Q) required by a substance of mass 1 kg to increase its temperature by 1 ºC.
Persamaan/Equation:
Haba (Q) Muatan haba tentu, c = Jisim (m) × Perubahan suhu (θ)
Specific heat capacity, c =
Heat (Q) Mass (m) × Temperature change (θ)
c =
Q mθ
Unit S.I. bagi c ialah S.I. unit of c is
Q = mcθ
J kg–1 ºC–1
J kg–1 ºC–1
© Nilam Publication Sdn. Bhd.
07 FIZ Form 4 Ch4 2F.indd 140
⇒
Muatan haba tentu air ialah 4 200 J kg-1 ºC-1. Specific heat capacity of water is 4 200 J kg-1 ºC-1. Ini bermakna untuk 1 kg air meningkat suhu sebanyak 1 ºC, air perlu 4 200 J haba. This means that for 1 kg of water, to increase its temperature by 1 ºC, the water needs 4 200 J of heat.
Penerangan menggunakan teori kinetik jirim: Explanation using kinetic theory of matter: • Apabila bahan menyerap haba, getaran atom/molekul sangat kuat. When a substance absorbs heat, the vibrations of atoms/molecules are strong. • Atom/molekul bergerak dengan laju yang lebih tinggi. Atoms/molecules move with a higher speed. • Tenaga kinetik atom/molekul meningkat. The kinetic energy of the atoms/ molecules increases. • Suhu bahan tersebut meningkat. The temperature of the substance increases. • Keadaan fizikal bahan tidak berubah. The physical state of matter is unchanged.
140
9/10/13 4:36 PM
MODUL • Fizik TINGKATAN 4
Hubungan antara muatan haba tentu dan perubahan suhu: Relationship between specific heat capacity and temperature change:
berkadar songsang
• Muatan haba tentu adalah Specific heat capacity is
inversely proportional
dengan perubahan suhu. to the change in temperature.
• Bahan dengan muatan haba tentu yang tinggi akan mempunyai peningkatan suhu yang small
A material with high specific heat capacity will have a
perlahan
• Ini bermakna bahawa bahan
slow
This means that the material is
kecil .
increase in temperature.
menjadi panas/sejuk.
to get hot/cool.
• Contoh: Muatan haba tentu air ialah 4 200 J kg-1 ºC-1
Example: The specific heat capacity of water is 4 200 J kg-1 ºC-1
• Penerangan: Untuk 1 kg air, ia menyerap 4 200 J haba tetapi suhunya meningkat sebanyak 1 ºC. Explanation: For 1 kg of water, it absorbs 4 200 J of heat but the temperature increases by 1 ºC.
• Air menyerap kuantiti haba yang besar tetapi peningkatan suhu adalah Water absorbs a big quantity of heat but the temperature increased is
kecil
.
small .
• Air bertindak sebagai agen penyejuk bagi enjin kereta. Water acts as cooling agent in a car engine.
• Bahan yang mempunyai muatan haba tentu yang rendah akan mengalami peningkatan suhu yang besar
. large
Materials with low specific heat capacity will have a
• Ini bermaksud bahan
mudah
This means that the material
increase in temperature. U N I T
menjadi panas/sejuk.
easily
gets hot/cool.
4
• Contoh: Muatan haba tentu kuprum ialah 390 J kg-1 ºC-1 Example: Specific heat capacity of copper is 390 J kg-1 ºC-1
• Penerangan: Untuk 1 kg kuprum, ia menyerap 390 J haba tetapi suhu meningkat sebanyak 1 ºC. Explanation: For 1 kg of copper, it absorbs 390 J of heat but the temperature increases by 1 ºC.
• Kuprum menyerap kuantiti haba yang Copper absorbs a
small
kecil
tetapi peningkatan suhu adalah tinggi.
quantity of heat but the temperature increase is high.
• Kuprum digunakan sebagai elemen pemanas. Copper is used as a heating element.
141
07 FIZ Form 4 Ch4 2F.indd 141
© Nilam Publication Sdn. Bhd.
9/10/13 4:36 PM
MODUL • Fizik TINGKATAN 4
Aplikasi muatan haba tentu: Applications of Specific Heat Capacity:
1 Air sebagai penyejuk dalam enjin kereta Water as coolant in a car engine
2 Peralatan dan radas rumah Household apparatus and utensils
3 Bayu laut Sea Breeze
4 Bayu darat Land Breeze
(1) Air sebagai penyejuk dalam enjin kereta/Water as coolant in a car engine Air dipam melalui pam air Water is circulated by the water pump
Udara masuk melalui kipas radiator Air drawn in by the radiator fan
Kipas radiator Radiator fan
Haba hilang dari sirip penyejuk ke persekitaran Heat is lost from the cooling fins to the surroundings
Aliran air memasuki enjin Water enters the engine
Air mempunyai muatan haba tentu yang Water has a
large
4
It can absorb a
besar .
specific heat capacity.
Ia boleh menyerap jumlah tenaga yang U N I T
Haba yang dihasilkan dari pembakaran dalam silinder dipindahkan ke air Heat generated from the combustion in the cyclinders is transferred to the water
large
besar
tetapi suhu yang meningkat adalah kecil.
amount of heat but the increase in temperature is small.
Ia boleh menyejukkan pembakaran dalam enjin seperti enjin kereta. It can cool down internal combustion engines such as the car engine.
(2) Peralatan memasak dan perkakas rumah/Household apparatus and utensils Kuali diperbuat daripada logam. A frying pan is made from metal.
Logam mempunyai muatan haba tentu yang Metal has a
low
rendah .
specific heat capacity.
Ia menyerap haba yang meningkat tinggi
kecil
tetapi suhu yang
It absorbs a small quantity of heat but the increase in temperature is high
Ia mudah menjadi panas./It is easy to get hot. Makanan masak dengan The food is cooked in a © Nilam Publication Sdn. Bhd.
07 FIZ Form 4 Ch4 2F.indd 142
cepat . shorter
time.
142
9/10/13 4:36 PM
MODUL • Fizik TINGKATAN 4
(3) Bayu Laut/Sea Breeze Waktu siang Day time
Udara panas Hot air
Udara sejuk Cool air
Darat/Land
Laut/Sea
Darat (pepejal) mempunyai muatan haba tentu yang rendah berbanding dengan laut (cecair). Land (solid) has a low specific heat capacity compared to the sea(liquid).
lebih cepat
Pada waktu siang, darat menjadi panas In the day time, land gets hot
faster
berbanding laut.
than the sea.
Maka udara panas di darat yang mempunyai ketumpatan rendah akan bergerak naik ke atas membentuk kawasan daratan yang bertekanan rendah . low
Therefore, hot air on the land that has
density will rise up and produces
low
Udara sejuk di laut yang mempunyai ketumpatan tinggi dan bertekanan high
The cool air on the sea that has
density and
high
pressure region on the land.
tinggi
akan bertiup ke darat.
pressure will blow towards the land.
(4) Bayu darat/Land Breeze U N I T
Waktu malam Night time Udara panas Hot air
Udara sejuk Cool air
Darat/Land
4
Laut/Sea
Darat (pepejal) mempunyai muatan haba tentu yang rendah berbanding dengan laut (cecair). Land(solid) has a low specific heat capacity compared to the sea (liquid).
Pada waktu malam, darat menjadi sejuk
lebih cepat
faster
than the sea.
In the night time, land cools down
Udara panas di laut yang mempunyai ketumpatan kawasan bertekanan rendah . The hot air on the sea that has
low
berbanding laut.
rendah
akan bergerak naik ke atas dan menghasilkan
density will rise up and produce a
low
Udara sejuk di darat yang mempunyai ketumpatan tinggi dan bertekanan The cool air on the land that has
high
density and
high
143
07 FIZ Form 4 Ch4 2F.indd 143
pressure region.
tinggi
akan bertiup ke laut.
pressure will blow towards the sea. © Nilam Publication Sdn. Bhd.
9/10/13 4:36 PM
MODUL • Fizik TINGKATAN 4
Untuk menentukan muatan haba tentu pepejal dan cecair
Eksperimen
To determine the specific heat capasity of a solid and a liquid
Experiment
Tujuan
Untuk menentukan muatan haba tentu aluminium.
Untuk menentukan muatan haba tentu air.
Aim
To determine the specific heat capacity of aluminium.
To determine the specific heat capacity of Water.
Senarai radas dan bahan
Blok aluminium, kertas tisu, helaian polisterina, Cawan polisterina, air, pemanas perendam, minyak, pemanas rendam, termometer, bekalan termometer, bekalan kuasa, pengacau, neraca kuasa, neraca tuas dan jam randik tuas atau penimbang elektronik, dan jam randik.
List of apparatus and materials
Aluminium block, tissue paper, polystyrene sheet, oil, immersion heater, thermometer, power supply, triple beam balance and stopwatch.
Polystyrene cup, water, immersion heater, thermometer, power supply, stirrer, triple beam balance or electronic balance and stopwatch. Pemanas Heater
Termometer Thermometer Bekalan kuasa Power supply
Susunan radas Arrangement of the apparatus
Bekalan kuasa Power supply
Blok aluminium Aluminium block Lubang diisi dengan minyak untuk meningkatkan konduksi Hole filled with oil to increase thermal conductivity
Pemanas Heater
Silinder penyukat Measuring cyclinder
Termometer Thermometer Kertas tisu Tissue paper Jam randik Stopwatch 12
Air Water
9
3 6
Blok kayu Wooden block
1. Jisim blok aluminium, m, ditentukan
1. Cawan diisikan dengan air, berjisim m.
(contoh, m = 150 g)./The cup is filled with
dengan menggunakan neraca tiga tuas.
The mass of the aluminium block, m is
water of mass, m (example, m = 150 g).
determined using the triple beam balance.
2. Suhu awal air, θ1, direkodkan.
2. Suhu awal blok aluminium, θ1 direkodkan. The initial temperature of the water, θ1, is recorded. U N I T
The initial temperature of the aluminium block, θ1,
3. Pemanas dihidupkan dan pada masa yang sama
Prosedur
is recorded.
jam randik dimulakan. / The heater is switched
Procedure
3. Pemanas dihidupkan dan pada masa yang sama on and the stopwatch is started simultaneously.
4
jam randik dimulakan. / The heater is switched 4. Air dikacau berterusan.
on and the stopwatch is started simultaneously.
5. Pemanas ditutup selepas 10 minit.
Tabulate the data
The heater is switched off after 10 minutes.
The heater is switched off after 10 minutes.
6. Suhu tertinggi, θ2, direkodkan.
The highest temperature, θ2, is recorded.
The highest temperature, θ2, is recorded.
Kuasa pemanas, P/Power of the heater, P
Kuasa pemanas, P/Power of the heater, P
W = Jisim blok aluminium, m
= Jisim air, m
Mass of aluminium block, m
Mass of water, m
g = Suhu awal, θ1 /Initial temperature, θ1
g = Suhu awal, θ1 /Initial temperature, θ1
° = C Suhu akhir, θ2 /Final temperature, θ2
° = C Suhu akhir, θ2 /Final temperature, θ2
= © Nilam Publication Sdn. Bhd.
07 FIZ Form 4 Ch4 2F.indd 144
The water is stirred continuously.
4. Pemanas ditutup selepas 10 minit. 5. Suhu tertinggi, θ2, direkodkan.
Penjadualan data
°
C
=
W
°
C
144
9/10/13 4:36 PM
MODUL • Fizik TINGKATAN 4
Analisis data Analysis of the data
Pengiraan muatan haba tentu aluminium, c:
Pengiraan muatan haba tentu air, c:
Calculation of specific heat capacity of aluminium, c:
Calculation of specific heat capacity of water, c:
Q = mcθ Pt = mc(θ2 – θ1) Pt ∴ c = m(θ2 – θ1)
Q = mcθ Pt = mc(θ2 – θ1) Pt ∴ c = m(θ2 – θ1)
Latihan/Exercises 1 Sebuah blok logam berjisim 2 kg. Hitungkan jumlah haba yang mesti dipindahkan kepada logam untuk meningkatkan suhu dari 30 ºC kepada 70 ºC. (Muatan haba tentu logam = 500 J kg-1 ºC-1) A metal block has a mass of 2 kg. Calculate the amount of heat that must be transferred to the metal to raise its temperature from 30 ºC to 70 ºC (Specific heat capacity of the metal = 500 J kg-1 ºC-1) Penyelesaian/Solution: Q = mcθ = (2 kg) × (500 J kg–1°C–1) × (70 – 30)°C = (2 × 500 × 40) J = 40 000 J
4 Sebuah 2 kW, 240 V pemanas elektrik digunakan untuk memanaskan 3 kg air. Jika kuasa dibekalkan selama 8 minit, berapakah peningkatan suhu air tersebut? (Muatan haba tentu air adalah 4 200 J kg-1 ºC-1) A 2 kW, 240 V electric heater is used to heat up 3 kg of water. If the power is supplied for 8 minutes, what is the increase in temperature of the water? (The specific heat capacity of water is 4 200 J kg-1 ºC-1)
Penyelesaian/Solution: Pt = mcθ (2 000 W)(8 × 60 s) = 3 kg × 4 200 J kg–1°C–1 × θ (2 × 103 × 8 × 60) J 3 × 4 200 J °C–1 = 76.2 ºC
2 8.4 × 105 J tenaga haba meningkatkan suhu 4 kg air dari 40 ºC hingga 90 ºC. Berapakah muatan haba tentu air tersebut? 8.4 × 105 J of heat energy raises the temperature of 4 kg of water from 40 ºC to 90 ºC. What is the specific heat capacity of the water? Penyelesaian/Solution: Q c = mθ (8.4 × 105) J = 4 kg × (90 – 40)°C = 4 200 J kg-1 ºC-1
5 Sebuah pemanas rendam 1.2 kW digunakan untuk meningkatkan 0.2 kg air dalam bekas kuprum berjisim 0.05 kg. Kirakan masa yang diambil agar suhu air dan bekas dinaikkan sehingga 20 °C. (Muatan haba tentu air, ca = 4 200 J kg-1 °C-1). (Muatan haba tentu kuprum, ck = 400 J kg-1 °C-1)
Heat released = Heat absorbed
(0.2 kg)(ca)(100 – θ)°C = (0.25 kg)(ca)(θ – 10)°C 20 – 0.2 θ = 0.25θ – 2.5 20 + 2.5 = 0.25θ + 0.2θ 22.5 = 0.45 θ 22.5 θ = = 50 ºC 0.45 di mana/where ca = muatan haba tentu air specific heat capacity of water
4
Penyelesaian/Solution: Pt = mcθ (1.2 × 103 W) × t = macaθ + mkckθ (1 200 W) × t = θ (maca + mkck) = 20°C [(0.2 kg × 4 200 J kg–1 °C–1 + (0.05 kg × 400 J kg °C–1)] 17 200 J t = 1 200 J s–1 = 14.33 s 145
07 FIZ Form 4 Ch4 2F.indd 145
U N I T
A 1.2 kW immersion heater is used to raise the temperature of 0.2 kg water in a copper container of mass 0.05 kg. Calculate the time taken so that the temperature of the water and the container is increased by 20 °C. (The specific heat capacity of water, ca = 4 200 J kg-1 °C-1) (The specific heat capacity of copper, ck = 400 J kg-1 °C-1)
3 0.2 kg air pada suhu 100 ºC dicampur dengan 0.25 kg air pada suhu 10 ºC. Berapakah suhu tertinggi yang dicapai oleh campuran tersebut?
0.2 kg of water at 100 ºC is mixed with 0.25 kg of water at 10 ºC. What is the maximum temperature reached by the mixture? Penyelesaian/Solution: Haba yang dibebaskan = Haba yang diserap
θ =
© Nilam Publication Sdn. Bhd.
9/10/13 4:36 PM
MODUL • Fizik TINGKATAN 4
4.3
MEMAHAMI HABA PENDAM TENTU (L)
UNDERSTANDING SPECIFIC LATENT HEAT (L)
Graf Pemanasan Suhu-masa / Temperature-time Heating Graph Haba boleh menyebabkan objek: Heat can make an object: (i) berubah suhu, θ atau change in temperature,θ or (ii) berubah keadaan fizikal change its physical state
Pada QR dan ST, keadaan fizikal berubah tetapi suhu adalah tetap: At QR and ST, the physical states change but the temperature is constant:
• Haba diserap digunakan untuk memecahkan
between the atoms/molecules. . / The physical state changes .
berubah
tidak berubah
• Tenaga kinetik atom/molekul
The kinetic energy of the atoms/molecules
tetap . / The temperature is
• Suhu
antara atom/molekul.
bonds
The heat absorbed is used to break the
• Keadaan fizikal
ikatan
.
remains unchanged constant
.
.
Suhu/°C Temperature/°C
U Gas Gas
Cecair + Gas Liquid + Gas S
θ5 Haba/Heat, Q5 = mcθ 5 T
Haba/Heat, Q4 = mL
Cecair Liquid
θ3 Haba/Heat, Q3 = mcθ3
U N I T
4
0 Pepejal Solid
Q
Pepejal + Cecair Solid + liquid
Masa/s Time/s
R
Haba/Heat, Q2 = mL
θ1 Haba/Heat, Q1 = mc θ1
P
Pada PQ, RS dan TU, suhu menaik tetapi keadaan fizikal tidak berubah: At PQ, RS and TU, the temperature increases but the physical states are unchanged:
• Haba yang diserap digunakan untuk kinetik atom/molekul. The heat absorbed is used to atoms/molecules.
increase
meningkatkan
the kinetic energy of the
meningkat ./The temperature increases • Ikatan antara atom/molekul tidak terputus . • Suhu
The bonds between atoms/molecules do
• Keadaan fizikal
not break
Perubahan Changes
Malar Constant
Persamaan Equation
Suhu meningkat, θ Temperature Increase, θ
Keadaan fizikal Physical state
Q = mcθ
Keadaan fizikal Physical state
Suhu Temperature
Q = mL
. .
tidak berubah .
The physical state is © Nilam Publication Sdn. Bhd.
07 FIZ Form 4 Ch4 2F.indd 146
tenaga
Graf pemanasan (Haba diserap) Heating graph (Heat is absorbed):
unchanged
.
146
9/10/13 4:36 PM
MODUL • Fizik TINGKATAN 4 Penerangan menggunakan Teori Kinetik jirim: Explanation using Kinetic theory of matter: • Apabila bahan menyerap haba, getaran atom/molekul adalah kuat. When a substance absorbs heat, the vibrations of atoms/ molecules are strong. • Ikatan antara atom/molekul menjadi lemah. The bonds between atoms/ molecules become weak. • Atom/molekul dengan tenaga yang tinggi boleh memecahkan ikatan dan mengubah kepada keadaan fizikal yang baru. Atoms/molecules with very high energy can break the bonds and change to a new physical state. • Keadaan fizikal bahan berubah. The physical state of matter changes. • Tenaga kinetik atom/molekul tidak berubah. The kinetic energy of the atoms/ molecules is unchanged. • Suhu tetap/tidak berubah. The temperature is constant.
Haba Pendam Tentu / Specific Latent Heat Definisi/Definition: Haba pendam tentu (Q) ialah haba yang diperlukan untuk mengubah bahan berjisim 1 kg dari satu keadaan fizikal kepada keadaan fizikal yang lain pada suhu yang tetap. Specific latent heat (Q) is the heat required to change 1 kg of a substance from one physical state to another physical state at constant temperature.
Persamaan/Equation:
Heat (Q) Haba (Q) / Specific Latent Heat, L = Mass (m) Jisim (m) Q L = ⇒ Q = mL m J kg–1 Unit S.I. bagi L ialah
Haba pendam tentu, L =
S.I. unit of L is
J kg–1
Perubahan keadaan fizikal (proses pemanasan): Change of physical state (Heating process):
Cecair
Jenis Haba Pendam Tentu: Types of Specific Latent Heat: (i) Haba pendam tentu pelakuran (Lf)
Pelakuran
Pengewapan
Specific latent heat of fusion (Lf)
Fusion
Vaporisation
Specific latent heat of vaporisation (Lv)
Pepejal
Gas
solid
Gas
(ii) Haba pendam tentu pengewapan (Lv)
Pelakuran – proses di mana pepejal berubah kepada cecair apabila haba diserap Fusion – A process where solid changes to liquid when heat is absorbed.
Liquid
Pengewapan – proses di mana cecair berubah kepada wap apabila haba diserap Vaporisation - A process where liquid changes to gas when heat is absorbed.
Jenis Haba Pendam Tentu, L/Types of Specific Latent Heat, L
Haba Pendam Tentu Pelakuran (Lf)
Haba Pendam Tentu Pengewapan (Lv)
Specific Latent Heat of Fusion (Lf)
Specific Latent Heat of Vapourisation (Lv)
Definisi/Definition Haba (Q) yang diperlukan untuk menukarkan cecair berjisim 1 kg kepada wap pada takat didih.
Definisi/Definition Haba (Q) yang diperlukan untuk mengubah pepejal berjisim 1 kg kepada cecair pada takat lebur. Heat (Q) required to change 1 kg of solid to liquid at its melting point.
Persamaan/Equation: Haba pendam tentu, Lf = Specific latent heat, Lf =
Q Lf = m
4
Heat (Q) required to change 1 kg of liquid to vapour at its boiling point. Ais Ice
Air Water
Haba (Q) Jisim (m)
Air / Water
Stim/Wap / Steam
Persamaan/Equation: Haba (Q) Haba pendam tentu, Lv = Jisim (m)
Heat (Q) Mass (m)
Specific latent heat, Lv =
⇒ Q = mLf
Unit S.I. bagi Lf ialah J kg-1/S.I. unit of Lf is J kg-1
147
07 FIZ Form 4 Ch4 2F.indd 147
U N I T
Heat (Q)
Mass (m)
Q L = ⇒ Q = mL m Unit S.I. bagi Lv ialah J kg-1/S.I. unit of Lv is J kg-1
© Nilam Publication Sdn. Bhd.
9/10/13 4:36 PM
MODUL • Fizik TINGKATAN 4
Aplikasi Haba Pendam Tentu
Applications of Specific Latent Heat
1 Apabila kita terlibat dalam aktiviti-aktiviti berat, peluh menyejukkan badan kita. Peluh haba pendam disejat dan haba badan dikeluarkan sebagai , menyebabkan suhu badan kita menurun. When we are engaged in strenuous activities, sweating cools our bodies. The sweat evaporates and latent heat the body heat is removed as , thus lowering our body temperature.
Ais Ice
2 Minuman boleh disejukkan dengan menambah beberapa ketulan ais. Apabila ais mencair, haba pendam pelakuran diserap dari minuman. Suhu air tersebut semakin berkurangan. Drinks can be cooled by adding in several ice cubes. When the ice melts, the latent heat of fusion is absorbed from the drinks. The temperature of the drinks is lowered.
Alat pengukus makanan Food Steamer
3 Makanan boleh dimasak menggunakan stim. Makanan seperti kek, telur ikan dan roti dan lain-lain menerima tenaga yang berjumlah besar apabila haba pendam pengewapan stim dibebaskan dari stim yang terkondensasi. Food can be cooked by using steam. Food such as cakes, eggs, fish, buns and others receive a large amount of latent heat of vaporisation released energy when the of steam from condensing steam.
Eksperimen Experiment
Tujuan U N I T
Aim
4 Pemboleh ubah Variables
Menentukan haba pendam tentu pelakuran ais dan haba pendam tentu pengewapan ais To determine the specific latent of fusion of ice and specific latent vaporisation of water
Untuk menentukan haba pendam tentu
Untuk menentukan haba pendam tentu
pelakuran ais. / To determine the specific latent
pengewapan air. / To determine the specific latent
heat of fusion of ice.
heat of vaporisation of water.
Pemboleh ubah dimanipulasi: Manipulated variable: Haba yang dibekalkan, Q/Heat supplied, Q
Pemboleh ubah dimanipulasi: Manipulated variable: Haba yang dibekalkan, Q/Heat supplied, Q
Pemboleh ubah bergerak balas/Responding variable: Jisim ais yang cair, m
Pemboleh ubah bergerak balas/Responding variable: Jisim air yang telah diwapkan, m
Mass of ice that melts, m
Mass of water which has vaporised, m
Pemboleh ubah dimalarkan/Fixed variable: Tempoh pemanasan, t/Period of heating, t
Pemboleh ubah dimalarkan/Fixed variable: Tempoh pemanasan, t/Period of heating, t
Pemanas elektrik Eletrical heater
Senarai radas dan bahan List of apparatus and materials
Ais Ice
Bekalan kuasa Power supply Bikar A Beaker A
© Nilam Publication Sdn. Bhd.
07 FIZ Form 4 Ch4 2F.indd 148
Bekalan kuasa Power supply
Ais Ice
Pemanas Heater Neraca tuas Lever balance
Bikar B Beaker B
148
9/10/13 4:36 PM
MODUL • Fizik TINGKATAN 4
1. Radas B digunakan sebagai kawalan.
1. Pemanas dengan kuasa, P disambungkan kepada bekalan kuasa.
Apparatus B is used as a control.
2. Pemanas disediakan dan disambung seperti
A heater with power, P is connected to the power
ditunjukkan dalam rajah.
supply.
The heater is set up and connected as shown in
2. Tuang air ke dalam bikar sehingga hampir
the diagram.
penuh./Pour water into a beaker until it is
3. Litar ditutup dan jam randik dimulakan
serentak.
3. Litar ditutup.
The circuit is closed and the stopwatch is started
Procedure
The circuit is closed.
4. Apabila air mula mendidih, bacaan neraca
simultaneously.
Prosedur
almost full.
4. Air yang turun dari penapis corong
tuas, m1, direkodkan dan jam randik juga
dikumpul dalam bikar A dan B.
dimulakan serentak.
Water which drips from the filter funnels is
When the water starts to boil, the reading
collected in beakers A and B.
on the lever balance, m1, is recorded and
5. Pemanas dan jam randik diberhentikan
the stopwatch is started simultaneously.
serentak selepas t = 3 minit
5. Selepas t = 4 minit, jam randik
The heater and the stopwatch are switched off
diberhentikan dan jisim terakhir air, m2
simultaneously after t = 3 minutes
direkodkan.
6. Jisim air dalam kedua-dua bikar diukur.
After t = 4 minutes, the stopwatch is stopped
The mass of the water in both beakers is
and final mass of the water, m2 is recorded.
measured.
Kuasa pemanas, P =
W
Power of the heater, P =
Masa pemanasan, t = Time of heating, t =
Penjadualan Jisim air dalam bikar A, mA = data Mass of water in beaker A, mA = Tabulate the Jisim air bikar B, mB = data
Mass of water in beaker B, mB =
Kuasa pemanas, P =
W
Power of the heater, P =
s
Masa pemanasan, t =
s
Time of heating, t =
g g
g g
W W
4
s s
Jisim air dalam bikar, m1 = ………….. g Initial mass of water, m1 =
Jisim terakhir air, m2 =
Final mass of water, m2 =
g
g g
Jisim air dari ais yang mencair oleh pemanas, m = mA– mB = g
Jisim air yang telah diwapkan, m = m1 – m2 = g
Mass of water from the ice melted by the heater, m = mA – mB = g
Mass of water that has vaporised, m = m1 – m2 = g
Kirakan haba pendam tentu pelakuran ais, Lf :
Kirakan haba pendam tentu pengewapan air, LV:
Calculation of specific latent heat of fusion of ice, Lf :
Data dianalisis
Analysis the data
Q = mLf Pt = mLf Pt Lf = m
Calculation of specific latent heat of vaporisation of water, Lv:
Q = mLV Pt = mLV Pt L V = m –1 Unit, Lv ialah J kg
Unit, Lf ialah J kg–1
The unit of Lf is J kg–1
The unit of Lv is J kg–1
149
07 FIZ Form 4 Ch4 2F.indd 149
U N I T
© Nilam Publication Sdn. Bhd.
9/10/13 4:36 PM
MODUL • Fizik TINGKATAN 4
Latihan/Exercises 1 Berapakah kuantiti haba yang diperlukan untuk mencairkan 2.0 kg ais pada 0 ºC? (Haba pendam tentu pelakuran ais = 3.34 × 105 J kg-1) What is the quantity of heat required to melt 2.0 kg ice at 0 ºC? (The specific latent heat of fusion of ice = 3.34 × 105 J kg-1)
Penyelesaian/Solution: Q = mLf = (2.0 kg) × (3.34 × 105 J kg–1) = 6.68 × 105 J
2 Berapakah tenaga harus dialihkan dari 4.0 kg air pada 20 ºC untuk menghasilkan ais pada 0 ºC? How much energy has to be removed from 4.0 kg of water at 20 ºC to produce a block of ice at 0 ºC?
[Muatan haba tentu air = 4.2 × 10 J kg ºC , haba pendam tentu pelakuran ais = 3.34 × 105 J kg-1] 3
-1
-1
4 Kirakan haba yang diperlukan untuk menukar 4 kg ais pada –15 ºC kepada stim pada 100 ºC. Calculate the heat required to convert 4 kg of ice at –15 ºC to steam at 100 ºC. [Muatan haba tentu ais/Specific heat capacity of ice
specific latent heat of fusion of ice = 3.34 × 105 J kg-1 dan/and
haba pendam tentu pengewapan air specific latent heat of vaporisation of water
= 2.26 × 106 J kg-1] Penyelesaian/Solution:
[Specific heat capacity of water = 4.2 × 103 J kg-1 ºC-1, specific latent heat of fusion of ice = 3.34 × 105 J kg-1] Air/Water 20 °C
mcθ
Air/Water 0 °C
mLf
Ais/Ice 0 °C
4
miciθi
Ais/Ice 0 °C
Air/Water 100 °C
mLv
mLf
Air/Water 0 °C
Stim/Steam 0 °C
Jumlah tenaga haba = miciθi + mLf + mwcwθw + mLv Total heat energy
3 Sebuah elektrik pemanas 800 W digunakan untuk memanaskan air. Berapakah masa yang diperlukan untuk mengurangkan air sebanyak 4 kg selepas air mencapai takat didih? [Haba pendam tentu pengewapan air = 2.26 × 106 J kg-1] A 800 W electric heater is used to boil water. What is the time required to reduce the mass of water by 4 kg after the water has reached its boiling point? [Specific latent heat of vaporisation of water = 2.26 × 106 J kg-1] Penyelesaian/Solution: Tenaga elektrik yang Tenaga haba yang dibekalkan = diterima Electric energy provided
Ais/Ice –15 °C mwcwθw
Jumlah tenaga haba/Total heat energy = mcθ + mLf = [(4.0 kg) × (4.2 × 103 J kg–1°C–1) × (20 – 0)°C] + [(4 kg) × (3.34 × 105 J kg–1)] = 3.36 × 105 J + 13.36 × 105 J = 16.72 × 105 J = 1.672 × 106 J U N I T
= 2.1 × 103 J kg-1 ºC-1, Muatan haba tentu air/Specific heat capacity of water = 4.2 × 103 J kg-1 ºC-1, haba pendam tentu pelakuran ais
miciθi = (4 kg) × (2.1 × 103 J kg–1°C–1) × (15°C) = 1.26 × 105 J mLf = (4 kg) × (3.34 × 105 J kg–1) = 1.336 × 106 J mwcwθw = (4 kg) × (4.2 × 103 J kg–1°C–1) × (100°C) = 1.68 × 106 J mLv = (4 kg) × (2.26 × 106 J kg–1) = 9.04 × 106 J ∴ Jumlah tenaga haba/Total heat energy = (1.26 × 105 J) + (1.336 × 106 J) + (1.68 × 106 J) + (9.04 × 106 J) = 1.218 × 107 J
Heat absorbed
Pt = mLv (800 W) t = 4 kg × 2.26 × 106 J kg–1 (4 × 2.26 × 106) J t = 800 J s–1 = 1.13 × 104 s
© Nilam Publication Sdn. Bhd.
07 FIZ Form 4 Ch4 2F.indd 150
150
9/10/13 4:36 PM
MODUL • Fizik TINGKATAN 4
5 Sebuah blok pepejal 0.5 kg dipanaskan oleh pemanas elektrik 100 W. Graf menunjukkan bagaimana suhu berubah dengan masa.
Hitungkan haba pendam tentu pelakuran pepejal itu. Calculate the specific latent heat of fusion of the solid. Penyelesaian/Solution: Pt = mLf
0.5 kg of a solid block is heated by a 100 W heater. The graph shows how the temperature varies with time.
(100 W) × (1 050 – 30)s = 0.5 kg × Lf 75 000 J 0.5 kg = 1.5 × 105 J kg–1
Suhu/Temperature / °C
100 80 60
Lf =
40 20 0
4.4
0
100 200 300 400 500 600 700 800 900 10001100 Masa/Time / s
MEMAHAMI HUKUM-HUKUM GAS UNDERSTANDING THE GAS LAWS
Hukum-hukum gas/Gas Laws: 4 kuantiti fizikal yang berkaitan:
Hukum Boyle Boyle’s Law
The 4 physical quantities involved are:
(i) Jisim (m) – sentiasa tetap dengan menggunakan bekas tertutup
Mass (m) – always kept constant by using a closed container
3 Hukum gas
(ii) Tekanan (P)
3 gas laws
Pressure (P)
(iii) Isi padu (V)
boleh berubah
can change
Volume (V)
(iv) Suhu mutlak (T)
Absolute temperature (T)
Hukum Charles
Hukum tekanan
Charles’ Law
Pressure Law
U N I T
4
Penerangan menggunakan Teori Kinetik/Explanation Using Kinetic Theory of Matter Kuantiti/Quantity
Penerangan/Explanation
Jisim (m)
Jumlah jirim dalam bekas yang tertutup, jumlah jirim adalah tetap. Jisim adalah malar.
Mass (m)
Amount of matter is in a closed container, the amount of matter is constant. The mass is constant.
Isi padu (V)
Isi padu gas adalah sama dengan isi padu bekas.
Volume (V)
Volume of gas is equal to the volume of container.
Tekanan (P)
Kadar perlanggaran antara molekul-molekul gas dengan dinding bekas. Apabila kadar perlanggaran antara molekul gas dengan dinding bekas bertambah, tekanan gas juga bertambah.
Pressure (P)
Rate of collision between the gas molecules and the walls of the container. When the rate of collisions between gas molecules and the walls of the container increases, the gas pressure also increases.
Suhu (T) Temperature (T)
Apabila molekul-molekul gas bergerak dengan halaju tinggi, tenaga kinetik bertambah, maka suhu gas juga bertambah. Kinetic energy of the gas molecules. When the gas moves at a higher speed, the kinetic energy increases; so the gas temperature also increases.
151
07 FIZ Form 4 Ch4 2F.indd 151
© Nilam Publication Sdn. Bhd.
9/10/13 4:36 PM
07 FIZ Form 4 Ch4 2F.indd 152
© Nilam Publication Sdn. Bhd.
Tenaga kinetik gas tetap, maka suhu gas juga tetap .
The kinetic energy of the gas is constant, so the temperature of the gas is constant .
V2 , T2
1
increases
to keep
The gas pressure is
tetap .
constant .
constant . T2 > T1 dan/and V2 > V1
Tekanan gas adalah
and the walls of the container is
the rate of collision between the gas molecules
The volume of the gas also
Isi padu gas juga bertambah menyebabkan kadar perlanggaran molekul gas dengan dinding bekas tidak berubah .
When the gas is heated, temperature increases . (T2 > T1)
2
Apabila gas dipanaskan, suhu akan meningkat . (T > T )
Explanation using Kinetic Theory of Matter:
constant .
increases .
the pressure of gas is directly proportional to the absolute temperature of the gas if the volume of the gas is kept constant.
the volume of the gas is directly proportional to the absolute temperature of the gas if the pressure is kept constant.
the volume of the gas if the temperature is kept
constant
.
Pressure Law states that for a fixed mass of gas, Charles' Law states that for a fixed mass of gas,
Boyle's Law states that for a fixed mass of gas, the gas pressure is inversely proportional to
adalah malar .
songsang dengan isi padu gas jika suhu
mutlak gas jika isi padu gas adalah tetap.
Hukum Tekanan menyatakan bahawa bagi suatu jisim gas yang tetap, tekanan gas
tekanan gas itu adalah tetap.
The gas pressure
Tekanan gas meningkat .
and the walls of the container
The rate of collision between the gas molecules
Kadar perlanggaran di antara molekul gas dengan dinding bekas itu meningkat .
the volume of the gas is
The volume of the container is kept constant, so
Isi padu bekas dikekalkan malar, jadi isi padu gas adalah tetap .
When the gas is heated, its temperature increases . (T > T ) 2 1
Apabila gas itu dipanaskan, suhu meningkat . (T2> T1)
Explanation using Kinetic Theory of Matter:
increases . T2 > T1 dan/and P2 > P1 (ii) Pernyataan/Statement:
adalah berkadar langsung dengan suhu
(ii) Pernyataan/Statement: Hukum Charles menyatakan bahawa untuk jisim gas yang tetap, isi padu gas berkadar
P2 , T2
(i) Penerangan menggunakan Teori Kinetik Jirim:
P1 , T1
Haba/Heat
Hukum Tekanan: (m & V)/Pressure Law: (m & V – pemalar/constant : P ∝ T)
langsung dengan suhu mutlaknya jika
V2 < V1 dan/and P2 > P1 (ii) Pernyataan/Statement: Hukum Boyle menyatakan bahawa untuk jisim gas yang tetap, tekanan gas berkadar
.
The gas pressure
increases
Tekanan gas turut
.
bertambah .
increases
The rate of collision between the gas molecules
and the walls of the container
Kadar perlanggaran antara molekul gas dengan dinding bekas bertambah .
decreases . (V < V ) 2 1
volume of the gas also
When the volume of the container decreases, the
Apabila isi padu bekas berkurang, isi padu gas juga berkurang . (V2 < V1)
Explanation Using Kinetic Theory of Matter:
V1 , T1
(i) Penerangan menggunakan Teori Kinetik Jirim:
4
(i) Penerangan dengan menggunakan Teori Kinetik Jirim:
Omboh/Piston P2 , V2
Haba/Heat
Hukum Charles/Charles’ Law: (m & P – pemalar/constant: V ∝ T)
U N I T
P1 , V1
Hukum Boyle/Boyle’s Law: 1 (m & T – pemalar/constant: P ∝ ) V
MODUL • Fizik TINGKATAN 4
152
9/10/13 4:36 PM
07 FIZ Form 4 Ch4 2F.indd 153
153
(c)
(b)
(a)
0
PV
0
P/Pa
0
P/Pa
P is constant
P adalah tetap
V P is directly 1 proportional to __ V
P berkadar langsung 1 dengan __
P/Pa
1 __ V
V
P is inversely proportional to V
P berkadar songsang dengan V
0
0 V
–273
(c)
(b)
(a)
V __ T
0
V
T/K
V
θ /°C
V varies linearly with θ
V berubah secara linear dengan θ
V is directly proportional to T
V berkadar langsung dengan T
P is constant
P adalah tetap
(v) Graf/Graph:
Volume of gas (V) is directly proportional to the absolute temperature of gas (T)
(v) Graf/Graph:
to
V∝T (iv) Persamaan/Equation: V = k (T) ; k adalah malar/is a constant k = V = pemalar/constant T V1 V2 ∴ = T1 T2
inversely proportional
Isi padu gas (V) adalah berkadar langsung dengan suhu mutlak gas (T)
(iii) Hubungan/Relationship:
P∝ 1 V (iv) Persamaan/Equation: P = k ( 1 ) ; k adalah malar/is a constant V k = P V = malar/constant ∴ P1 V1 = P2 V2
Gas pressure (P) is the gas volume (V)
dengan isi padu gas (V)
(iii) Hubungan/Relationship: berkadar songsang Tekanan gas (P)
(c)
–273
(b)
(a)
0
P T
0
P/Pa
0
P/Pa
(v) Graf/Graph:
P is constant
P adalah tetap
P varies linearly with θ
P berubah secara linear dengan θ
P/Pa
θ /°C
T/K
P is directly proportional to T
P berkadar langsung dengan T
P∝T (iv) Persamaan/Equation: P = k (T) ; k adalah malar/is a constant k = P = pemalar/constant T P1 P2 ∴ = T1 T2
Pressure of gas (P) is directly proportional to the absolute temperature of gas (T).
(iii) Hubungan/Relationship: Tekanan gas (P) berkadar langsung dengan suhu mutlak gas (T).
MODUL • Fizik TINGKATAN 4
4
U N I T
© Nilam Publication Sdn. Bhd.
9/10/13 4:36 PM
MODUL • Fizik TINGKATAN 4
Eksperimen Experiment
(i) Mengkaji hubungan antara tekanan dan isi padu gas
To investigate the relationship between the pressure and volume of a gas
To investigate the relationship between the volume and temperature of a gas
(ii) Mengkaji hubungan antara isi padu dan suhu gas
Untuk menyiasat hubungan antara tekanan
Untuk menyiasat hubungan antara isi padu
dan isi padu untuk jisim gas yang tetap pada
dan suhu untuk jisim gas yang tetap pada
suhu yang sama.
suhu yang sama.
Tujuan
To investigate the relationship between the
To investigate the relationship between the
Aim
pressure and volume for a fixed mass of gas at
volume and temperature for a fixed mass of
constant temperature.
gas at constant pressure.
Pemboleh ubah dimanipulasi:
Pemboleh ubah dimanipulasi:
Manipulated variable:
Manipulated variable:
Tekanan udara, P/Pressure of air, P
Suhu udara, T/Temperature of air, T
Pemboleh ubah bergerak balas:
Pemboleh ubah bergerak balas:
Responding variable:
Responding variable:
Isi padu udara,V/Volume of air, V
Isi padu udara, V/Volume of air, V
Pemboleh ubah dimalarkan Fixed variable: Jisim dan suhu udara/Mass and temperature of air
Pemboleh ubah dimalarkan Fixed variable: Jisim dan tekanan udara/Mass and pressure of air
Pam basikal, tiub kapilari, tiub getah dan tolok bourdon.
Tiub kapilari, asid sulfurik, pembaris meter separa, kaki retort, bikar, tungku kaki tiga, tiub getah, termometer dan penunu Bunsen.
Pemboleh ubah Variables
U N I T
4
Senarai radas dan bahan List of apparatus and materials
Bicycle pump, capillary tube, rubber tube and Bourdon gauge.
Skala isi padu Volume scale
Susunan radas Arrangement of the apparatus
Capillary tube, sulphuric acid, half metre rule, retort stand, beaker, tripod stand, rubber tube, thermometer and Bunsen burner.
Tiub kapilari Capillary tube
Pembaris meter separuh Half metre rule
Udara Air Minyak Oil Tolok Bourdon Bourdon gauge
Tiub kapilari Capillary tube
Termometer Thermometer
Pengacau/Stirrer
Pam Pump
Ais Ice Bikar Beaker
Air Water Xxxxxxxxxxxx
Gas terpendam Trapped air
Tangki minyak Oil reservoir
© Nilam Publication Sdn. Bhd.
07 FIZ Form 4 Ch4 2F.indd 154
154
9/10/13 4:36 PM
MODUL • Fizik TINGKATAN 4
1. Radas disediakan seperti rajah di atas.
1. Radas disediakan seperti rajah di atas.
The apparatus is set up as shown in the
The apparatus is set up as shown in the
diagram above.
diagram above.
2. Tekanan awal dan isi padu udara dalam tiub kapilari direkodkan.
lahan dan dikacau dengan seragam dan
The initial pressure and volume of the air in
perlahan. Apabila bacaan termometer itu
the capillary tube are recorded.
adalah 30 °C, panjang turus udara yang
3. Tolak pam sehingga tekanan udara
terperangkap, l cm, direkodkan.
adalah 100 kPa.
The water in the beaker is heated slowly
Push the pump until the pressure of air is 100 kPa.
4. Catatkan isi padu udara dalam tiub kapilari. Record the volume of the air in the capillary tube.
Prosedur Procedure
2. Air di dalam bikar dipanaskan perlahan-
and stirred unifromly and gently. When the reading of the thermometer is 30 °C, the length of the trapped air, l cm is recorded.
5. Ulangi eksperimen dengan tekanan udara,
3. Ulangi eksperimen dengan suhu
yang berbeza, iaitu, P = 120 kPa, 140 kPa,
berlainan iaitu θ = 40 °C, 50 °C, 60 °C
160 kPa dan 180 kPa. / Repeat the
dan 70 °C.
experiment with different pressures of air that is,
Repeat the experiment with different
P = 120 kPa, 140 kPa, 160 kPa and 180 kPa.
1 6. Nilai-nilai tekanan, P, isi padu, V, dan V
temperatures of water that is θ = 40 °C, 50° C, 60 °C and 70 °C.
4. Nilai-nilai suhu, θ, suhu mutlak, T, dan
dijadualkan.
1 The values of pressure, P, volume, V, and V
panjang turus udara yang terperangkap, l,
are tabulated.
dijadualkan.
1 7. Graf P melawan V dan P melawan V
The values of temperature, θ, absolute temperature,
diplotkan.
T, and length of air trapped, l, are tabulated.
Plot a graph P against V and a graph of P
1 against V .
Tekanan gas, P/kPa Pressure of air, P/kPa
5. Plotkan graf l terhadap T (dalam Kelvin). Plot a graph l against T (in Kelvin).
Isi padu gas/cm3 Volume of air/cm3
V1
V2
Vavg
1 /cm-3 V
100
Suhu gas, θ/°C
Suhu mutlak, T/K
U N I T
4
Panjang udara terperangkap, / cm
Length of trapped Temperature Absolute of air, θ/°C temperature, air, /cm T/K l1 l2 lavg
Penjadualan data
120
Tabulate the data
140
30
160
40
180
50 60 70
155
07 FIZ Form 4 Ch4 2F.indd 155
© Nilam Publication Sdn. Bhd.
9/10/13 4:36 PM
MODUL • Fizik TINGKATAN 4 l/cm
P/ kPa
Analisis data Analysis of the data
V/cm3
0 P/ kPa
l/cm
1 ( — ) /cm–3 V
0
Eksperimen Experiment
θ / °C
0
0
T/K
Mengkaji hubungan antara tekanan dengan suhu gas
To investigate the relationship between the pressure and temperature of a gas
Untuk menyiasat hubungan antara tekanan dengan suhu untuk jisim gas yang ditetapkan Tujuan
pada isi padu yang sama.
Aim
To investigate the relationship between the pressure and temperature for a fixed mass of gas at constant volume.
Pemboleh ubah dimanipulasi: Manipulated variable: Suhu udara, T/Temperature of air, T U N I T
Pemboleh ubah Variables
4
Pemboleh ubah bergerak balas: Responding variable: Tekanan udara, P/Pressure of air, P Pemboleh ubah dimalarkan: Fixed variable: Jisim dan isi padu udara/Mass and volume of air
Senarai radas dan bahan
Kelalang dasar bulat, termometer, kaki retort, tolok Bourdon, pengacau, ais, tiub getah, tungku kaki tiga, kasa dawai dan penunu Bunsen.
List of apparatus and materials
Round-bottomed flask, thermometer, retort stand, Bourdon gauge, stirrer, ice, rubber tube, tripod stand, wire gauze and Bunsen burner. Termometer Thermometer Kaki retort Retort stand
Susunan radas Arrangement of the apparatus
Ais Ice
07 FIZ Form 4 Ch4 2F.indd 156
Tolok Bourdon Bourdon gauge
Bikar Beaker Udara Air
Air Water
Kelalang Xxxxxxxxxxxxxxxxxx dasar bulat RoundBottomed Flask
© Nilam Publication Sdn. Bhd.
Tiub getah Rubber tube
Kasa dawai Wire gauze
156
9/10/13 4:36 PM
MODUL • Fizik TINGKATAN 4
1. Radas disediakan seperti yang ditunjukkan dalam rajah di atas. The apparatus is set up as shown in the diagram above.
2. Kelalang dasar bulat direndam dalam sebuah bikar berisi air dan ais bersama. The round-bottomed flask is immersed in a beaker of water containing ice.
3. Campuran air dan ais dikacau supaya udara di dalam kelalang mempunyai suhu yang sama seperti air. The mixture of water and ice is stirred so that the air in the flask has the same temperature as the water.
4. Apabila bacaan termometer adalah 30 °C, bacaan tekanan, P, pada tolok Bourdon Prosedur Procedure
direkodkan. When the reading of the thermometer is 30 °C, record the reading of the pressure, P on Bourdon gauge.
5. Ulangi eksperimen dengan suhu air yang berlainan, iaitu θ = 40 °C, 50 °C, 60 °C dan 70 °C. Repeat the experiment with different temperatures of water, that is θ = 40 °C, 50 °C, 60 °C and 70 °C.
6. Nilai-nilai suhu, θ, suhu mutlak, T dan tekanan udara terperangkap, P dijadualkan. The values of temperature, θ, absolute temperature, T and pressure of the air trapped, P are tabulated.
7. Graf P melawan T (dalam Kelvin) diplotkan. Plot a graph P against T (in Kelvin).
Suhu gas, θ/°C
Temperature of air, θ/°C
Suhu mutlak, T/K
Tekanan udara, P/kpa
Absolute temperature, T/K
Air pressure,P/kPa
30 Penjadualan data Tabulate the data
40
U N I T
4
50 60 70 P/ kPa
P/ kPa
Analisis data Analysis the data θ / °C
0
157
07 FIZ Form 4 Ch4 2F.indd 157
0
T/K
© Nilam Publication Sdn. Bhd.
9/10/13 4:36 PM
MODUL • Fizik TINGKATAN 4
Latihan/Exercises 1 Isi padu gelembung udara pada dasar laut sedalam 50 m ialah 250 cm3. Jika tekanan atmosfera ialah 10 m air, cari isi padu gelembung udara apabila ia tiba ke permukaan laut. The volume of an air bubble at a 50 m deep seabed is 250 cm3. If the atmospheric pressure is 10 m of water, find the volume of the air bubble when it reaches the surface of the sea.
Penyelesaian/Solution: P 2 = 10 m air
Menggunakan Hukum Boyle,/Using Boyle's Law, P1 V1 = P2 V2 1 atm × 100 cm3 = P2 × 20 cm3 100 cm3 P2 = × 1 atm = 5 atm 20 cm3
3
Menggunakan Hukum Boyle,/Using Boyle's Law, P1 V1 = P2 V2 (60 m air) (250 cm3) = (10 m air) × V2 (60 m air × (250 cm3) V2 = 10 m air = 1 500 cm3
4 Isi padu gas 20 m3 pada suhu 37 °C dipanaskan sehingga menjadi 87 °C pada tekanan malar. Berapakah peningkatan isi padunya? A gas of volume 20 m3 at 37 °C is heated until its temperature becomes 87 °C at constant pressure. What is the increase in volume?
2 Rajah menunjukkan tiub kaca yang mengandungi udara yang terperangkap di dalamnya. Pada 17 °C, turus udara yang terperangkap ialah 29 cm.
U N I T
The diagram shows a glass tube containing some trapped air inside it. At 17 °C, the vertical column of trapped air is 29 cm. Merkuri Mercury
4
Udara Air
5 cm 29 cm pada/at 17 °C
What is the vertical column of trapped air at a temperature of 57 °C?
Penyelesaian/Solution: Menggunakan Hukum Charles,/Using Charles' Law, V1 V2 di mana A = luas keratan = T1 T2 rentas tiub where A = cross-sectional A (L1) A (L2) area of the tube = , T1 T2 29 cm L2 = (17 + 273) K (57 + 273) K 29 cm × 330 K L2 = 290 K = 33 cm
07 FIZ Form 4 Ch4 2F.indd 158
Penyelesaian/Solution: Menggunakan Hukum Charles,/Using Charles' Law, V1 V2 = T1 T2 3 20 m V2 = (37 + 273) K (87 + 273) K 20 m3 × 360 K V2 = = 23.23 m3 310 K Peningkatan isi padu/Increase in volume = (23.23 – 20) m3 = 3.23 m3
Berapakah panjang turus udara yang terperangkap pada suhu 57 °C?
© Nilam Publication Sdn. Bhd.
A mixture of air and petrol vapour is injected into the cylinder of a car engine when the volume of the cylinder is 100 cm3. Its pressure is 1.0 atm. The valve is closed and the mixture is compressed to 20 cm3. Find the pressure now.
Penyelesaian/Solution:
P I = (50 m + 10 m) air = 60 m air
V 1 = 250 cm
3 Satu campuran udara dan wap petrol disuntik ke dalam enjin silinder kereta apabila isi padu silinder itu ialah 100 cm3. Tekanan adalah 1.0 atm. Injap ditutup dan campuran dimampatkan kepada 20 cm3. Cari tekanan sekarang.
5 Tekanan udara di dalam bekas pada 33 °C adalah 1.4 × 105 N m-2. Bekas itu dipanaskan sehingga suhu 55 °C. Berapakah tekanan udara akhir jika isi padu bekas ditetapkan?
The air pressure in a container at 33 °C is 1.4 × 105 N m-2. The container is heated until the temperature is 55 °C. What is the final air pressure if the volume of the container is fixed?
Penyelesaian/Solution: Menggunakan Hukum Tekanan,/Using Pressure Law, P1 P2 = T1 T2 1.4 × 105 N m–2 P2 = (33 + 273) K (55 + 273) K 1.4 × 105 N m–2 × 328 K P2 = 306 K = 1.5 × 105 N m-2
158
9/10/13 4:36 PM
MODUL • Fizik TINGKATAN 4
UNIT
5
CAHAYA LIGHT
5.1 MEMAHAMI PANTULAN CAHAYA
UNDERSTANDING REFLECTION OF LIGHT
• Menghuraikan ciri-ciri imej yang terhasil melalui pantulan cahaya Describe the characteristics of the image formed by reflection of light
• Menyatakan hukum-hukum pantulan State the laws of reflection of light
• Melukis gambar rajah untuk menunjukkan kedudukan dan ciri-ciri imej yang terbentuk oleh Draw ray diagrams to show the position and characteristics of the image formed by a
(i) cermin satah plane mirror
(ii) cermin cembung convex mirror
(iii) cermin cekung concave mirror
• Menghuraikan aplikasi pantulan cahaya Describe applications of reflection of light
• Menyelesaikan masalah yang melibatkan pantulan cahaya Solve problems involving reflection of light
• Membina alat berdasarkan aplikasi pantulan cahaya
Construct a device based on the application of reflection of light
5.2 MEMAHAMI PEMBIASAN CAHAYA
UNDERSTANDING REFRACTION OF LIGHT
• Menerangkan pembiasan cahaya Explain refraction of light
• Mendefinisi indeks pembiasan sebagai Define refractive index as
sin i n = sin r • Menentukan indeks pembiasan sebuah blok kaca atau blok perspek
U N I T
Determine the refractive index of a glass or perspex block
• Menyatakan indeks pembiasan,
5
State the refractive index,
kelajuan cahaya dalam vakum speed of light in a vacuum n = kelajuan cahaya dalam medium / speed of light in a medium • Menghuraikan fenomena yang disebabkan pembiasan Describe phenomena due to refraction
• Menyelesaikan masalah yang melibatkan pembiasan cahaya Solve problems involving the refraction of light
159
08 FIZ Form 4 Ch5 2F.indd 159
© Nilam Publication Sdn. Bhd.
9/10/13 4:39 PM
MODUL • Fizik TINGKATAN 4
5.3 MEMAHAMI PANTULAN DALAM PENUH CAHAYA
UNDERSTANDING TOTAL INTERNAL REFLECTION LIGHT
• Menerangkan pantulan dalam penuh cahaya Explain total internal reflection of light
• Mendefinisi sudut genting, c Define critical angle, c
1 • Menghubung kait sudut genting dengan indeks pembiasan, n = sin c 1 Relate the critical angle to the refractive index, n = sin c
• Menghuraikan fenomena semula jadi berkaitan pantulan dalam penuh Describe natural phenomenon involving total internal reflection
• Menghuraikan aplikasi pantulan dalam penuh Describe applications of total internal reflection
• Menyelesaikan masalah berkaitan pantulan dalam penuh Solve problems involving total internal reflection
5.4 MEMAHAMI KANTA
UNDERSTANDING LENSES
• Menerangkan titik fokus dan panjang fokus Explain focal point and focal length
• Menentukan titik fokus dan panjang fokus bagi kanta cembung Determine the focal point and focal length of a convex lens
• Menentukan titik fokus dan panjang fokus kanta cekung Determine the focal point and focal length of a concave lens
• Melukis rajah sinar untuk menunjukkan kedudukan dan ciri-ciri imej yang terhasil oleh Draw ray diagrams to show the positions and characteristics of the images formed by a
(i) kanta cembung convex lens (ii) kanta cekung concave lens v • Definisi pembesaran m = u Define magnification as m =
v u
• Menghubung kait panjang fokus, f, dengan jarak objek, u, dan jarak imej, v, iaitu, Relate focal length, f, to the object distance, u, and image distance, v, i.e. U N I T
5
1 1 1 = + f u v
1 1 1 = + f u v
• Menghuraikan kegunaan kanta dalam alat optik dengan menggunakan rajah sinar Describe with the aid of ray diagrams, the use of lenses in optical devices
• Membina alat-alat optik yang menggunakan kanta Construct optical devices that use lenses
• Menyelesaikan masalah berkaitan kanta Solve problems involving lenses
© Nilam Publication Sdn. Bhd.
08 FIZ Form 4 Ch5 2F.indd 160
160
9/10/13 4:39 PM
MODUL • Fizik TINGKATAN 4
5.1
MEMAHAMI PANTULAN CAHAYA
UNDERSTANDING REFLECTION OF LIGHT Sudut tuju = sudut pantulan Angle of incidence = angle of reflection
Hukum pantulan: The laws of reflection:
1 Sinar tuju, sinar pantulan dan garis normal berada pada satah yang sama. The incident ray, the reflected ray and the normal to the surface all lie in the same plane.
2 Sudut tuju, i, sama dengan sudut pantulan, r. The angle of incidence, i, is equal to the angle of reflection, r.
Cermin satah • Sudut tuju, i, ialah sudut antara tuju
normal
dan sinar
The angle of incidence, i, is the angle between the normal incident and the ray
• Sudut pantulan, r ialah sudut antara pantulan sinar
normal
Plane mirror Sinar tuju Incident ray Normal Normal
i r
dan Sinar pantulan Reflected ray
The angle of reflection, r, is the angle between the normal reflected and the ray
• Garis normal ialah garis yang berserenjang dengan permukaan cermin The normal is the line that is mirror
perpendicular
to the
Sinar rajah: imej yang terhasil oleh cermin satah Ray diagram: The image formed by a plane mirror Objek/Object u O
v
u = jarak objek
Imej/Image I
Cara melukis/Drawing method 1. Kedudukan imej ditentukan. The position of its image is located. 2. Jarak imej sama dengan jarak objek, u = v. The image distance is equal to the object distance, u = v. 3. Saiz imej sama dengan saiz objek. The image size is same as the object size. 4. Garis yang menyambung objek dan imej adalah bersudut tegak dengan cermin. The line joining the object and the image is perpendicular to the mirror. 5. Sinar pantulan dipanjangkan di belakang cermin sehingga bertemu di I (imej). The reflected rays are extrapolated to the back of the mirror until they meet at the image, I. 6. Sinar di belakang cermin sebenarnya tidak wujud. Sinar ini diwakili oleh garis putus-putus. The rays behind the mirror do not exist. They are represented by dotted lines. 7. Garis berterusan dari cermin ke mata menunjukkan sinar pantulan. The continuous lines from the mirror to the eye indicate the reflected rays.
u = object distance
U N I T
v = jarak imej v = image distance
5
i r
Mata Eye
Ciri-ciri imej cermin satah/Characteristics of the image in a plane mirror Saiz/Size : Saiz sama/Same size : Tegak/Upright Gambaran/Appearance : Maya, songsang sisi/Virtual, laterally inverted Kedudukan/Location : Jarak objek (u) = Jarak imej (v) Keadaan/Condition
Object distance (u) = Image distance (v)
161
08 FIZ Form 4 Ch5 2F.indd 161
© Nilam Publication Sdn. Bhd.
9/10/13 4:39 PM
MODUL • Fizik TINGKATAN 4
Aktiviti/Activity Gambar rajah menunjukkan seorang budak perempuan melihat cermin satah. Tuliskan imej yang dilihat oleh budak itu. The diagram shows a girl looking at a plane mirror. Write the image seen by the girl.
W, V, U
Cermin satah Plane mirror
Istilah pada cermin Common terminology of mirror Pusat lengkungan, C Centre of curvature, C
Pusat sfera di mana cermin melengkung terbentuk. The geometric centre of a sphere of which the concave or convex mirror is a part.
Kutub cermin, P Pole of mirror, P
Titik tengah pada cermin bulat. The centre point on the spherical mirror.
Paksi utama Principal axis
Garis lurus yang melalui pusat lengkungan bersudut tegak (90°) dengan satah cermin. Straight line which passes through the optical centre at right angles (90°) to the plane of the mirror.
Titik fokus, F Focal point, F
Titik pada paksi utama yang mana sinar tuju yang selari dengan paksi utama: Is the point on the principal axis to which incident rays of light travelling parallel to the principal axis: a. mencapah pada titik itu selepas dipantulkan keluar daripada cermin cembung. diverge after reflected outwards on a convex mirror. b. menumpu pada titik itu selepas dipantulkan ke dalam cermin cekung. converge to after reflected inwards to a concave mirror.
Jarak objek, u Object distance, u
Jarak antara objek dengan titik P. Distance between object and point P.
Jarak imej, v Image distance, v
Jarak antara imej dengan titik P. Distance between image and point P.
Cermin cekung Concave mirror
Paksi utama Principal axis
C
F
P f
R
U N I T
R : Jejari lengkung / Radius of curvature F : Titik fokus / Focal point f : panjang fokus = ½ R / focal length = ½ R
5 Titik focus, F Focal point, F C F
P
Cermin cekung akan memantulkan ke titik F yang selari
Pusat lengkungan, C Centre of curvature, C
© Nilam Publication Sdn. Bhd.
08 FIZ Form 4 Ch5 2F.indd 162
A concave mirror will to point F light
reflect
cahaya
the parallel
162
9/10/13 4:39 PM
MODUL • Fizik TINGKATAN 4
Nota penting/Important notes: Fahamilah konsep ini. Understand this concept.
Y A
C
Objek Object
X
Cermin cekung Concave mirror
T
Bagi cermin cekung, sebarang sinar cahaya yang melintasi pusat lengkungan, C, akan terpantul kembali di sepanjang sinar tuju, kenapa? For a concave mirror, any ray that passes through the centre of curvature, C, will be reflected along its original path (the incident ray). Why is this so?
• Pada X, TXY ialah At X, TXY is the
• CXT =
tangen tangent
kepada bulatan. to the circle.
90°
(i) • ∴ Dari (i) di atas, sinar tuju, ACX, akan menghasilkan sinar terpantul XCA.
U N I T
∴ From (i) above, the incident ray, ACX, will produce the reflected ray XCA.
5
163
08 FIZ Form 4 Ch5 2F.indd 163
© Nilam Publication Sdn. Bhd.
9/10/13 4:39 PM
MODUL • Fizik TINGKATAN 4
Hukum rajah sinar bagi cermin cekung Rules of ray diagram for concave mirror
Sinar/Ray 1 Peraturan/Rule 1: Cahaya selari dengan paksi utama akan terpantul ke F Objek C F Object Cermin cekung Concave mirror
P
Light parallel to principal axis will reflect to F
Sinar/Ray 2
Contoh, f = +10 cm Example, f = +10 cm
Peraturan/Rule 2:
Cermin cekung Concave mirror Objek Object
Cermin cekung Concave mirror Sinar yang selari dengan paksi utama menumpu pada titik yang dipanggil titik fokus (F) pada paksi utama. Ray parallel to the principal axis converges to a point, called the focal point, on the principal axis. PF = panjang fokus, f focal length, f = jarak antara kutub cermin (P) ke titik fokus (F) distance between pole of mirror (P) and focal point (F)
C
F
P
pusat lengkungan Cahaya yang melintasi kembali sepanjang cahaya tuju (garis lurus)
akan terpantul
Light passing through the centre of curvature will reflect back along the incident light (straight line)
Sinar/Ray 3 Peraturan/Rule 3:
Cermin cekung Concave mirror
Objek Object
U N I T
C
F
P
Cahaya yang menuju ke P akan mematuhi hukum pantulan Light passing through to P will laws of reflection
dipantulkan
reflect
dan
and obey the
Sinar/Ray 4
5
Cermin cekung Concave mirror Objek C Object
F
© Nilam Publication Sdn. Bhd.
08 FIZ Form 4 Ch5 2F.indd 164
P
Peraturan/Rule 4: Cahaya yang melintasi F akan dipantul. Sinar cahaya yang terpantul selari adalah dengan paksi utama Light passing through F will be reflected. The reflected ray will be parallel to the principal axis
164
9/10/13 4:39 PM
MODUL • Fizik TINGKATAN 4
(i) Jarak objek: Pada C (u = 2f)
Object distance: At C (u = 2f)
Ciri-ciri imej Characteristics of image
Objek Object C Imej Image
Songsang/Inverted
P
F
Nyata/Real
Cermin cekung Concave mirror
Sama saiz dengan objek Same size as the object
(ii) Jarak objek: Antara C dan F (f < u < 2f )
Object distance: Between C and F (f < u < 2f)
Ciri-ciri imej Characteristics of image
O
C
Songsang/Inverted
P
F
Nyata/Real
Cermin cekung Concave mirror
I
Diperbesarkan/Magnified
(iii) Jarak objek: Antara F dan P (u < f )
Object distance: Between F and P (u < f) I
Ciri-ciri imej Characteristics of image
O C
F
Tegak/Upright Maya/Virtual
P Cermin cekung Concave mirror
Diperbesarkan/Magnified
Kesimpulan/Conclusion: Ciri-ciri imej cermin cekung bergantung pada jarak objek (u) The characteristics of the image in a image concave mirror depends on the object distance (u)
Cermin cekung boleh digunakan: Concave mirrors can be used: 1. Doktor gigi untuk memeriksa gigi pesakit by dentists to examine the teeth of a patient.
U N I T
5
2. Cermin mencukur As shaving mirrors 3. Cermin solek Make-up mirrors
165
08 FIZ Form 4 Ch5 2F.indd 165
© Nilam Publication Sdn. Bhd.
9/10/13 4:40 PM
MODUL • Fizik TINGKATAN 4
Cermin cembung/Convex mirror Sinar yang selari dengan paksi utama seolah-olah terpesong dari satu titik di belakang cermin. Rays parallel to the principal axis appear to diverge from a point behind the mirror.
• Hanya menghasilkan imej maya.
Cermin cembung Convex mirror
Only produce a virtual image. P
F
• Menghasilkan medan penglihatan yang lebih luas.
C
Titik ini yang terletak pada paksi utama dipanggil titik fokus. This point which lies on the principal axis is called the focal point.
Provides a wide field of view.
• Imej yang tegak. Bagi sinar cahaya yang selari dengan paksi utama, cermin cembung akan memantulkan
Image always upright.
• Saiz imej akan mengecil. Size of image will be reduced.
PF = panjang fokus, f PF = focal length, f = jarak antara titik fokus dan kutub cermin (P) = Distance between focal point and the pole of the mirror (P)
cahaya dari F selepas pantulan For light rays parallel to the principal axis, a convex mirror will diverge light from F upon reflection.
Hukum untuk rajah sinar bagi cermin cembung Rules for ray diagrams for convex mirror
Sinar/Ray 2
Sinar/Ray 1
Cermin cembung Convex mirror
Cermin cembung Convex mirror Objek Object
P
F
Objek Object
C
Peraturan/Rule 1: Sinar cahaya selari dengan paksi utama akan memantul dari F Light rays parallel to the principal axis will reflect from F U N I T
5
Sinar/Ray 3
P
pusat lengkungan (C) Sinar cahaya yang melintasi akan dipantulkan semula sepanjang sinar cahaya tuju (garis lurus). Anda harus dapat menerangkan fenomena ini sekarang. Light rays passing through the centre of curvature, C, is reflected back along the incident light rays (straight line). Now you should be able to explain why.
Sinar/Ray 4
P
F
Cermin cembung Convex mirror
C
Objek Object
Peraturan/Rule 3: Sinar cahaya yang mengena P akan terpantul dan mematuhi hukum pantulan Light going through to P will laws of reflection obey the © Nilam Publication Sdn. Bhd.
08 FIZ Form 4 Ch5 2F.indd 166
C
Peraturan/Rule 2:
Cermin cembung Convex mirror Objek Object
F
reflect
and
P
F
C
Panjang fokus, f Focal length, f = FP = 1 × jejari lengkungan 2 radius of curvature = 1 CP 2
Peraturan/Rule 4: Sinar cahaya yang melintasi F yang terpantul adalah selari dengan paksi utama Light rays passing through to F which reflect are parallel to the principal axis
166
9/10/13 4:40 PM
MODUL • Fizik TINGKATAN 4
Rajah sinar untuk menghuraikan imej bagi cermin cembung Ray diagrams to describe the image by a convex mirror
Objek Object
F
P
Imej Image
F
Mengecil/Diminished
Ciri-ciri imej cermin cembung adalah 'DUV' Characteristics of the image for convex mirrors is 'DUV'
Tegak/Upright
U = Tegak/Upright
Ciri-ciri imej
Cermin cembung Convex mirror
Characteristics of image C
D = Mengecil/Diminished V = Maya/Virtual
Maya/Virtual Cermin cembung Convex mirror
Apabila objek mendekati cermin, saiz imej akan membesar When the object is nearer to the mirror, the size of image increases
Ciri-ciri imej Characteristics of image
F
Objek P Object
Imej Image
F
Mengecil/Diminished
C
Tegak/Upright Maya/Virtual
Aplikasi pantulan cahaya
Applications of reflection of light
1 Cermin cembung: Medan penglihatan luas berbanding cermin satah Convex mirror: Has a wider field of view than plane mirror Cermin satah Plane mirror
Cermin cembung Convex mirror
Mata Eye
Mata/Eye
Medan penglihatan sempit Narrow f ield of view
Medan penglihatan luas Wide f ield of view
Boleh digunakan sebagai/Can be used as 1 Cermin memandu/Driving mirrors 2 Cermin sekuriti kedai/Shop security mirrors 3 Cermin selekoh/Blind corner mirrors 2 Cermin parabola/Parabolic mirror Boleh digunakan dalam lampu suluh Can be used in
torches
and
dan
lampu kereta
car headlamps
Can maintain a uniform intensity for a greater distance
Cermin parabola Parabolic mirror
167
08 FIZ Form 4 Ch5 2F.indd 167
as reflectors
5
Sinaran selari/Parallel rays Kelebihan/Advantages: Boleh mengekalkan keamatan seragam untuk jarak yang lebih besar
F Mentol pada titik fokus Bulb at focal point
sebagai pemantul
Sebuah lampu kecil diletakkan pada titik tumpuan cermin untuk menghasilkan sinar yang selari A small lamp is placed at the focal point of the mirror to produce parallel rays
U N I T
© Nilam Publication Sdn. Bhd.
9/10/13 4:40 PM
MODUL • Fizik TINGKATAN 4
Pembinaan alat yang berdasarkan aplikasi pantulan cahaya
The construction of devices which are based on the applications of reflection of light
Periskop/Periscope Cermin Mirror
Objek Object
Fakta Periskop: Facts of a periscope: 1. Dua cermin mestilah selari dan menghadap satu sama lain The two mirrors must be parallel and face each other
Ciri-ciri imej Characteristics of image
Tegak, sama saiz dan maya
2. Kedua-dua cermin diletakkan pada sudut 45° dengan sinar cahaya Both mirrors are set at an angle of 45° to the path of the light rays
Upright, same size and virtual Cermin Mirror
5.2
Mata Eye
MEMAHAMI PEMBIASAN CAHAYA
UNDERSTANDING REFRACTION OF LIGHT
Hukum pembiasan: Laws of refraction: (a) sinar tuju, sinar biasan dan normal berada dalam satah yang sama. the incident ray, the refracted ray and the normal all lie in the same plane. (b) nisbah sin i kepada sin r adalah suatu pemalar, the ratio of sin i to sin r is a constant, sin i = pemalar/constant sin r
Pembiasan ialah/Refraction is: bengkokan sinar cahaya di sempadan apabila ia bergerak dari satu medium ke medium lain yang mempunyai ketumpatan yang berbeza./the bending of a light ray at the boundary as it travels from one medium to another medium of different densities.
Pembiasan cahaya Refraction of light
Udara Air Kaca Glass
Udara Air Kaca Glass
Tiada pembiasan U N I T
5
No
refraction
Sudut tuju Angle of incidence
= 0°
Udara Air Kaca Glass
Normal Normal i i Udara Udara Air Air Kaca Kaca Glass Glass r r
Normal Normal r Udara r i Udara/air Udara/air Air Kaca Kaca Kaca Glass Glass Glass i i
Normal Normal
r
Udara/air
r
Kaca Glass
i
• Sinar merambat dari kaca ke dalam udara.
The ray travels from air into glass.
The ray travels from glass into air.
• Sinar merambat dari medium kurang tumpat ke medium lebih tumpat .
• Sinar merambat dari medium yang lebih tumpat ke medium kurang tumpat .
The ray travels from a less dense medium to a denser medium.
The ray travels from a denser to a less dense medium.
The angle of incidence, i, is greater than the angle of refraction, r.
• Dalam kaca, sinar terbias mendekati normal kerana halaju berkurang . In glass, the ray is bent towards the normal because the velocity decreases .
08 FIZ Form 4 Ch5 2F.indd 168
Normal Normal
• Sinar merambat dari udara ke dalam kaca.
• Sudut tuju, i, lebih besar daripada sudut pantulan, r.
© Nilam Publication Sdn. Bhd.
Normal Normal
medium
lebih besar • Sudut biasan, r, daripada sudut tuju, i. The angle of refraction, r, is than angle of incidence, i.
greater
• Sinar terbias menjauhi dari normal kerana halaju bertambah . The ray is bent away from the normal because its velocity increases .
168
9/10/13 4:40 PM
MODUL • Fizik TINGKATAN 4
Vakum Vacuum
i
Normal Normal
• Sinar cahaya merambat dari medium lebih ke medium tumpat;
tumpat
A light ray travels from less dense medium to denser medium; • Terbias mendekati normal/Refracts towards the normal;
Kaca Glass r
kurang
i
• Sudut tuju, i
lebih besar
daripada sudut pembiasan, r
Angle of incidence, i is bigger than angle of refraction, r r
• Sinar cahaya merambat dari medium
lebih
tumpat ke medium
kurang
tumpat;
denser
A light ray travels from medium to less dense medium; • Terbias menjauhi normal/Refracts away from normal;
• Sudut tuju, i lebih kecil daripada sudut pembiasan, r; Angle of incidence, i is smaller than angle of refraction, r;
Indeks pembiasan dan halaju cahaya Refractive index and speed of light
sin i (vakum atau udara) Mengikut Hukum Snell, indeks pembiasan, n = sin r (medium) sin i (vacuum or air) According to Snell's Law, refractive index, n = sin r (medium)
Latihan/Exercise 60°
1 Tentukan indeks pembiasan kaca.
Determine the refractive index of the glass. Penyelesaian/Solution: sin 60° = 1.51 n= sin 35°
Kaca Glass
Udara Air 35°
Bahan Substance Pepejal/Solid Intan Diamond Kaca Glass Ais Ice Cecair/Liquid Air Water Alkohol Alcohol Benzena Benzene Gas/Gas Udara Air Karbon dioksida Carbon dioxide
Indek pembiasan Refractive index n 2.42 1.46 1.31
1.33 1.36 1.50
1.000293 1.00045
2 Halaju cahaya melintasi medium ialah 1.6 × 108 m s-1. Kirakan indeks pembiasan medium tersebut.
U N I T
5
The speed of light passing through a medium is 1.6 × 108 m s-1. Calculate the refractive index of the medium.
Penyelesaian/Solution:
Halaju dalam vakum/Speed in vacuum, c = 3.0 × 108 m s-1 Halaju dalam medium/Speed in medium, v = 1.6 × 108 m s-1 Halaju cahaya di udara, c Speed of light in air, c n = = Speed of light in medium, v Halaju cahaya dalam medium, v
=
3.0 × 108 m s-1 = 1.88 1.6 × 108 m s-1
169
08 FIZ Form 4 Ch5 2F.indd 169
© Nilam Publication Sdn. Bhd.
9/10/13 4:40 PM
MODUL • Fizik TINGKATAN 4
3 Berapakah halaju cahaya dalam medium dengan indeks pembiasan 2.2? What is the speed of light in a medium with a refractive index of 2.2?
Penyelesaian/Solution: n = 2.2, c = Halaju dalam vakum/Speed in vacuum, c = 3.0 × 108 m s-1 3.0 × 108 m s-1 Halaju cahaya di udara, c Speed of light in air, c = 2.2 n= = Speed of light in medium, v = Halaju cahaya dalam medium, v v v=
3.0 × 108 m s-1 = 1.36 × 108 m s-1 2.2
Fenomena yang melibatkan pembiasan cahaya Phenomena due to refraction of light
Objek kelihatan bengkok dalam air Objects appear to bend in water
Udara Air
Normal Normal
Air Water
Dalam ketara, d, ialah jarak imej dari permukaan air. The apparent depth, d, is the distance of the image from the surface of the water.
Normal Normal
Dalam nyata, D, ialah jarak objek dari permukaan air. Real depth, D, is the distance of the object from the surface of the water.
Dalam ketara, d Apparent depth, d Dalam nyata, D Real depth, D
Imej, I Image, I
U N I T
Objek Object
5 1 Sinar cahaya dari objek
dibiaskan
The light ray from the object is
di sempadan air-udara dan membengkok
refracted
at the water-air boundary and bends
menjauhi
away from
2 Apabila sinar cahaya masuk ke dalam mata, ia seolah-olah datang dari imej, I, yang daripada kedudukan objek. higher
When the light ray reaches the eye, it appears to come from image, I, which is
3 Oleh itu objek seolah-olah berada di kawasan yang The object therefore appears to be © Nilam Publication Sdn. Bhd.
08 FIZ Form 4 Ch5 2F.indd 170
shallower
lebih cetek
normal.
the normal.
lebih tinggi up.
.
.
170
9/10/13 4:40 PM
MODUL • Fizik TINGKATAN 4
Dalam nyata dan dalam ketara Real depth and apparent depth
Normal Normal
Udara Air Air Water
Dalam ketara, d Apparent depth, d
n=
I Dalam nyata, D Real depth, D
di mana n ialah indeks biasan medium where n is the refractive index of the medium
O
menjauhi
1 Sinar cahaya yang merambat dari ikan terbias (medium lebih tumpat) ke udara (medium Rays of light coming from the fish are bent denser less ( medium) to air (
normal apabila ia merambat dari air
kurang
away from
D , d
tumpat). the normal as they passing through from water
dense medium).
2 Apabila sinar cahaya sampai ke mata, ia seolah-olah datang dari ikan maya yang berada di ikan sebenar. When the light rays reach the eye, they appear to come from a virtual fish which is
imej, I
3 Dalam ketara ialah jarak dari permukaan air ke
the real fish.
. image, I
The apparent depth is the distance from the surface of the water to the
4 Dalam sebenar ialah jarak dari permukaan air ke
above
atas
objek, O
. object, O
The real depth is the distance from the surface of the water to the
.
.
5 Hubungan indeks biasan, n, dalam nyata dan dalam ketara ialah: The relationship between of refractive index, n, to real depth and apparent depth is:
n = Dalam nyata Dalam ketara
n=
Real depth Apparent depth
6 Dari rajah di atas, ikan nampaknya muncul di bahagian bawah kolam pada kedudukan 50 cm dari permukaan air. Berapakah kedalaman nyata ikan itu? From the diagram above, the fish at the bottom of a pond appears to be 50 cm from the water surface. What is the real depth of the fish?
U N I T
5
(Pembiasan Indeks air/Refractive index of water = 1.33) Penyelesaian/Solution: Dalam nyata Real depth n = Apparent depth Dalam ketara Dalam nyata Real depth 1.33 = 1.33 = 0.5 m 0.5 m Dalam nyata = 1.33 × 0.5 m Real depth = 1.33 × 0.5 m = 0.67 m = 0.67 m n=
171
08 FIZ Form 4 Ch5 2F.indd 171
© Nilam Publication Sdn. Bhd.
9/10/13 4:40 PM
MODUL • Fizik TINGKATAN 4
Normal Normal
7 Dalam rajah di sebelah, lukiskan rajah sinar dari titik Z ke mata untuk menunjukkan objek kelihatan lebih tinggi.
Imej Image Objek Object
In the diagram on the right, draw a ray diagram from point Z to the eye to show how the object appears higher up.
5.3
Z
MEMAHAMI PANTULAN DALAM PENUH CAHAYA
UNDERSTANDING TOTAL INTERNAL REFLECTION OF LIGHT
Pantulan dalam penuh cahaya ialah Total internal reflection of light is Fenomena yang berlaku hanya apabila cahaya merambat dari medium yang lebih tumpat ke medium yang kurang tumpat dan sudut tuju melebihi sudut genting. A phenomenon which occurs only when light travels from a denser medium to a less dense medium and the angle of incidence is greater than the critical angle.
lebih tumpat ) ke 1 Apabila cahaya merambat dari kaca (medium yang udara (medium yang kurang tumpat ), ia akan terbias menjauhi normal. denser When light travels from glass ( away ( less dense medium), it will bend
2 Sudut biasan, r, adalah The angle of refraction, r, is U N I T
5
medium) to air
lebih besar
daripada sudut tuju, i.
larger
than the angle of incidence, i.
meningkat
When the angle of incidence, i, increases, the angle of refraction, r, until r = 90°.
i
r
Udara Air Kaca Glass
i
increases
4 Apabila sudut biasan, r, menjadi 90°, sinar yang dibiaskan merambat di sepanjang sempadan di antara kaca dan udara. Pada peringkat ini, sudut tuju, i, di dalam medium yang lebih tumpat dipanggil sudut genting, c . When the angle of refraction, r, is 90°, the refracted ray moves along the boundary between glass and air. At this stage, the angle of incidence, i, in the denser critical angle, c . medium is called the
08 FIZ Form 4 Ch5 2F.indd 172
Kaca Glass
from the normal.
3 Apabila sudut tuju, i, meningkat, sudut biasan, r, sehingga r = 90°.
© Nilam Publication Sdn. Bhd.
r
Udara Air
Udara Air Kaca Glass
i=c
172
9/10/13 4:40 PM
MODUL • Fizik TINGKATAN 4
Sudut genting, c, ialah sudut tuju di dalam medium yang lebih tumpat apabila sudut biasan, r, dalam medium yang kurang tumpat ialah 90° The critical angle, c, is the angle of incidence in the denser medium when the angle of refraction, r, in the less dense medium is 90°
5 Apabila sudut tuju, i, adalah lebih besar daripada sudut genting, c, cahaya tidak lagi dibiaskan tetapi semuanya dipantulkan ke dalam semula dan ini mematuhi hukum pantulan .
Udara Air Kaca Glass
When the angle of incidence, i, is larger than the critical angle, c, the light is no refracted reflected longer but is internally and obeys the laws of reflection
i
.
6 Fenomena ini dinamakan This phenomenon is called
pantulan dalam penuh total internal reflection
. .
7 Dua syarat untuk pantulan dalam penuh cahaya berlaku ialah: The two conditions for total internal reflection to occur are:
(a) Cahaya merambat dari medium yang
The light travels from a
Sudut tuju
(b)
The
denser
lebih tumpat
less dense
medium to a
kurang tumpat
.
medium.
sudut genting, c
, i, adalah lebih besar daripada
angle of incidence , i, is greater than the
ke medium
critical angle, c
. .
Kesimpulan Conclusion
❶ Cahaya yang merambat dari medium lebih tumpat ke menjauhi medium kurang tumpat akan dibiaskan
Normal Normal
U N I T
normal. Light rays which travel from a denser medium to a less dense medium are refracted away from the normal.
❷ Apabila sudut pembiasan sama dengan 90°, sudut tuju disebut sudut genting . When the angle of refraction is equal to 90°, the angle of incidence is called the critical angle .
r1 Udara Air
r2
5
Kaca Glass
i1 i2
Pantulan dalam penuh Total internal reflection
❸ Apabila sudut tuju lebih besar daripada sudut genting, pantulan dalam penuh berlaku. When the angle of incidence is more than the critical angle,
173
08 FIZ Form 4 Ch5 2F.indd 173
total internal reflection
occurs.
© Nilam Publication Sdn. Bhd.
9/10/13 4:40 PM
MODUL • Fizik TINGKATAN 4
Hubungan antara sudut genting, c, dengan indeks pembiasan, n The relationship between critical angle, c, and refractive index, n
1 Indeks pembiasan kaca adalah: The refractive index of glass is given as: sin i (dalam udara/in air) sin r (dalam kaca/in glass) sin 90° n= sin c 1 n= sin c
Udara Air
n=
Kaca Glass
c
Contoh/Example Kira sudut genting kaca yang mempunyai indeks pembiasan, n = 1.5. Calculate the critical angle for glass with refractive index, n = 1.5.
Penyelesaian/Solution sin 90° = n sin c sin c = 1 = 1 n 1.5 = 0.6667 c = 41.8°
Latihan/Exercises Rajah menunjukkan satu sinar cahaya, P, ditujukan ke dalam blok kaca. Sudut genting kaca ialah 42°. Ke arah manakah cahaya bergerak dari titik Q? The diagram shows a light ray, P, directed into the glass block. The critical angle of the glass is 42°. In which direction does the light move from point Q? A
U N I T
B
D
Q
5
C
Blok kaca Glass block
42˚ P
Jawapan/Answer: C Penerangan: Sudut tuju = sudut genting = 42°, maka cahaya merambat di sepanjang sempadan kaca-udara Explanation: Incident angle = critical angle = 42°, thus light travels along to the boundary of glass-air
© Nilam Publication Sdn. Bhd.
08 FIZ Form 4 Ch5 2F.indd 174
174
9/10/13 4:40 PM
MODUL • Fizik TINGKATAN 4
Fenomena semula jadi berkaitan pantulan dalam penuh Natural phenomena involving total internal reflection
Logamaya Mirage
Udara sejuk (lebih tumpat) Cool air (denser)
Udara panas (kurang tumpat) Warm air (less dense)
Permukaan panas Hot surface
sangat panas
1 Di padang pasir, tanah (pasir) adalah jadi lapisan udara di permukaan pasir itu kurang tumpat ringan serta .
very hot
In the desert, the land (sand) is
heated up
of air in contact with it gets less dense .
pada waktu siang,
dipanaskan
dan menjadi lebih
during day time, so the layer and becomes lighter and becomes
Cahaya yang merambat dari medium lebih tumpat ke medium kurang tumpat dibiaskan menjauhi normal The light passing through from dense medium to less dense medium is refracted away from normal (MLA = More to Less = Away)
2 Hasilnya, lapisan udara di bahagian atas itu kurang panas dan daripada lapisan di bawahnya. As a result, the successive upper layers of air are not as hot and are
lebih tumpat denser
than those below it.
3 Sinar cahaya dari objek yang jauh, dibiaskan daripada medium lebih tumpat ke medium kurang tumpat. A ray of light from a distance object, gets refracted from a
4 Cahaya yang dibiaskan membengkok away from
The refracted rays bend
At the surface of the ground,
6 Logamaya disebabkan oleh berbeza .
total internal reflection
pantulan dalam penuh
the total internal reflection
175
08 FIZ Form 4 Ch5 2F.indd 175
medium to a
less dense
medium. U N I T
normal.
the normal.
pantulan dalam penuh
5 Dekat permukaan tanah,
The mirage is caused by
menjauhi
denser
5
berlaku. occurs.
cahaya di lapisan udara yang berketumpatan of light at layers of air of
different densities
.
© Nilam Publication Sdn. Bhd.
9/10/13 4:40 PM
MODUL • Fizik TINGKATAN 4
Aplikasi pantulan dalam penuh
Applications of total internal reflection
Periskop
Periscope
Ciri-ciri imej: saiz sama, tegak Characteristics of the image: same size, upright
45°
Kegunaan periskop: Uses of the periscope: Untuk melihat objek di belakang halangan. Dalam kapal selam, periskop digunakan untuk memerhatikan kapal-kapal di permukaan laut To view objects behind obstacles. In submarines, periscopes
45°
are used to observe ships on the surface of the sea
Kelebihan periskop prisma berbanding periskop cermin: Advantages of the prism periscope compared to a mirror periscope: (a) imej lebih terang kerana kesemua cahaya dipantulkan the image is brighter because all the light is reflected (b) imej lebih jelas kerana tiada imej berganda dihasilkan The image is clearer because there are no multiple images formed as in a mirror
Gentian optik Optical fibres
• Gentian optik sangat nipis dan fleksibel. An optical fibre is very thin and flexible.
Lapisan luar Outer cladding
• Gentian optik terdiri daripada satu bungkusan gentian optik. A cable of optical fibres consists of many fine optical fibres bundled together.
Norma Normal
• Gentian optik boleh digunakan dalam sistem gentian-optik seperti endoskop. Optical fibre can be used in a fibreoptic system such as an endoscope.
Teras dalam Inner core Sinar cahaya Light ray Perambatan cahaya dalam gentian optik Propagation of light in an optical fibre
U N I T
5
1 Suatu gentian optik terdiri daripada
teras dalam
indeks biasan kaca yang tinggi dan dikelilingi oleh yang mempunyai indeks biasan yang lebih rendah. inner core An optical fibre consists of outer cladding surrounded by the
yang mempunyai lapisan luar
of high refractive index glass and of lower refractive index.
• Seorang doktor boleh melihat organ dengan menyambung hujung terakhir endoskop, yang dimasukkan ke dalam badan. A doctor can view an organ within the body via one end of the endoscope, which is inserted into the body. • Ia juga digunakan dalam telekomunikasi. It is also used in telecommunications. • Gentian optik adalah ringan, mudah digunakan, murah dan bebas dari pengaruh gelombang elektrik. Optical fibres are light, easily handled, cheap and free from electrical interference signals.
teras dalam 2 Apabila sinar cahaya merambat ke dalam pada satu hujungnya, ia akan merambat secara zig zag di sepanjang gentian dalam satu siri pantulan dalam penuh inner core When a light ray moves into the from one end, it will propagate along the fibre in a zigzag path undergoing a series of total internal reflections
© Nilam Publication Sdn. Bhd.
08 FIZ Form 4 Ch5 2F.indd 176
176
9/10/13 4:40 PM
MODUL • Fizik TINGKATAN 4
5.4
MEMAHAMI KANTA
UNDERSTANDING LENSES
Kanta cekung
Kanta cembung
Concave lens
Convex lens
Kanta cembung = kanta penumpu = kanta positif Convex lens = converging lens = positive lens Kanta cekung = kanta pencapah = kanta negatif Concave lens = diverging lens = negative lens
F
F
f
F
F
f
f
f
Sinar cahaya yang selari dengan paksi utama dibiaskan ke dalam dan menumpu di
Sinar cahaya yang selari dengan paksi utama keluar dibiaskan dan mencapah dari
titik fokus.
titik fokus.
Light rays parallel to principal axis are refracted inwards converge and to the focal point.
Light rays parallel to the principal axis are outwards diverge refracted and from the focal point.
Titik fokus, F/Focal point, F Titik penumpuan di mana semua sinar yang selari dengan paksi utama bertumpu kepadanya (kanta cembung) atau menyimpang daripadanya (kanta cekung) selepas melalui kanta. The concentration point where all rays parallel to the principal axis converge to it (convex lens) or diverge from it (concave lens) after passing through the lens
Panjang fokus, f /Focal length, f Jarak di antara titik fokus dan pusat optik kanta The distance between the focal point and optical centre of the lens
177
08 FIZ Form 4 Ch5 2F.indd 177
• Kanta cembung adalah lebih tebal di tengah-tengah daripada di sisinya. Convex lenses are thicker in the centre than at the edge. • Kanta cekung adalah lebih nipis di tengah-tengah daripada di sisinya. Concave lenses are thinner in the centre than at the edge. • Kanta cembung membiaskan sinar tuju, yang selari dengan paksi utama, oleh itu, ia tertumpu kepada suatu titik di atas paksi utama. Convex lenses refract incident rays of light, which are parallel to the principal axis, so that they are converged, to a point on the principal axis. • Kanta cekung membiaskan sinar tuju yang selari dengan paksi utama dan seolah-olah sinar yang terbias itu mencapah dari titik fokus yang terletak di sebelah yang sama dengan sinar tuju di paksi utama. Concave lenses refract incident rays of light, which are parallel to the principal axis, so that they appear to diverge from a point located on the incident ray of the principal axis.
U N I T
5
© Nilam Publication Sdn. Bhd.
9/10/13 4:40 PM
MODUL • Fizik TINGKATAN 4
Imej terhasil melalui kanta Images formed by lenses
Peraturan untuk rajah sinar Rules for ray diagrams
Kanta cembung
Kanta cekung
Convex lenses
Concave lenses
(a)
(a) Objek Objek Object Object
Objek Object
Objek Object
(b)
(b) F
F
F
F
(c)
(c) F
F
Rajah sinar untuk kanta cembung
Istilah untuk kanta Common terminology of lenses
Ray diagrams for convex lenses
f
Objek Object
U N I T
Characteristics of image
Pusat optik, C Optical centre, C
Songsang/Inverted Nyata/Real Mengecil/Diminished
Pusat geometri kanta. Sinar yang merambat melalui pusat optik kanta dalam garis lurus. The geometric centre of the lens. Rays travelling through the optical centre pass through the lens in a straight line.
Paksi utama Principal axis
Garis lurus yang melalui pusat optik pada sudut tegak (90°) dengan satah kanta. Straight line which passes through the optical centre at right angle (90°) to the plane of the lens.
Titik fokus, F Focal point, F
Titik pada paksi utama yang mana sinar tuju yang selari dengan paksi utama: Is the point on the principal axis to which incident rays of light travelling parallel to the principal axis: a. menumpu daripada titik itu selepas pembiasan melalui kanta cembung converge after refraction through a convex lens. b. mencapah daripada titik itu selepas pembiasan melalui kanta cekung. diverge after refraction through a concave lens.
Jarak objek, u Object distance, u
Jarak antara pusat optik, O, dengan objek. Distance between the optical centre, O, to the object.
Jarak imej, v Image distance, v
Jarak antara pusat optik, O, dengan imej. Distance between the optical centre, O, to the image.
Ciri-ciri imej
(a) u > 2f
F
2F
F
u
v
Imej Image
5 (b) u = 2f
F
F
Ciri-ciri imej Characteristics of image
Songsang/Inverted Nyata/Real Sama saiz dengan objek
f
2F
F
F
u
Imej v Image
© Nilam Publication Sdn. Bhd.
08 FIZ Form 4 Ch5 2F.indd 178
Same size as the object
178
9/10/13 4:40 PM
MODUL • Fizik TINGKATAN 4
(c) f < u < 2f f
Ciri-ciri imej Characteristics of image
Imej Image F
2F
F
Songsang/Inverted Nyata/Real Diperbesarkan/Magnified
2F v
u
(d) u < f f
Ciri-ciri imej Characteristics of image
Imej Image F
2F
Tegak/Upright Maya/Virtual Diperbesarkan/Magnified
F u
v
Rajah sinar untuk kanta cekung Ray diagrams for concave lenses
Objek Object
2F
Objek Object
F Imej Image
F
2F
F Imej Image
F
Ciri-ciri imej bagi kanta cekung adalah sama dengan ciri-ciri cermin cembung The characteristics of the image for concave lens are the same as the characteristics of convex mirrors
Ciri-ciri imej Characteristics of image
Diperkecilkan/Diminished Tegak/Upright Maya/Virtual
DUV D = Mengecil Diminished U = Tegak Upright V = Maya Virtual
Pembesaran
U N I T
5
Magnification
Pembesaran, m Magnification, m
Pembesaran, m Magnification, m
= =
Saiz imej/Size of image hI = Saiz objek/Size of object hO Jarak imej/Image distance v = Jarak objek/Object distance u
179
08 FIZ Form 4 Ch5 2F.indd 179
m=
hI v = hO u
© Nilam Publication Sdn. Bhd.
9/10/13 4:40 PM
MODUL • Fizik TINGKATAN 4
Latihan/Exercises Penyelesaian/Solution:
Rajah menunjukkan kedudukan imej, I, bagi objek, O yang dibentuk oleh kanta cembung. Ketinggian objek, ho ialah 6 cm.
Jarak imejImage distance, v = 70 cm Jarak objek/Object distance, u = 100 cm – 70 cm = 30 cm ho = 6 cm h v m = I = hO u 70 cm hI = 6 cm 30 cm 70 cm hI = × 6 cm 30 cm = 14 cm
The diagram shows the position of an image, I, of an object, O formed by a convex lens. The height of the object, ho is 6 cm. 100 cm I O 70 cm
Berapakah tinggi imej, hI?
What is the height of the image, hI?
Hubungan antara u, v, f
Relationship between u, v and f
1 1 1 + = f u v
Jenis kanta
Panjang fokus, f
Jarak objek, u
Jarak imej, v
Type of lens
Focal length, f
Object distance, u
Image distance, v
Kanta cembung
Nilai positif
Nilai positif
Nilai positif atau negatif
Convex lens
Positive value
Positive value
Positive value or negative value
Kanta cekung
Nilai negatif
Nilai positif
Nilai negatif
Concave lens
Negative value
Positive value
Negative value
Latihan/Exercises U N I T
5
Satu objek dengan tinggi 8 cm diletakkan pada jarak 20 cm dari kanta cekung. Panjang fokus ialah 10 cm. Berapakah (i) jarak imej dan (ii) saiz imej? An object of height 8 cm is placed at a distance of 20 cm from a concave lens. Its focal length is 10 cm. What is (i) the image distance and (ii) size of the image?
Penyelsaian/Solution: (i)
ho = 8 cm u = + 20 cm f = –10 cm 1 1 1 = + f u v 1 1 1 = + v –10 cm 20 cm v = –6.7 cm
© Nilam Publication Sdn. Bhd.
08 FIZ Form 4 Ch5 2F.indd 180
hI v = ho u 6.7 cm × 8 cm hI = 20 cm = 2.68 cm (ii)
180
9/10/13 4:40 PM
MODUL • Fizik TINGKATAN 4
Kegunaan kanta dalam alat optik
The uses of lenses in optical devices
1 Kanta pembesar Jarak objek, u: u < f Ciri-ciri imej: diperbesarkan, tegak, maya
Magnifying glass
Object Distance, u: u < f
I
The characteristics of the image: Magnified, upright, virtual F
O
F
Mata Eye
2 Mikroskop majmuk Compound microscope
Kanta mata Eyepiece
F0 = titik fokus kanta objektif
Kanta objektif Objective lens
focal point of objective lens
Objek Object F0
F0
Fe
I1
Mata Eye
Fe
Fe = titik fokus kanta mata focal point of eyepiece
I2
Ciri-ciri imej pertama, I1
Characteristics of first image, I1
Songsang, nyata, diperbesarkan Ciri-ciri imej terakhir, I2
Inverted, real, magnified
Characteristics of final image, I2
Songsang, maya, diperbesarkan Inverted, virtual, magnified U N I T
3 Teleskop astronomi (digunakan dalam pelarasan normal) Astronomical telescope (used in normal adjustment) Kanta objektif Objective lens
5
Kanta mata Eyepiece lens
Fo
Fe
Cahaya selari dari objek jauh Parallel rays from distant object
Mata Eye
I1 Imej, I1, di infiniti Image, I1, i s formed at infinity
181
08 FIZ Form 4 Ch5 2F.indd 181
© Nilam Publication Sdn. Bhd.
9/10/13 4:40 PM
MODUL • Fizik TINGKATAN 4
Perbandingan antara mikroskop majmuk dan teleskop astronomi
Comparison between a compound microscope and an astronomical telescope
Kesamaan /Similarities kanta cembung
1 Terdiri daripada dua Consists of two
convex lenses
nyata
2 Imej yang pertama adalah The first image is
real
inverted
and
dan bertindak sebagai objek bagi kanta mata
and acts as the object for the eyepiece
magnifying lens
The eyepiece acts as a The final image is
songsang
kanta pembesar
3 Kanta mata bertindak sebagai 4 Imej akhir adalah
dan
maya virtual
, ,
songsang inverted
dan
and
diperbesar
magnified
Perbezaan/Differences Mikroskop majmuk Compound microscope Dua kanta cembung berkuasa tinggi Two high-powered convex lenses
U N I T
5
Jenis kanta dan kuasa kanta Types of lenses and power of lenses
fo < fe
Panjang fokus
Imej pertama adalah besar
Imej pertama
First image is magnified
Kedudukan dekat dengan mata pemerhati At the near point of the observer’s eye
Teleskop astronomi
Aspek
Astronomical telescope
Aspects
Focal length
Kanta cembung berkuasa rendah dan kanta cembung berkuasa tinggi A low powered convex lens and a high powered convex lens
fo > fe
Imej pertama adalah kecil
First image
First image is diminished
Kedudukan imej terakhir
Pada kedudukan infiniti At infinity
Position of final image
Lebih besar/Greater than ( f o + fe) D > ( f o + fe)
Jarak antara dua kanta
m = mo × me
Pembesaran linear, m
© Nilam Publication Sdn. Bhd.
08 FIZ Form 4 Ch5 2F.indd 182
Distance between lenses
(D)
Linear magnification, m
Sama/Equal to ( f o + fe) D = (f o + fe)
m=
fo fe
182
9/10/13 4:40 PM
FORMULA PENTING IMPORTANT FORMULAE Maklumat berikut mungkin berfaedah. Simbol-simbol mempunyai makna yang biasa. The following information may be useful. The symbols have their usual meaning. v-u t
16
n=
sin i sin r
17
n=
dalam nyata dalam ketara
=
real depth apparent depth
1
a=
2
v2 = u2 + 2as
3
s = ut +
4
Momentum = mv
18
5
F = ma
19
Pembesaran linear v m= Linear magnification u
6
Tenaga kinetik 1 = mv2 Kinetic energy 2
20
v = ƒλ
7
Tenaga keupayaan graviti = mgh Gravitational potential energy
21
ax λ= D
22
Q = It
23
E = VQ
24
V = IR
25
Kuasa / Power, P = IV
26
g = 10 m s-2
27
NS VS NP = VP
1 2 at 2
8
Tenaga keupayaan kenyal 1 = Fx Elastic potential energy 2
9
Kuasa, P =
tenaga masa energy Power, P = time
10
m ρ= V
11
F Tekanan / Pressure, p = A
12
Tekanan / Pressure, p = hρg 28
13
Haba / Heat, Q = mcƟ
14
Haba / Heat, Q = mL
15
pV = pemalar / constant T
1 1 1 f = u + v
Kecekapan / Efficiency IS VS = I V × 100% P P
29
E = mc2
30
c = 3.0 × 108 m s-1
© Nilam Publication Sdn. Bhd.
ATAN
TINGK
4
Dw
Edisi Guru
ibahasa
l Modul P&P yang berkesan memudahkan dan mempercepatkan
pengajaran guru.
l Menggunakan kaedah pengurusan grafik yang memudahkan pelajar
memahami dan mengingati konsep-konsep fizik dalam Ingatan Jangka Panjang.
l Modul yang efisien mengandungi nota, latihan dan eksperimen yang
ringkas, lengkap dan padat menyerupai format SPM.
l Mengikut Sukatan Pelajaran Tingkatan 4 yang terkini.
Tajuk-tajuk dalam siri ini:
M O D U L
Matapelajaran
Tingkatan
1
2
3
4
5
SPM
Chemistry Kimia Fizik Biologi Matematik Matematik Tambahan Bahasa Malaysia English Sains Sejarah Geografi
Berminat untuk Menjadi Penulis?... Sila Hubungi:
03-6178 0132
[email protected]
Tingkat 1, No. 35, Jalan 5/10B, Spring Crest Industrial Park 68100 Batu Caves, Selangor, Malaysia. 4EL s &AX WWWNILAMPUBLICATIONCOM s NILAMPUBLICATION YAHOOCOM
WM RM10.95 EM RM10.95